Land MCQS Flashcards

1
Q

A person describes themselves as the owner of a piece of land, which includes a house and garden. What do they own?

A) A right to possess the land

B) The physical land

C) The physical building on the land only

A

A) A right to possess the land

(Correct. We are completely wrong in everyday language when we refer to ourselves as owning a piece of land. The Crown owns all land in England and Wales. What we technically own is a right to possess the land)

How well did you know this?
1
Not at all
2
3
4
5
Perfectly
2
Q

What is the focus of the study of Land law?

A) The physical land

B) The Crown’s ownership of land

C) The sale of property

D) The history of land ownership

E) Rights in the land

A

E) Rights in the land

(Correct. Land law is all about rights in the land, rather than the physical land itself. These rights can be very powerful. Land law is also the study of different land-related relationships, such as landlord and tenant)

How well did you know this?
1
Not at all
2
3
4
5
Perfectly
3
Q

Which of the following pieces of statute is the foundation of modern land law?

A) The Law of Property Act 2002

B) The Law of Property Act 1925

C) The Land Registration Act 2002

D) The Law of Property Act 1066

A

B) The Law of Property Act 1925

(Land law was radically overhauled by a statute called the Law of Property Act 1925 (LPA). This is the reason that the majority of land law has its roots in statute, rather than case law, and the LPA 1925 is the real foundation of modern land law)

How well did you know this?
1
Not at all
2
3
4
5
Perfectly
4
Q

Which of the following options correctly describes the process you must go through to determine if a right being claimed is proprietary or not?

A) Determine if the right is on the list of recognised proprietary rights, whether it satisfies the substantive characteristics, and has been created/acquired in accordance with the formalities for the right

B) Determine whether the right in question has been registered

C) Examine whether the right in question relate to personal or real property

D) Conduct a physical inspection of the land to ascertain whether there is evidence of the right in question

A

A) Determine if the right is on the list of recognised proprietary rights, whether it satisfies the substantive characteristics, and has been created/acquired in accordance with the formalities for the right

(Proprietary rights are very powerful. As a result, these requirements will have to be met for a right to have proprietary status in the circumstances)

How well did you know this?
1
Not at all
2
3
4
5
Perfectly
5
Q

Two landowners are neighbours. One of the landowners grants the other a right to store coal in their coal shed. The grantor of the right sells their house. The new owner asks the neighbour to move the coal from the shed. Which of the following statements best describes the legal position?

A) The neighbour will have to move the coal because such a right can never be enforced against a third party

B) The neighbour will have to move the coal because it is personal property

C) The neighbour does not need to move the coal if the right is an easement and if it has been registered properly at the Land Registry

D) The neighbour does not need to move the coal because it is real property

E) The neighbour does not need to move the coal because they have a proprietary right in the land

A

C) The neighbour does not need to move the coal if the right is an easement and if it has been registered properly at the Land Registry

(The neighbour does not need to move the coal if the right to store is an easement, provided it has been registered at the Land Registry. A right to store is capable of being an easement. Provided that it satisfies the substantive requirements of easements and has been created in accordance with the correct formalities then it will be an easement. If the easement is registered it would then be binding the new landowner)

How well did you know this?
1
Not at all
2
3
4
5
Perfectly
6
Q

Which of the following statements best explains what it means if someone holds a proprietary right in the land?

A) Compensation is available for breach of a proprietary right

B) The right is in personal property

C) The right is capable of being enforced against third parties

D) The right will automatically be enforceable against a new owner of the land

E) The right is enforceable in personam

A

C) The right is capable of being enforced against third parties

(A proprietary right is capable of being enforced against third parties. It will not just be enforceable against the grantor of the right. A proprietary right is not, however, automatically enforceable against a third party)

How well did you know this?
1
Not at all
2
3
4
5
Perfectly
7
Q

A landowner grants their friend a right to live in the landowner’s house for 5 years. One year later the landowner decides they would like the house back. The landowner goes into the house and changes the locks. Which of the following statements correctly explains what it means if the right to live in the house is determined to be a proprietary right known as a lease?

A) If the landowner sold the house then the friend would not be able to enforce the right to live in the house against the new owner

B) The friend would not be able to do anything as the landowner owns the land

C) The friend would have to settle for damages for breach of the right

D) The friend would be able to recover the right to live in the house for the remainder of the term

E) The Crown owns the land and would be able to evict the both the landowner and the friend if it would like to

A

D) The friend would be able to recover the right to live in the house for the remainder of the term

(A lease is a proprietary right which is enforceable in rem ie the right is in the land rather than against the landowner who granted the right. This means if the right is a lease then the friend would be entitled to recover possession and use of the house for the remainder of the lease term)

How well did you know this?
1
Not at all
2
3
4
5
Perfectly
8
Q

Which of the following correctly describes an estate in land?

A) A right to possess the land

B) Physical ownership of the land

C) A right to use or enjoy the land

A

A) A right to possess the land

(We cannot own the physical land, we can just own a right to possess the land. This is known as an estate)

How well did you know this?
1
Not at all
2
3
4
5
Perfectly
9
Q

What is the technical name for a leasehold estate?

A) Fee simple years absolute

B) Fee simple absolute in possession

C) Term of years in possession

D) Term of years absolute

A

D) Term of years absolute

(This is how the leasehold is defined in LPA 1925, s 1(1)(b). However, in practice it is always referred to as the leasehold)

How well did you know this?
1
Not at all
2
3
4
5
Perfectly
10
Q

Does the freeholder need to be physically in possession of a piece of land?

A) Yes, all of the land

B) No

C) Yes, at least part of the land

A

B) No

(Physical possession is not necessary as ‘in possession’ includes receiving rent)

How well did you know this?
1
Not at all
2
3
4
5
Perfectly
11
Q

The particulars of sale for a third floor, two-bedroomed flat in Camden state that the price is £400,000 and there are 125 years left to run on the term. If you bought this flat, what would you be buying?

A) A personal right in the land

B) A leasehold estate

C) The physical land

D) The fee simple

E) A freehold estate

A

B) A leasehold estate

(Though you may think you will be the owner of the flat, what you would actually be the owner of is a 125 year lease of the flat. If you bought this flat, you would have a proprietary right to possess the land for 125 years)

How well did you know this?
1
Not at all
2
3
4
5
Perfectly
12
Q

Which of the following interests in land is only capable of being equitable in nature?

A) Right of entry

B) Beneficial interest behind a trust

C) Easement

D) Mortgage

A

B) Beneficial interest behind a trust

(This is correct. A beneficial interest behind a trust is not a recognised legal interest under LPA 1925, s (1)(2), therefore it can only take effect as an equitable interest under LPA 1925, s 1(3))

How well did you know this?
1
Not at all
2
3
4
5
Perfectly
13
Q

A and B buy a piece of land together each contributing 50% of the purchase price. A is registered at the Land Registry as the legal owner. What interest (if any) does B have in the land?

A) No interest

B) A freehold estate

C) An estate contract

D) An interest under a trust of land

E) A restrictive covenant

A

D) An interest under a trust of land

(This is correct. As B has contributed to the purchase price, but is not a legal owner, B does not have a legal interest in the land (i.e. B does not own a freehold estate) but equity regards B has having a beneficial interest in the land behind a trust. A trust is automatically imposed in these circumstances, with A holding the property on trust for him/herself and B)

How well did you know this?
1
Not at all
2
3
4
5
Perfectly
14
Q

Where are the list of legal interests found?

A) LPA 1925, s 1(3)

B) LPA 1925, s 52

C) LPA 1925, s 1(2)

D) LPA 1924, s 1 (1)

E) LP(MP)A 1989, s 1

A

C) LPA 1925, s 1(2)

(This is correct. S.1(2) of the LPA lists the interests that are capable of being legal interests)

How well did you know this?
1
Not at all
2
3
4
5
Perfectly
15
Q

Which of the following interests in land is only capable of being equitable in nature?

A) Restrictive covenant

B) Mortgage

C) Right of entry

D) Easement

A

A) Restrictive covenant

(This is correct. A restrictive covenant is not a recognised legal interest under LPA 1925, s (1)(2), therefore it can only take effect as an equitable interest under LPA 1925, s 1(3))

How well did you know this?
1
Not at all
2
3
4
5
Perfectly
16
Q

Which of the following statements is true in respect of an interest in land?

A) An interest in land gives the holder of the interest a right to possess the land

B) If the holder of an equitable interest is deprived of their right, they are automatically entitled to damages

C) Only legal interests in land have proprietary status

D) An interest in land is not capable of being enforced against third parties

E) An interest in land can be enforced in rem, this means the right can be recovered if the holder is deprived of it

A

E) An interest in land can be enforced in rem, this means the right can be recovered if the holder is deprived of it

(This is correct. An interest in land is a proprietary right which means it is enforceable in rem and is capable of being enforced against third parties)

How well did you know this?
1
Not at all
2
3
4
5
Perfectly
17
Q

A loans B some money. B grants A rights over their property in exchange for the loan. What interest in the land does A have?

A) A mortgage

B) An easement

C) No interest

D) An interest under a trust of land

E) An estate contract

A

A) A mortgage

(This is correct. A mortgage can be defined as being a loan of cash which is secured by the lender being given rights over the prop offered as security by borrower)

How well did you know this?
1
Not at all
2
3
4
5
Perfectly
18
Q

A agrees to sell a piece of land to B. They enter into a written document which details the agreement and which they both sign. What interest in the land does B have?

A) A restrictive covenant

B) An interest under a trust of land

C) An estate contract

D) No interest

E) A freehold estate

A

C) An estate contract

(This is correct. Put very simply, an estate contract is a contract in land i.e. a contract where the subject matter of the contract is land and that complies with LP(MP)A 1989, s 2 which this does. This is one of the most common examples of an estate contract. When the parties enter into a contract to buy/sell the Land, which is then exchanged, the contract gives the buyer an equitable interest in the land he has contracted to buy)

How well did you know this?
1
Not at all
2
3
4
5
Perfectly
19
Q

Which of the following interests in land is only capable of being equitable in nature?

A) Mortgage

B) Easement

C) Right of entry

D) Beneficial interest behind a trust

A

D) Beneficial interest behind a trust

(This is correct. A beneficial interest behind a trust is not a recognised legal interest under LPA 1925, s (1)(2), therefore it can only take effect as an equitable interest under LPA 1925, s 1(3))

How well did you know this?
1
Not at all
2
3
4
5
Perfectly
20
Q

If the land being transferred is unregistered land, does the transfer need to be registered?

A) Yes the land transfer will trigger the requirement to register the land for the first time

B) No there is never a requirement to register a land transfer

C) No because the legal title transfers at the point at which the deed is completed

D) Yes the registered title will need to be updated

A

A) Yes the land transfer will trigger the requirement to register the land for the first time

(The land transfer will trigger a requirement to trigger the land for the first time. This is called compulsory land registration)

How well did you know this?
1
Not at all
2
3
4
5
Perfectly
21
Q

A buyer and seller enter into a document that is described as a ‘transfer deed’ for the sale of a piece of land. The buyer and seller both sign the deed. The buyer transfers the purchase money and the seller’s solicitor then dates the document. Which of the following statements correctly explains whether the deed is valid?

A) The deed is invalid because it has not been correctly executed

B) The deed is valid because it complies with LP(MP)A 1989, s 2

C) The deed is valid because it complies with LPA 1925, s 52

D) The deed is valid because it complies with LP(MP)A 1989, s 1

E) The deed is invalid because it has not been delivered

A

A) The deed is invalid because it has not been correctly executed

(The deed is invalid. It has been clearly intended as a deed by being described as such, but it has not been correctly executed – although signed by the seller, the seller’s signature must be witnessed and the witness must also sign their name. The deed has been delivered, which was done by dating the document. The deed does not therefore comply with the requirements laid down in LP (MP) A 1989, s.1)

How well did you know this?
1
Not at all
2
3
4
5
Perfectly
22
Q

Which of the 3 stages of transferring a freehold estate is not necessary to effect a legal transfer of the title?

A) Contract

B) Deed

C) Registration

D) All stages are necessary

A

A) Contract

(A contract is not required to legally transfer the land. However, it is usual for the parties to enter into a contract as part of the conveyancing process)

How well did you know this?
1
Not at all
2
3
4
5
Perfectly
23
Q

Which of the following options is the correct statutory authority for the rule that a deed must be used to transfer a freehold estate?

A) LP(MP)A 1989, s 2

B) LPA 1925, s 52

C) LP(MP)A 1989, s 1

D) LRA 2002, s 27

A

B) LPA 1925, s 52

(A legal estate must be transferred or created by deed. LPA, s 52 relates to the transfer of a freehold estate. It also relates to the creation of a leasehold estate and a legal mortgage and a legal easement as you will learn about)

How well did you know this?
1
Not at all
2
3
4
5
Perfectly
24
Q

A buyer and seller of land agree that the buyer will buy a piece of land from the seller for a price of £100,000. They instruct solicitors. The buyer’s solicitor carries out various investigations into the land before the buyer confirms it is happy to proceed. The terms of the agreement are put into a document, which is signed by the buyer and the seller.

Which of the following statement correctly explains the validity of this document?

A) The document is invalid because it has not been witnessed

B) The document is valid because it satisfies the requirements in LRA 2002, s 27

C) The document is invalid because it has not been delivered

D) The document is valid because it complies with LP(MP)A 1989, s 2

E) The document is valid because it complies with LP(MP)A 1989, s 1

A

D) The document is valid because it complies with LP(MP)A 1989, s 2

(The buyer and seller have not entered into a contract to buy/sell the land. The contract is valid because it is in writing, contains all the agreed terms and has been signed by both the buyer and seller. It complies with LP(MP)A 1989, s 2, which is the correct statutory authority)

How well did you know this?
1
Not at all
2
3
4
5
Perfectly
25
Q

If a piece of unregistered land is transferred, at what point is the buyer recognised as being the legal owner of the land?

A) Registration

B) When the purchase money is paid

C) When the land is registered for the first time

D) Exchange of contracts

E) Completion of the transfer deed

A

E) Completion of the transfer deed

(Legal title (ie ownership) transfers at the point of completion if the land being transferred is unregistered)

How well did you know this?
1
Not at all
2
3
4
5
Perfectly
26
Q

If a piece of registered land is transferred, at what point is the buyer recognised as being the legal owner of the land?

A) Exchange of contracts

B) When the land is registered for the first time

C) Registration

D) Completion of the transfer deed

E) When the purchase money is paid

A

C) Registration

(Legal title (ie ownership) transfers at the point at which the buyer is registered at the Land Registry)

How well did you know this?
1
Not at all
2
3
4
5
Perfectly
27
Q

When a land contract is signed, the parties need not sign the same physical copy. The parties can sign different copies of the same document.

Is this statement true or false?

A) True because s 2(1) permits all the terms to be in two documents provided they are identical

B) False the parties must sign the same physical document

C) True because the contractual terms can be incorporated by reference under s 2(2)

A

A) True because s 2(1) permits all the terms to be in two documents provided they are identical

(The statement is true. The agreed land contract can be engrossed in duplicate, with the parties signing different copies of the same document, which are then physically swapped. This is standard conveyancing practice to save time and is known as ‘exchange of contracts’)

How well did you know this?
1
Not at all
2
3
4
5
Perfectly
28
Q

Two people enter into an agreement for lease of land. The agreement refers to various plans and a document which contains a scheme of building works that must be completed by the landlord prior to the commencement of the lease. The agreement is drafted by the seller’s solicitor and signed by both the buyer and seller. The landlord fails to undertake the building works and the prospective tenant refuses to enter into the lease. Which of the following statements best describes whether the landlord can compel the prospective tenant to enter into the lease under the terms of the agreement?

A) The prospective tenant need not enter into the lease because the agreement does not comply with the necessary statutory formalities

B) The prospective tenant must enter into the lease because the plans and scheme of building works are not part of the agreement for lease

C) The prospective tenant need not enter into the lease because the landlord has not complied with plans and scheme of building works, which have been included in the sale agreement by incorporation

D) The prospective tenant need not enter into the lease because the agreement has not been executed correctly

E) The prospective tenant must enter into the lease because the agreement complies with the necessary statutory formalities

A

C) The prospective tenant need not enter into the lease because the landlord has not complied with plans and scheme of building works, which have been included in the sale agreement by incorporation

(The two parties have entered into an agreement for lease, which is a contract to enter into a lease in the future. Such a contract must comply with LP(MP)A 1989, s.2, which it does. The contract is in writing, has been signed by both parties and contains all the agreed terms, including the building works and plans, which have been included by incorporation. This is permitted by LP(MP)A 1989, s 2(2), which provides the contractual terms may be incorporated into the contractual document either by being set out in the document or by referring to some other document. The landlord has not complied with the terms by undertaking the works. Therefore, the other party need not enter into the lease because it is conditional upon these being completed)

How well did you know this?
1
Not at all
2
3
4
5
Perfectly
29
Q

Two neighbouring land owners enter into an agreement in which one of the landowners will offer their neighbour the right to buy the land first if they decide to sell. The landowners subsequently orally decided to extend the period of the agreement. Which of the following statements correctly describes whether the agreement has been validly extended?

A) The agreement has not been extended because the variation does not comply with the necessary statutory formalities

B) The agreement has been extended because the variation complies with correct statutory formalities

C) The agreement has been extended because all parties agreed to the variation

D) The agreement has been extended because the variation does not relate to a material term

E) The agreement has not been extended because a right of pre-emption cannot be varied

A

A) The agreement has not been extended because the variation does not comply with the necessary statutory formalities

(The agreement is a right of pre-emption, which is a type of land contract. In order for a variation to be valid, it must also comply with LP(MP)A 1989, s.2. This was made clear in the case of McCausland v Duncan Lawrie Ltd [1997] 1 WLR 38, where the parties tried to agree an oral variation to the agreed completion date. This variation was held to be void and the court stated that whenever a material term in land contract is varied, that variation must comply with LP(MP)A 1989, s.2. The term of the pre-emption agreement is no doubt a material term)

How well did you know this?
1
Not at all
2
3
4
5
Perfectly
30
Q

If you hold the benefit of a valid land contract, what kind of interest (if any) do you have in the land that is the subject matter of the contract?

A) The contract gives no interest in the land

B) The contract gives a legal interest in the land

C) The contract gives a personal right in the land

D) The contract gives a contractual right in the land

E) The contract gives an equitable interest in the land

A

E) The contract gives an equitable interest in the land

(The effect of a land contract is to pass an equitable interest in the land. This is a proprietary right in the land known technically as an ‘estate contract’)

How well did you know this?
1
Not at all
2
3
4
5
Perfectly
31
Q

A buyer and seller enter into a valid contract for the sale of a piece of land. The seller subsequently decides to pull out of the deal because a higher offer from a third party is received. The buyer still wishes to proceed with the purchase. Which of the following options correctly describes the best advice to the buyer?

A) The buyer should ask the court for the common law remedy of an injunction to force the seller to comply with the contract

B) The buyer should ask the court for an injunction because this would force the seller to comply with the contract

C) The buyer should sue for damages because that is the only remedy available for breach of contract

D) The buyer should ask for the court for an order of specific performance which would compel the seller to comply with the contract

E) The buyer should ask for court for an order of specific performance because this would stop the sale to the third party

A

D) The buyer should ask for the court for an order of specific performance which would compel the seller to comply with the contract

(As the contract for sale grants the buyer a proprietary equitable interest in the land, the buyer is entitled to damages as a right for breach. Given the buyer would like to proceed with the purchase, the buyer should ask the court for an order of specific performance. This is an order which, if granted, would compel the seller to comply with the terms of the contract and sell the land to the buyer. This, like an injunction, is an equitable remedy, which means it will only be granted at the discretion of the court)

How well did you know this?
1
Not at all
2
3
4
5
Perfectly
32
Q

Which of the following correctly describes the requirements of a land contract?

A) Intended as a contract, contains all the agreed terms and is signed by both parties

B) Intended as a contract, executed by the grantor in the presence of a witness who attests their signature and delivered

C) In writing, contains all the terms and is signed by both parties

D) The agreement need not be in writing if the term of the agreement is for less than 3 years and the conditions in LPA, s 54(2) are met

E) In writing, is signed by both parties and is delivered

A

C) In writing, contains all the terms and is signed by both parties

(A land contact must comply with LP(MP)A 1989, s.2. The contract must be in writing, all the terms must be included and the contract must be signed by both parties. There is no requirement that the signatures of both parties be witnessed, unlike the requirements for a deed)

How well did you know this?
1
Not at all
2
3
4
5
Perfectly
33
Q

To which of the following types of document would LP(MP)A 1989, s.2 NOT apply?

A) A five-year lease

B) An option agreement over a field

C) A contract for sale of land

D) A right of pre-emption over a house

E) An agreement for lease

A

A) A five-year lease

(LP(MP)A 1989, s 2 applies to all land contracts, not just a contract for the sale of land. If the subject matter of the agreement is land, s.2 will apply. A lease is not, however, a contract. It is the grant of an estate in land, which must be by deed)

How well did you know this?
1
Not at all
2
3
4
5
Perfectly
34
Q

Which system for proving ownership of land is governed by the LRA 2002?

A) Unregistered Land

B) The Land Registry

C) Registered Land

A

C) Registered Land

(The LRA 2002 governs the registered land system. The system was introduced by the Land Registration Act 1925, but this has not been repealed and replaced in its entirety by the LRA 2002)

How well did you know this?
1
Not at all
2
3
4
5
Perfectly
35
Q

Which of the following options best explains why approximately 15% of land in England and Wales remains unregistered?

A) The land is subject to an overriding interest

B) The land will be in an area which is not designated as being subject to compulsory registration

C) The landowner has chosen not to register the land

D) You only have to register a piece of land if a triggering event occurs eg sale of the freehold estate

A

D) You only have to register a piece of land if a triggering event occurs eg sale of the freehold estate

(From 1926 onwards central government designated various areas as being areas of compulsory registration, so eventually all of England and Wales would be registered. It was not until 1990 that all of England and Wales became designated as an area subject to compulsory registration. The effect of designating an area as being subject to compulsory registration is that if there is then a transaction or event (disposition) with the land which triggers compulsory registration (a triggering event) then the land must be registered for the first time. If there has not be a triggering event with a piece of land it will remain unregistered unless the land owner chooses to make a voluntary first registration application)

How well did you know this?
1
Not at all
2
3
4
5
Perfectly
36
Q

Which of the following options best explains the advantages of the system of Land Registration?

A) The Land Registry offer reduced fees if that land is registered

B) There are no overriding interests which could bind the land

C) Dealing with registered land is very repetitive and so speeds up the process of conveyancing

D) Approximately 75% of land in England and Wales is now registered so solicitors will be more familiar with the registered land system

E) The process of conveyancing is quicker, simpler and ownership is guaranteed by the state

A

E) The process of conveyancing is quicker, simpler and ownership is guaranteed by the state

(A buyer of registered land does not need to investigate numerous title deeds to establish ownership and the rights a piece of land has the benefit/burden of. As a result, it is quicker and simpler to investigate a piece of registered land and speeds up the process of conveyancing. Also, ownership of a registered pieces of land is guaranteed by the state, so there is a mistake with the register then the innocent party who has suffered loss can make a claim for compensation against the Land Registry)

How well did you know this?
1
Not at all
2
3
4
5
Perfectly
37
Q

Producing the bundle of title deeds would prove your ownership of what type of land?

A) Both registered and unregistered land

B) Registered Land

C) Unregistered Land

A

C) Unregistered Land

(This is correct. The historic title deeds, which trace the ownership of property are what you would need to show to prove you own an unregistered piece of land)

How well did you know this?
1
Not at all
2
3
4
5
Perfectly
38
Q

Which of the following options is the correct statutory authority for the requirement to register a piece of land for the first time following the grant of a legal mortgage over the unregistered freehold estate?

A) LRA 2002, s 4

B) LPA 1925, s 27

C) LRA 2002, sch 2

D) LPA 1925, s 4

E) LRA 2002, s 27

A

A) LRA 2002, s 4

(LRA 2002, s 4 lists the transactions over unregistered land that trigger a requirement to register the land for the first time. The grant of a legal mortgage over a piece of unregistered land will trigger a requirement to register the land for the first time)

How well did you know this?
1
Not at all
2
3
4
5
Perfectly
39
Q

Which of the following options correctly lists the dispositions (ie transactions) that trigger a requirement to register the land for the first time?

A) The grant of a 9 year lease, the gifting of a freehold and the grant of a legal mortgage

B) The grant of a 1 year lease, the assent of a freehold and the grant of an equitable mortgage

C) The grant of a 6 year lease, the sale of a freehold and the grant of a legal mortgage

D) The grants of a 7 years lease, the sale of a freehold and the grant of a legal mortgage

E) The grant of a 3 year lease, the sale of a freehold and the grant of an equitable mortgage

A

A) The grant of a 9 year lease, the gifting of a freehold and the grant of a legal mortgage

(Under LRA 2002, s 4 (first registration) and LRA 2002, s 27 (transactions involving already registered land) the circumstances which trigger compulsory registration now include all of these transactions. It is not necessary for there to be a sale of a freehold estate, the gift or assent of a freehold estate will also trigger a registration requirement. The grant of a lease over 7 years must be completed by registration, as must the grant of a legal mortgage)

How well did you know this?
1
Not at all
2
3
4
5
Perfectly
40
Q

Which register in an official copy contains the details of any easements that burden a piece of land?

A) Charges Register

B) Proprietorship Register

C) Property Register

A

A) Charges Register

(This register contains details of any rights that burden a piece of land eg leases, mortgages, restrictive covenants, easements)

How well did you know this?
1
Not at all
2
3
4
5
Perfectly
41
Q

Which register in an official copy contains the description of the piece of land that is registered?

A) Property Register

B) Charges Register

C) Proprietorship Register

A

A) Property Register

(This register contains a description of the piece of land and the details of any rights that benefit a piece of land)

How well did you know this?
1
Not at all
2
3
4
5
Perfectly
42
Q

Which class of title would you to expect to see if the Land Registry is satisfied as to the ownership of the estate and that there is no defect with the title?

A) Possessory Title

B) Good Leasehold

C) Qualified Title

D) Absolute Title

A

D) Absolute Title

(This is the best form of ownership and is really what you would want to see on the official copies if you were buying a property. Most properties are registered with this class of title and it means the Land Registry does not consider there are any defects with the ownership. The land is only bound by interests that are registered on the title or overriding interests)

How well did you know this?
1
Not at all
2
3
4
5
Perfectly
43
Q

Which register in an official copy contains details of the class of title?

A) Charges Register

B) Property Register

C) Proprietorship Register

A

C) Proprietorship Register

(This register contains details of the the class of title such as title absolute, possessory title)

How well did you know this?
1
Not at all
2
3
4
5
Perfectly
44
Q

Which of the following options best explains why the mirror principle of Land Registration has never fully been realised?

A) A buyer of land will have to investigate the equitable ownership of a piece of land in addition to the legal ownership

B) Certain interests do not need to be registered, but will still be binding on a third party

C) Around 15% of land in England and Wales remains unregistered

D) If someone is in possession of a piece of land but has lost the title deeds, they cannot be registered as the legal owner

A

B) Certain interests do not need to be registered, but will still be binding on a third party

(The mirror principle is the ideal that the register should reflect all matters that the property has the benefit of and all the matters that the property is subject to. It should be a clear and comprehensive account of the ownership and rights that benefit and burden a piece of land. However, this has arguably never fully been realised due to the existence of ‘overriding interests’, which are interests that do not need registering and so do not appear on the register, but will still be binding)

How well did you know this?
1
Not at all
2
3
4
5
Perfectly
45
Q

Which of the following statements correctly describes the insurance principle of Land Registration?

A) The accuracy of the register is guaranteed by the state

B) Certain interests do not need to be registered, but will still be binding on a third party

C) A buyer of land will have to investigate the equitable ownership of a piece of land in addition to the legal ownership

D) A buyer of land need not be concerned with the equitable ownership of an estate

E) The register should reflect all matters that the property has the benefit of and all the matters that the property is subject to

A

A) The accuracy of the register is guaranteed by the state

(This is the insurance principle of Land Registration. The accuracy of the register is guaranteed by the state. This is one of the advantages of registered land over unregistered land. Therefore if there is an error the register will be corrected and anyone who has suffered any loss will be compensated (LRA 2002, s 23.) You often see the compensation referred to as ‘state indemnity’)

How well did you know this?
1
Not at all
2
3
4
5
Perfectly
46
Q

Which statutory provision contains the legal definition of what is land?

A) s.1 LPA 1925

B) s.62 LPA 1925

C) s.205(1)(ix) LPA 1925

D) s.52 LPA 1925

A

C) s.205(1)(ix) LPA 1925

(This is correct. The statutory definition includes; the surface of the physical land, building on the land, fixtures and incorporeal hereditaments)

How well did you know this?
1
Not at all
2
3
4
5
Perfectly
47
Q

Which of the following is not included within the definition of land?

A) Fittings

B) Buildings

C) Mines and minerals

D) Fixtures

E) The benefit of an easement

A

A) Fittings

(This is correct. Fitting are not included within the definition of land under LPA, 205(1)(ix))

How well did you know this?
1
Not at all
2
3
4
5
Perfectly
48
Q

Which of the following options correctly explains why you might need to determine if something is land or not?

A) If you are selling a piece of land, you need to know what you are entitled to take and what must be left for the buyer as everything classed as land passes to the buyer under LPA 1925, s 62

B) If you are buying a piece of land, you need to know what is included in the purchase as everything that counts as land is included under LPA 1925, s 52

C) If a lender takes possession proceedings and exercises it rights to sell, you need to determine whether to use a deed or not

D) You must use a contract to sell anything that counts as land under LPA 1925, s 52

A

A) If you are selling a piece of land, you need to know what you are entitled to take and what must be left for the buyer as everything classed as land passes to the buyer under LPA 1925, s 62

(This is correct. This is particularly relevant in the context of fixtures/fittings. If an item is a fixture, it is part of the land and must therefore be left for the buyer. However, if something is a fitting, the seller can take it)

How well did you know this?
1
Not at all
2
3
4
5
Perfectly
49
Q

Which of the following statements correctly explains the current position at law in respect of the airspace above an owner’s piece of land?

A) A landowner owns the lower airspace above his/her land

B) A landowner only owns the surface of the land upon which the buildings rest. It does not own any of the air above it.

C) A landowner owner the upper airspace above his/her land

D) A landowner owns everything up to the heaven’s above

A

A) A landowner owns the lower airspace above his/her land

(This is correct. A land owner has rights in what is classed as the lower airspace above his/her land)

How well did you know this?
1
Not at all
2
3
4
5
Perfectly
50
Q

A satellite dish is overhanging onto an adjoining piece of land, though is not causing any damage to the land. Which of the following options correctly explains whether this is trespass?

A) Yes the satellite dish would be committing trespass as it is in the upper airspace above the land

B) No the satellite dish is not damaging the land, therefore it is not committing trespass

C) No the satellite dish would not be committing trespass as it is in the upper airspace above the land.

D) Yes the satellite dish would be committing trespass as it is in the lower airspace above the land

A

D) Yes the satellite dish would be committing trespass as it is in the lower airspace above the land

(This is correct. The satellite dish would be classed as trespassing into the lower airspace of the land. The lower airspace is the area above the physical land which is reasonably necessary for a landowner’s use and enjoyment of the land, it does not matter that no damage is being caused to the land. The facts of this case are similar to Kelsen v Imperial Tobacco Co)

How well did you know this?
1
Not at all
2
3
4
5
Perfectly
51
Q

Which of the following statements correctly explains the current position at law in respect of the ground below an owner’s piece of land?

A) A landowner owns the ground under the surface to depths above 300 metres

B) A landowner owns only the surface of the land

C) S/He who owns the land owns everything to the depths below.

D) A landowner owns up to depths of 600 metres under the Infrastructure Act 2015, s43

A

A) A landowner owns the ground under the surface to depths above 300 metres

(This is correct. The Infrastructure Act 2015, s43 effectively states there is no trespass at depths below 300 metres)

How well did you know this?
1
Not at all
2
3
4
5
Perfectly
52
Q

Within the definition of land under LPA 1925, s.205 (1)(ix) a fixture would be classed as which of the following?

A) A corporeal hereditament

B) An incorporeal hereditament

C) A privilege derived from land

D) Chattel

A

A) A corporeal hereditament

(This is correct. Corporeal hereditaments are things attached/fixed to the land, what we call fixtures in practice)

How well did you know this?
1
Not at all
2
3
4
5
Perfectly
53
Q

Which of the following statements correctly describes the legal position in respect of fixtures?

A) An item not attached to the land can never be a fixture

B) An item not attached to the land can be a fixture if it part of the architectural design of a building

C) If something is attached to the land it will always be a fixture

D) Fixtures will only pass to a buyer of land if they are included in the sale

A

B) An item not attached to the land can be a fixture if it part of the architectural design of a building

(This is correct. If chattels are incorporated into the architectural design of a building, they may consequently be classified as fixtures even though they are not firmly affixed. This can be seen in D’Eyncourt v Gregory (1866) LR 3 Eq 382 where a stone garden seat and ornamental statues standing on their own weight were held to be fixtures as forming part of the architectural design of the house and its grounds)

How well did you know this?
1
Not at all
2
3
4
5
Perfectly
54
Q

A picture is attached to the wall of a house. Which of the following statements explains the legal position as to whether the picture is likely to be a fixture or not?

A) Pictures will always be fixtures if they are attached to the wall

B) The degree of annexation test will lead to a presumption that the picture is a fixture because it can be removed without damage to the wall

C) It will depend on the circumstances of the case. If the picture can be removed easily and put in place to enjoy as pictures, it is likely to be a chattel

D) Pictures will never be fixtures because they are personal to the owner for the time being

E) If the purpose of attaching the picture to the wall was to improve the building, this indicates the picture is a chattel

A

C) It will depend on the circumstances of the case. If the picture can be removed easily and put in place to enjoy as pictures, it is likely to be a chattel

(This is correct. Such a situation is analogous to the facts of Berkley v Poulett [1977] 1 EGLR 86, and as Lord Justice Scarman states: “if ordinary skill was used, as it was, in their removal, they could be taken down, and in the event were taken down, without much trouble and without damage to the structure of the rooms. The decisive question is therefore as to the object and purpose of their affixing. …I find, applying the second test, that the pictures were not fixtures. They were put in place on the wall to be enjoyed as pictures.”)

How well did you know this?
1
Not at all
2
3
4
5
Perfectly
55
Q

Which test prevails in determining if an item is a fixture or chattel?

A) The degree of annexation test

B) The purpose of annexation test

C) Both tests are equally decisive

A

B) The purpose of annexation test

(This is correct. The ‘purpose test’ prevails over the ‘degree test’ where there is a discrepancy between them (Holland v Hodgson (1872) LR 7 CP 328))

How well did you know this?
1
Not at all
2
3
4
5
Perfectly
56
Q

What is the most common type of mortgage arrangement today?

A) Sharia – compliant

B) Capital and interest repayments

C) Interest only

D) Endowment

E) Pension

A

B) Capital and interest repayments

(Correct. Each month the borrower pays an instalment which consists partly of repayment of capital and partly interest on the outstanding balance)

How well did you know this?
1
Not at all
2
3
4
5
Perfectly
57
Q

Which of the following statements is correct?

A) The mortgagor is the borrower

B) The mortgagee is the borrower

C) The mortgagor is the lender

D) A mortgage is a personal right

A

A) The mortgagor is the borrower

(This is correct. The mortgagor is the borrower, who grants the mortgage as security for the loan)

How well did you know this?
1
Not at all
2
3
4
5
Perfectly
58
Q

Which of the following statements correctly describes the likely outcome for a low risk borrower?

A) A bank is unlikely to loan money to a low risk borrower

B) They will be charged a high interest rate

C) They will be charged a low interest rate

A

C) They will be charged a low interest rate

(When the borrower is a low risk party, s/he will usually be charged a low interest rate)

How well did you know this?
1
Not at all
2
3
4
5
Perfectly
59
Q

Which one of the following statements is true?

A) A mortgage is capable of being a legal interest in land only

B) A mortgage is capable of being an equitable estate in land

C) A mortgage is capable of being an equitable interest in land only

D) A mortgage is capable of being a legal or equitable interest in land

E) A mortgage is capable of being a legal estate in land

A

D) A mortgage is capable of being a legal or equitable interest in land

(Correct. See LPA 1925 ss 1(2)(c) and 1(3))

How well did you know this?
1
Not at all
2
3
4
5
Perfectly
60
Q

If a mortgage intended to be legal is not created properly, what is the consequence?

A) The mortgage will fail as an interest in land

B) The mortgage will be equitable if it is in writing and signed by the borrower

C) The mortgage will be equitable provided that it is in writing which contains all the terms and is signed by both parties

A

C) The mortgage will be equitable provided that it is in writing which contains all the terms and is signed by both parties

(Correct. A failed legal mortgage will be treated as a contract to create a legal mortgage if it complies with LP(MP) A 1989, s 2)

How well did you know this?
1
Not at all
2
3
4
5
Perfectly
61
Q

What formalities must be complied with in order to create a mortgage over an equitable interest in land?

A) Deed and substantive registration

B) Contract

C) In writing signed by the mortgagor

D) Deed

E) Contract and registration

A

C) In writing signed by the mortgagor

(Correct. A mortgage over an equitable interest need only be in writing and signed by the grantor (LPA 1925, s 53(1)(c)))

How well did you know this?
1
Not at all
2
3
4
5
Perfectly
62
Q

When might an option for the lender to purchase the mortgaged property be upheld?

A) When it is granted in a substantially separate transaction

B) When it is granted in a separate document.

C) When it is granted at a later date than the mortgage.

D) When the parties have equal bargaining power

A

A) When it is granted in a substantially separate transaction

(Correct. Warnborough V Garmite said that the whole transaction must be looked at to see if it is substantially a mortgage or not. If it is a mortgage, the option will not be allowed and vice versa)

How well did you know this?
1
Not at all
2
3
4
5
Perfectly
63
Q

A mortgage loan contains a a legal date of redemption which falls 2 months before the end of a 25 year mortgage term. Which one of the following rights which make up the equity of redemption is likely to be relied upon here by the mortgagor to have the clause thrown out by the courts?

A) The equitable right to redeem

B) No collateral advantages

C) No postponement or prevention of redemption
to redeem is rendered valueless.

A

C) No postponement or prevention of redemption

(Correct. The equitable right to redeem does not arise until 6 weeks before the end of the mortgage term. Courts look at clauses which postpone the legal date for redemption very closely and would consider whether, on the facts, the right to redeem is rendered valueless)

How well did you know this?
1
Not at all
2
3
4
5
Perfectly
64
Q

When does the legal right to redeem arise?

A) On the day after the legal date for redemption has passed

B) On the date set out in the mortgage deed

C) Immediately the mortgage deed is signed

A

B) On the date set out in the mortgage deed

(Correct. The legal right exists for one day, and is the date set out in the mortgage deed. Review the materials on the equitable right to redeem)

How well did you know this?
1
Not at all
2
3
4
5
Perfectly
65
Q

In Fairclough v Swan Brewery the court examined a clause which postponed the legal date for redemption. What was the outcome of the clause?

A) The clause was upheld because once the loan was repaid, the borrower would get back what he had mortgaged and during the mortgage he had had the benefit of a low interest rate

B) The clause was upheld because it was a commercial bargain made between two experienced business parties

C) The clause was struck out because once the loan was repaid, the borrower would get back an estate which was worth much less than at the time of the mortgage

A

C) The clause was struck out because once the loan was repaid, the borrower would get back an estate which was worth much less than at the time of the mortgage

(Correct. The borrower would not get back an unencumbered leasehold which was exactly what he had mortgaged. A leasehold estate is a depreciating asset, and a lease with only 6 weeks left to run, is very different to a lease with 17 years left to run)

How well did you know this?
1
Not at all
2
3
4
5
Perfectly
66
Q

Which one of the following statements on solus ties is true?

A) The solus tie will be permitted in a commercial transaction if the tie ends before or at the end of the mortgage term

B) The solus tie will not be permitted in a commercial transaction if the tie extends beyond the mortgage term even if it is a genuinely separate transaction

C) The solus tie will be permitted in a commercial transaction where the tie ends after the mortgage term ends

A

A) The solus tie will be permitted in a commercial transaction if the tie ends before or at the end of the mortgage term

(Correct. This is the decision in Biggs v Hoddinot)

How well did you know this?
1
Not at all
2
3
4
5
Perfectly
67
Q

One of the situations where undue influence may arise is where there is a relationship of influence of which unfair advantage is taken. There are a number of relationships where there is an irrebuttable presumption that one party has influenced the other. Which ONE of the following is NOT within that number?

A) Solicitor and client

B) Trustee and beneficiary

C) Husband and wife

D) Parent and child

E) Doctor and patient

A

C) Husband and wife

(Correct. In cases where undue influence is claimed, it will not be presumed but will need to be positively shown)

How well did you know this?
1
Not at all
2
3
4
5
Perfectly
68
Q

In addition to a relationship of trust and confidence, what else must be shown for a claim of undue influence to succeed?

A) A transaction which involves fraud

B) A transaction which requires explanation

C) A transaction which involves misrepresentation

A

B) A transaction which requires explanation

(Correct. If there is a relationship of trust and confidence the transaction must be one that initiates the question ‘why has X signed this?’ If it is, undue influence is established)

How well did you know this?
1
Not at all
2
3
4
5
Perfectly
69
Q

In Barclays Bank v O’Brien, why was the bank unable to enforce its charge against Mrs O’Brien?

A) Mr O’Brien had unduly influenced his wife to sign the charge without ensuring she was fully informed

B) The bank had constructive notice of Mr O’Brien’s undue influence and failed to take reasonable steps to ensure that Mrs O’Brien was fully informed.

C) The bank had unduly influenced Mrs O’Brien into signing the charge without ensuring she was fully informed

A

B) The bank had constructive notice of Mr O’Brien’s undue influence and failed to take reasonable steps to ensure that Mrs O’Brien was fully informed

(Correct. The bank had notice of the potential risk of undue influence and should have ensured that Mrs O’Brien fully understood what she was signing)

How well did you know this?
1
Not at all
2
3
4
5
Perfectly
70
Q

Which case sets any guidance as to the steps a lender should take to avoid being fixed with constructive notice of undue influence?

A) Barclays Bank v O’Brien

B) RBS v Etridge

C) CIBC v Pitt

A

B) RBS v Etridge

How well did you know this?
1
Not at all
2
3
4
5
Perfectly
71
Q

In RBS v Etridge, the court extended the scope of the principles of constructive notice of undue influence. Which one of the following is the most accurate statements of the effect of this case?

A) The principles are relevant where the relationship between the person claiming undue influence and the debtor is one of husband and wife.

B) The principles are relevant in every case where the relationship between the person claiming undue influence and the debtor is non-commercial

C) The principles are relevant where the relationship between the person claiming undue influence and the debtor is one of husband and wife and civil partners

A

B) The principles are relevant in every case where the relationship between the person claiming undue influence and the debtor is non-commercial

(Correct. The idea of constructive notice now extends far beyond family and spousal relationships)

How well did you know this?
1
Not at all
2
3
4
5
Perfectly
72
Q

Where a borrower has created three legal mortgages, in favour of three different lenders, on consecutive days, how do we work out which takes priority?

A) Priority between legal charges depends on the order specified in the mortgage deeds

B) Priority between legal charges depends on the order in which they have been registered

C) Priority between legal charges depends on the order in which the deeds are executed

A

B) Priority between legal charges depends on the order in which they have been registered

(Correct. This is the rule in LRA 2002, s48)

How well did you know this?
1
Not at all
2
3
4
5
Perfectly
73
Q

Where a borrower has created three equitable mortgages, in favour of three different lenders, on consecutive days, how do we work out which takes priority?

A) Priority between equitable mortgage depends on the order in which they are created

B) Priority between equitable charges depends on the order specified in the mortgage deeds.

C) Priority between equitable mortgages depends on the order in which they are protected by registration

A

A) Priority between equitable mortgage depends on the order in which they are created

(Correct. Equitable mortgages do not have to be registered in order to be created properly. LRA 2002, s28 which relates to equitable interests, says that equitable interests take effect in the order in which they are created)

How well did you know this?
1
Not at all
2
3
4
5
Perfectly
74
Q

An equitable mortgage can be (but does not have to be) protected by entering a s32 Notice on the Charges Register. If an equitable mortgage has been protected, what does that mean in terms of priority?

A) It takes priority over all equitable mortgages even those which are created before it

B) It takes priority over ALL subsequent mortgages, whether legal or equitable

C) It takes priority over ALL subsequent equitable mortgages only

A

B) It takes priority over ALL subsequent mortgages, whether legal or equitable

(Correct. LRA 2002, s29(2) says that an interest which is protected takes priority over all subsequent transactions)

How well did you know this?
1
Not at all
2
3
4
5
Perfectly
75
Q

The legal lender has the right to sue the borrower in person for the contractual debt. What is the limitation period for a contractual debt for a legal mortgage?

A) 12 years for capital and 6 years for interest

B) 6 years for capital and interest.

C) 6 years for capital and 12 years for interest.

D) 12 years for capital and interest

A

A) 12 years for capital and 6 years for interest

(This is correct. The limitation period for a contractual debt is not straightforward: if the mortgage has been created by deed, as all legal mortgages must be, then the period for recovery of the debt stated in the deed (the capital) is twelve years. The limitation period for recovery of interest is six years. What this means in reality is that borrowers can find themselves involved in contractual debt actions many years after repossession, when they may have thought that their troubles were behind them)

How well did you know this?
1
Not at all
2
3
4
5
Perfectly
76
Q

What is the purpose of the Pre-Action Protocol 2008?

A) It sets out an expectation that lenders will explore alternative arrangements with borrower before taking possession of residential or commercial properties

B) It sets out an expectation that lenders will explore alternative arrangements with borrower before taking possession of commercial properties

C) It enables lenders to enter residential or commercial premises to prepare them for sale

D) It enables the lender of a commercial property to manage it rather than sell it straight away

E) It sets out an expectation that lenders will explore alternative arrangements with a borrower before taking possession of residential properties

A

E) It sets out an expectation that lenders will explore alternative arrangements with a borrower before taking possession of residential properties

(Correct. It was intended to reduce the number of residential properties which were being repossessed and the number of families losing their homes. Lenders agreed that possession should be a last resort)

How well did you know this?
1
Not at all
2
3
4
5
Perfectly
77
Q

What is the consequence if a lender exercises its power of sale under a legal mortgage, but the sale proceeds are insufficient to discharge the mortgage debt?

A) The lender cannot recover the shortfall

B) The lender may sue the borrower in contract for the outstanding debt

C) The lender can recover a shortfall of capital but not interest by suing the borrower in contract for the outstanding debt

A

B) The lender may sue the borrower in contract for the outstanding debt

(Correct. The right to sell is a proprietary remedy which coexists with the contractual remedy which does not extinguish it)

How well did you know this?
1
Not at all
2
3
4
5
Perfectly
78
Q

Foreclosure is rarely used by lenders, yet it does have an advantage for a borrower who is in serious debt and is in ‘negative equity’. What is that advantage?

A) The foreclosure process takes a long time and buys the borrower time to make alternative arrangements

B) The court can award a sale in lieu of foreclosure

C) All subsequent mortgages and the contractual debt are extinguished

A

C) All subsequent mortgages and the contractual debt are extinguished

(Correct. The borrower cannot be sued personally either by the lender who forecloses or any subsequent lender)

How well did you know this?
1
Not at all
2
3
4
5
Perfectly
79
Q

To whom does a receiver owe a duty to act with due diligence?

A) The borrower

B) The court

C) The lender

A

A) The borrower

(Correct. This may seem odd, as the receiver is appointed by the lender, but it means that the lender is not liable for the receiver’s negligence)

How well did you know this?
1
Not at all
2
3
4
5
Perfectly
80
Q

Why might a lender choose to apply to the court for an order for possession?

A) To avoid possible criminal proceedings following allegations of use of force to gain entry to premises

B) The Pre-Action Protocol of 2008 requires an order for possession to be obtained

C) To avoid allegations of trespass

A

A) To avoid possible criminal proceedings following allegations of use of force to gain entry to premises

(Correct. The order for possession gives the lender authority to possess and protect the lender from criminal proceedings under Criminal Law Act 1977, s6)

How well did you know this?
1
Not at all
2
3
4
5
Perfectly
81
Q

The court has jurisdiction under AJA 1970, s 36 in relation to applications relating to a mortgaged property by the lender. Which of the following best describes the scope of s 36?

A) It enables a court to prevent a lender from exercising its power of sale

B) It enables a court to postpone possession of residential and commercial properties

C) It enables a court to postpone possession of a residential property where an order has been applied for by the lender

D) It enables a court to prevent a lender from possessing residential and commercial properties

E) It enables a court to postpone possession even in cases where no possession order has been applied for

A

C) It enables a court to postpone possession of a residential property where an order has been applied for by the lender

(Correct. The court can postpone possession of residential properties only and has discretion as to timing and conditions)

How well did you know this?
1
Not at all
2
3
4
5
Perfectly
82
Q

AJA 1970, s 36 enables a court to postpone an order for possession provided that the borrower can pay ‘any sums due’ within a ‘reasonable period’. Which of the following best explains this?

A) The borrower must pay the arrears and accrued interest within 24 months of the arrears arising

B) The borrower must pay the whole of the mortgage debt including arrears on a date set by the court before the end of the term

C) The borrower must pay the arrears before the end of the mortgage term

A

C) The borrower must pay the arrears before the end of the mortgage term

(Correct. This generous interpretation of the effect of s 36 is set out in AJA 1973, s 8 and Cheltenham & Gloucester Building Society v Norgan)

How well did you know this?
1
Not at all
2
3
4
5
Perfectly
83
Q

When does the mortgagee acquire the right to possess mortgaged property?

A) Immediately the mortgage deed is signed

B) When the court has granted order for possession

C) When a mortgage payment is missed

D) When interest is in arrears for two months

A

A) Immediately the mortgage deed is signed

(Correct. This is set out in Four Maids v Dudley Marshall and LPA 1925, s95(4). It is set out expressly in most mortgage deeds, although there will be an acknowledgement by the lender that it will not exercise the right whilst payments are met)

How well did you know this?
1
Not at all
2
3
4
5
Perfectly
84
Q

Will a court postpone an order for possession to allow the borrower to sell the property?

A) Yes. Courts prefer borrowers to sell property themselves to save costs and ensure the sale proceeds quickly

B) No. Courts are suspicious of potential delaying tactics by the borrower and will not postpone for this reason

C) Yes, if there is firm evidence of an imminent exchange of contracts

D) No. Courts must act in the lender’s best interests once arrears have arisen

A

C) Yes, if there is firm evidence of an imminent exchange of contracts

(Correct. This was decided in Mortgage Services Funding plc v Steele. Simply instructing a solicitor will not be enough)

How well did you know this?
1
Not at all
2
3
4
5
Perfectly
85
Q

When does the lender’s rights to sell arise, where the mortgage is a capital and interest repayment mortgage?

A) As soon as one monthly instalment has become payable

B) When the legal date of redemption has passed

C) As soon as the ink is dry on the mortgage deed

A

A) As soon as one monthly instalment has become payable

(Correct. LPA 1925, s 101(1) says that the right arises when the mortgage money ‘has become due’. In an interest-only mortgage, that will happen on the legal redemption date; in a capital and interest repayment mortgage, that happens when any payment of capital has become due)

How well did you know this?
1
Not at all
2
3
4
5
Perfectly
86
Q

In which ONE of the following circumstances does the right to sell become exercisable under LPA 1925, s 103?

A) Three months after the lender has served notice requesting arrears to be paid

B) When some of the interest is unpaid for two months or more

C) When two months’ interest payments are outstanding

A

B) When some of the interest is unpaid for two months or more

(Correct. LPA 1925, s 103(ii) states that some interest must be outstanding for two months. It does not mean that two months’ worth of arrears is owed! The amount can be very small)

How well did you know this?
1
Not at all
2
3
4
5
Perfectly
87
Q

A property has two charges secured on it. The second lender sells under the right of sale. Which of the following represents the correct procedure?

A) The second loan is redeemed first, then the first loan, with any surplus being paid to the borrower

B) The first loan is redeemed first, then the second loan with any surplus being paid to the borrower

C) The first loan is redeemed first, then the second loan, and any surplus proceeds are divided proportionately between the first lender and the second lender

A

B) The first loan is redeemed first, then the second loan with any surplus being paid to the borrower

(Correct. The first lender takes priority and is paid first, even though the second lender actually sold. The surplus proceeds are held in trust of the first borrower)

How well did you know this?
1
Not at all
2
3
4
5
Perfectly
88
Q

The lender owes the borrower a duty to take reasonable care to obtain the true market value for the property. Which of the following is the most accurate statements of what ‘true market value’ means in this context?

A) The lender cannot be expected to achieve the perfect sale price, but the price must be in the correct bracket

B) The lender owes a duty to carry out repairs and improvements to the property to obtain the highest possible price

C) The lender must obtain the maximum possible price in that particular market but need not delay sale

A

A) The lender cannot be expected to achieve the perfect sale price, but the price must be in the correct bracket

(Correct. This was the decision in Michael v Miller)

How well did you know this?
1
Not at all
2
3
4
5
Perfectly
89
Q

When selling, which of the following statements best expresses the lender’s duty as to the timing of the sale in a slow market?

A) The lender has an unfettered discretion as to when to sell and need not delay

B) The lender must wait a reasonable time for an upturn in the market

C) The lender must sell immediately the right to sell becomes exercisable

D) The lender must delay the sale if there is any prospect of planning permission being granted

A

A) The lender has an unfettered discretion as to when to sell and need not delay

(Correct. This was decided in Silven Properties Limited v Royal Bank of Scotland plc)

How well did you know this?
1
Not at all
2
3
4
5
Perfectly
90
Q

Part of C’s land is sold by C to D. In the transfer deed there is no right mentioned for C to continue to use the drain which runs under land which now belongs to D. Which phrase best describes this situation?

A) express reservation

B) a licence

C) implied grant

D) implied reservation

E) express grant

A

D) implied reservation

(This is correct. The right is not in writing and must therefore be implied. It is a reservation as it is a right which C has kept back for himself out of the land sold to D)

How well did you know this?
1
Not at all
2
3
4
5
Perfectly
91
Q

Which statutory provision states that an easement granted for three years is capable of being legal?

A) Law of Property Act 1925, s 1(1)(a)

B) Law of Property Act 1925, s 1(3)

C) Law of Property Act 1925, s 1(2)(a)

D) Law of Property Act 1925, s 1(2)(c)

E) Law of Property Act 1925, s 1(1)(b)

A

C) Law of Property Act 1925, s 1(2)(a)

(This is correct. This section states that an easement granted or reserved for the equivalent of a term of years absolute is capable of being a legal interest in land)

How well did you know this?
1
Not at all
2
3
4
5
Perfectly
92
Q

What is the correct term for a benefit which confers on the holder a right to take something from the land? e.g. fish or minerals

a licence

a quasi-easement

C) a profit a prendre

D) an easement

E) a covenant

A

C) a profit a prendre

(This is correct. A profit a prendre confers on the holder of the right to take anything, such as produce, animals, fish, or minerals, from the land)

How well did you know this?
1
Not at all
2
3
4
5
Perfectly
93
Q

Part of A’s land is sold by A to B. In the transfer deed, B is given a right of way over A’s land. Which phrase best describes this situation?

A) an implied reservation grant

B) a licence

C) an express reservation

D) an implied grant

E) an express grant

A

E) an express grant

(This is correct. The right is express as it is in writing, and it is a grant as it is a right which A creates in favour of B)

How well did you know this?
1
Not at all
2
3
4
5
Perfectly
94
Q

For an easement to be created by prescription, how many years of uninterrupted user would the claimant need to demonstrate?

A

20

(Easements may arise by prescription or ‘long use’. Generally an easement is claimed by prescription where it has been exercised over land for a long time (at least 20 years), yet no express grant or reservation can be traced)

How well did you know this?
1
Not at all
2
3
4
5
Perfectly
95
Q

Can a tenant acquire an easement by prescription if it exercises a right without force, secrecy and permission for a 20 year uninterrupted period?

A

No

(Although the right has been exercised for over 20 years, a prescriptive right can only be created as between two freehold owners)

How well did you know this?
1
Not at all
2
3
4
5
Perfectly
96
Q

The freehold owner of a property is concerned about building works taking place on neighbouring land. The building works will block the sunlight to the garden.

Could the freehold owner have an easement for a right to light in the circumstances?

A) No, there is no defined aperture in a garden

B) Yes, if the freehold owner can show 20 years of uninterrupted use of the right

C) Yes, there is a general right to light

A

A) No, there is no defined aperture in a garden

(There is no general right to light. A right to light must be via a defined aperture eg a window. Rights to light do not attach to gardens or open land)

How well did you know this?
1
Not at all
2
3
4
5
Perfectly
97
Q

Which one of these is a negative easement?

A) a right of parking

B) a right to enjoy a garden

C) a right of way

D) a right of drainage

E) a right to light

A

E) a right to light

(This is correct. A negative easement is an easement which is enjoyed from the dominant owner’s own land and does not involve entering or using the servient land at all)

How well did you know this?
1
Not at all
2
3
4
5
Perfectly
98
Q

Part of C’s land is sold by C to D. In the transfer deed there is no right mentioned for C to continue to use the drain which runs under land which now belongs to D. Which phrase best describes this situation?

A) express grant

B) implied grant

C) express reservation

D) a licence

E) implied reservation

A

E) implied reservation

(This is correct. The right is not in writing and must therefore be implied. It is a reservation as it is a right which C has kept back for himself out of the land sold to D)

How well did you know this?
1
Not at all
2
3
4
5
Perfectly
99
Q

A prescriptive easement must be exercised ‘nec vi, nec clam, nec precario’ - without force, without secrecy and without permission.

If a right of way is exercised for 20 years by a freeholder against a freeholder but in breach of a sign on the servient land saying “private, for use of patrons only” will the right have been acquired by prescription?

A) No, the right has been exercised with the use of force

B) Yes, the right has been exercised ‘nec vi, nec clam, nec precario’

C) No, the right has been used with permission

D) No, the right has not be used without secrecy

E) No, the right has not been exercised for the requisite period

A

A) No, the right has been exercised with the use of force

(This is correct. A prescriptive easement must be exercised without force. Force includes the removal of obstructions, or ignoring the protests of the servient owner, which would include the ignoring of signage)

How well did you know this?
1
Not at all
2
3
4
5
Perfectly
100
Q

In the case of P&A Swift Investments Ltd v Combined English Stores Group plc, Lord Oliver sets out a detailed test for establishing whether a right accommodates the dominant tenement. Which one of the following options is NOT part of that test?

A) The right is not expressed to be personal to the dominant owner

B) The right should benefit the dominant land owner even if they were to sell the dominant land

C) The right must not simply benefit a business on the dominant land.

D) The right only benefits the owner whilst that person owns the dominant land.

E) The right makes the dominant land more valuable

A

B) The right should benefit the dominant land owner even if they were to sell the dominant land

(This is correct. Lord Oliver’s test in essence requires the easement to benefit the land itself, not simply the dominant owner personally and actually, this means that the easement should cease to be of any benefit to the dominant land owner if they sold the dominant land)

How well did you know this?
1
Not at all
2
3
4
5
Perfectly
101
Q

Which of the following rights has NOT been judicially recognised as an easement?

A) A right to use a swimming pool

B) A right to park

C) A right to drainage

D) A right to a scenic view

E) A right of way

A

D) A right to a scenic view

(Judicial recognition is an essential element which must be satisfied if a right is to be capable of being an easement. The court is willing to recognise new easements if they are analogous to existing recognised easements, but a right to a scenic view has been specifically discounted from being an easement because it it not capable of reasonably exact definition - how do you begin to define ‘a scenic view’?!)

How well did you know this?
1
Not at all
2
3
4
5
Perfectly
102
Q

Which case sets out the criteria which must be met for a right to be capable of being an easement?

A) Hill v Tupper

B) Borman v Griffith

C) P&A Swift Investments Ltd v Combined English Stores Group plc

D) Tulk v Moxhay

E) Re Ellenborough Park

A

E) Re Ellenborough Park

(This is correct. This case sets out the four criteria which must be met in order for an easement to be capable in principle of being an easement)

How well did you know this?
1
Not at all
2
3
4
5
Perfectly
103
Q

What types of rights may in future be recognised as new types of easements?

A) Negative easements only

B) No new easements can be recognised: the list is closed.

C) Positive or negative easements

D) Positive easements only

A

D) Positive easements only

(This is correct. New negative easements would unduly restrict the use of the servient land: Phipps v Pears)

How well did you know this?
1
Not at all
2
3
4
5
Perfectly
104
Q

Which one of these is not one of the criteria set out in re Ellenborough Park as essential for a right to be capable of an easement?

A) The right must lie in grant

B) The right must accommodate the dominant tenement

C) There must be a dominant and servient tenement

D) There must be no common ownership of the dominant and servient tenements

E) The use of the right must not amount to exclusive possession of the servient land

A

E) The use of the right must not amount to exclusive possession of the servient land

(This is correct. A right which passes the tests in re Ellenborough Park will nevertheless be disqualified on this basis)

How well did you know this?
1
Not at all
2
3
4
5
Perfectly
105
Q

In which of the following cases was a right of way judicially recognised as being capable of being an easement?

A) Regency Villas v Diamond Resorts

B) Attwood v Bovis Homes

C) Phipps v Pears

D) Re Ellenborough Park

E) Borman v Griffith

A

E) Borman v Griffith

(This is correct. Judicial recognition is an essential element which must be satisfied if a right is to be capable of being an easement)

How well did you know this?
1
Not at all
2
3
4
5
Perfectly
106
Q

Which of the following is the most accurate summary of the ‘ouster principle’?

A) The easement must not rob the servient owner of ultimate possession and control of the servient land

B) The easement must not obstruct the servient owner’s normal use of the servient tenement

C) The easement must not leave the servient owner with no reasonable use of the servient tenement

A

C) The easement must not leave the servient owner with no reasonable use of the servient tenement

(This is correct. The ouster principle is sometimes called the reasonable user test)

How well did you know this?
1
Not at all
2
3
4
5
Perfectly
107
Q

Which of the following options correctly describes the disqualifying factors which will prevent a right from being an easement?

A) If the right does not comply with the correct formalities for creation, it will be disqualified from being an easement

B) If the right is not granted for a certain term, it will be disqualified from being an easement

C) If the right grants exclusive possession to the dominant land owner, requires payment by the servient land owner or requires permission from the servient land owner everytime it is exercised, it will not be an easement

D) If the right grants exclusive possession to the servient land owner, requirement payment by the dominant land owner or requires permission from the dominant land owner everytime it is exercised, it will not be an easement

E) If the right does not grant exclusive possession to the dominant land owner, it will be disqualified from being an easement

A

C) If the right grants exclusive possession to the dominant land owner, requires payment by the servient land owner or requires permission from the servient land owner everytime it is exercised, it will not be an easement

(This is correct. A right cannot be an easement if the user amounts to exclusive possession by the servient owner, if is requires payment by the servient land owner or if permission is required to exercise the right)

How well did you know this?
1
Not at all
2
3
4
5
Perfectly
108
Q

Which of the following rights will be disqualified because of exclusive possession?

A) A right to store in part of the shed where the servient land owner is prevented from accessing the shed

B) A right to store in all of the shed on the servient land owner’s land which the servient land owner retains access to

C) A right to park in one of five spaces all day/any day

D) A right to use a heated swimming pool

A

A) A right to store in part of the shed where the servient land owner is prevented from accessing the shed

(This is correct. The servient land owner is not left with reasonable use or possession and control of the shed, it will be disqualified on the facts)

How well did you know this?
1
Not at all
2
3
4
5
Perfectly
109
Q

A man grants a woman a right of way over their lane. The lane is in a very poor state of repair, making it almost impossible to drive across.

Which one of the following statements is correct?

A) The man need not repair the lane but must allow the woman access to do the repairs

B) The man must repair the lane but can claim a proportion of the money back from the woman

C) The man must repair the lane but as this will mean that the man has to spend money, the right cannot be an easement

A

A) The man need not repair the lane but must allow the woman access to do the repairs

(This is correct. There is no obligation on the servient owner to carry out repairs: Jones v Pritchard)

How well did you know this?
1
Not at all
2
3
4
5
Perfectly
110
Q

Which of the following rights will be disqualified because permission is required?

A) A right to use a tennis court where the servient land owner can stop the dominant land owner from playing so that they can use the court

B) A right to use a heated swimming pool

C) A right to store in all of a shed where the servient land owner cannot access the shed

D) A right to park at the weekend only

E) A right to use a tennis court where the servient land owner gives an initial permission but then the dominant land owner does not have to ask again

A

A) A right to use a tennis court where the servient land owner can stop the dominant land owner from playing so that they can use the court

(This is correct. The essence of an easement is that it is enjoyed as of right. If the dominant land owner has to stop exercising their right when asked by the servient land owner, the right will be disqualified for permission)

How well did you know this?
1
Not at all
2
3
4
5
Perfectly
111
Q

If a right fails as an express legal easement for lack of formalities, what is the position?

A) The right will automatically be recognised as an equitable easement

B) The right may be recognised as an equitable easement if it is in writing, signed by both parties and contains all the agreed terms

C) The right will be a personal licence only

D) The right may be recognised as an equitable easement if it is in writing and signed by the grantor

A

B) The right may be recognised as an equitable easement if it is in writing, signed by both parties and contains all the agreed terms

(This is correct. A failed legal easement may be treated as an enforceable contract to create a legal easement if the failed legal easement complies with the formalities for an estate contract in the Law of Property (Miscellaneous Provisions) Act 1989, s 2)

How well did you know this?
1
Not at all
2
3
4
5
Perfectly
112
Q

What are the correct formalities to create an easement for an uncertain term?

A) It must be created in writing, signed by the grantor and witnessed

B) It must be created in writing, signed by the grantor only

C) It must be created in writing, signed by both parties.

D) It must be created by deed

A

B) It must be created in writing, signed by the grantor only

(This is correct. These are the formalities for a basic equitable interest set out in Law of Property Act 1925, s 53(1)(a))

How well did you know this?
1
Not at all
2
3
4
5
Perfectly
113
Q

What formalities must be met in order to create a legal easement of 3 years?

A) No formalities as the easements is created for 3 years or less

B) Contract

C) Deed only (registration is not required as the easement will be an overriding interest)

D) Deed + registration

A

D) Deed + registration

(This is correct. LPA 1925, s 52 and LRA 2002, s 27(2)(d) respectively provide a deed must be used, which must then be substantively registered)

How well did you know this?
1
Not at all
2
3
4
5
Perfectly
114
Q

Which is the definition of a legal easement?

A) An easement granted in a transfer deed will be legal, an easement granted in a lease will be equitable

B) To be legal, an easement must be granted for an uncertain term

C) An easement must be granted for a certain term and grant exclusive possession to the dominant land owner

D) To be legal, an easement must be granted it reserved for the equivalent of a freehold or leasehold term

E) An easement must be granted by deed otherwise it can only be equitable

A

D) To be legal, an easement must be granted it reserved for the equivalent of a freehold or leasehold term

(This is correct. Section 1(2)(a) of the LPA 1925 makes this clear)

How well did you know this?
1
Not at all
2
3
4
5
Perfectly
115
Q

Which statutory provision sets out the type of document which must be used to create an express legal easement?

A) Law of Property (Miscellaneous Provisions) Act 1989, s 2

B) Law of Property (Miscellaneous Provisions) Act 1989, s 1

C) Law of Property Act 1925, s 53(1)(a)

D) Law of Property Act 1925, s 52

A

D) Law of Property Act 1925, s 52

(This is correct. An express legal easement must be created by deed)

How well did you know this?
1
Not at all
2
3
4
5
Perfectly
116
Q

Which of the following rights can be acquired by necessity?

A) Rights of way

B) Rights to park

C) Rights of storage

D) Rights of drainage and sewerage

A

A) Rights of way

(This is correct. ‘Necessity’ is strictly construed and any other rights, including to use drains and sewers are not absolutely necessary for the enjoyment of land, so the claim would fail)

How well did you know this?
1
Not at all
2
3
4
5
Perfectly
117
Q

Sweet v Sommer softened the harsh requirements of ‘necessity’ for easements implied by necessity. Which of these statements accurately states how the case expanded the definition?

A) It recognised a right of drainage as being implied by necessity

B) It recognised a right of way by vehicle as being implied by necessity, even though there was already a right of way on foot and the land was not inaccessible

C) It recognised rights which were agreed informally but mistakenly omitted from the transfer deed as being implied by necessity

D) It recognised a right of way on foot as being implied by necessity, even though there was already a right of way by vehicle and the land was not inaccessible

A

B) It recognised a right of way by vehicle as being implied by necessity, even though there was already a right of way on foot and the land was not inaccessible

(This is correct. The case appears to expand the scope to allow rights of way for land which is not actually landlocked)

How well did you know this?
1
Not at all
2
3
4
5
Perfectly
118
Q

Which is the most accurate definition of ‘necessity’ in the context of easements implied by necessity?

A) ‘Highly convenient’: the right is amongst the rights a landowner would normally expect to enjoy with land of this nature

B) ‘Necessary for the comfortable enjoyment of the land’: the land can be put to better use with the easement

C) ‘Absolute necessity’: the land cannot be used at all without the easement

A

C) ‘Absolute necessity’: the land cannot be used at all without the easement

(This is correct. This is the test which was applied in Manjang v Drammeh and Pryce v McGuinness.)

How well did you know this?
1
Not at all
2
3
4
5
Perfectly
119
Q

For acquisition by common intention to work, what does there need to be?

A) A lease to imply the easement into

B) A common intention that the premises be used for residential use

C) A specific common purpose in respect of use of the land that cannot be achieved without the easement

D) Land that is otherwise landlocked without the easement

A

C) A specific common purpose in respect of use of the land that cannot be achieved without the easement

(This is correct. The right must be essential to the agreed common purpose as to use otherwise common intention cannot be used)

How well did you know this?
1
Not at all
2
3
4
5
Perfectly
120
Q

Which one of the following statements is NOT a requirement for an easement to be implied by the common intention of the parties?

A) The contemplated use of the land must be very specific and not simply generic domestic or commercial use

B) The original parties must have been landlord and tenant

C) The premises cannot be used for the contemplated purpose without the easement

D) There must be a common purpose or use for the dominant land, which is known to both parties

A

B) The original parties must have been landlord and tenant

(This is correct. This method of implying easements does not require the original parties to be landlord and tenant. This method would also work if one party had bought part of the other’s land, instead of leasing it)

How well did you know this?
1
Not at all
2
3
4
5
Perfectly
121
Q

In which situation will acquisition by common intention NOT work?

A) A right to park claimed by a tenant who occupies premises under a lease granted for a car garage. The premises cannot be used as car garage without the right to park

B) A right of storage claimed by a tenant who uses the land as a restaurant

C) Rights of drainage and sewerage where land sold and transferred is clearly intended to be developed for residential use

D) A right of way required so that an expressly granted right to use a garage can be exercised

A

B) A right of storage claimed by a tenant who uses the land as a restaurant

(This is correct. Premises can be used as a restaurant without storage, the right is not necessary to achieve the common intention - use of the premises as a restaurant)

How well did you know this?
1
Not at all
2
3
4
5
Perfectly
122
Q

What types of easements can be implied by the common intention of the parties?

A) Grants and reservations

B) Grants only

C) Reservations only

A

A) Grants and reservations

(This is correct. Whilst some methods of implied acquisition operate only on grants, this method is not so restricted)

How well did you know this?
1
Not at all
2
3
4
5
Perfectly
123
Q

One of the criteria for the rule in Wheeldon v Burrows to operate is that the right must be necessary for the reasonable enjoyment of the land. Which of these statements is the best definition of ‘necessary’ in this context?

A) The right is absolutely necessary for the land to be used

B) The right is necessary for the convenient and comfortable enjoyment of the land

C) The right increases the value of the land.

D) The right benefits the dominant land owner even if they part with the dominant land

E) The right is essential for the contemplated use of the land

A

B) The right is necessary for the convenient and comfortable enjoyment of the land

(This is correct. The definition of the word ‘necessity’ in this context is much more relaxed than in cases on easements implied by necessity or by common intention of the parties. The right must enhance the land in some way)

How well did you know this?
1
Not at all
2
3
4
5
Perfectly
124
Q

Into what type of documents may easements be implied by the rule in Wheeldon v Burrows?

A) Licences

B) Deeds or contracts

C) Deeds only

D) Contracts only

A

B) Deeds or contracts

(This is correct. Wheeldon v Burrows has a wider scope than Law of Property Act 1925, s 62 in that it operates on contracts as well as deeds)

How well did you know this?
1
Not at all
2
3
4
5
Perfectly
125
Q

What type of rights can be implied as easements under the rule in Wheeldon v Burrows?

A) Reservations only

B) Grants only

C) Grants and reservations

A

B) Grants only

(This is correct. Courts are reluctant to ‘derogate from grant’: in other words they are reluctant to acknowledge a right which was not specifically reserved by the dominant landowner, whether that person let or sold the servient land)

How well did you know this?
1
Not at all
2
3
4
5
Perfectly
126
Q

What is the essential pre-requisite for the operation of the rule in Wheeldon v Burrows?

A) There must be a pre-existing landlord and tenant situation where the landlord occupies one part of the land and the tenant the other

B) There must be a quasi-easement exercised for any part of the land

C) There must be a pre-existing landlord and tenant situation where both parts of the land are occupied by different tenants

D) There must be a quasi-easement exercised for the benefit of the land now owned by the claimant

A

D) There must be a quasi-easement exercised for the benefit of the land now owned by the claimant

(This is correct. There must have been a single owner who enjoyed the right now claimed as an easement, for the benefit of that part of the land which is now the dominant tenement. The benefit becomes an easement once the ownership of the land is in different hands)

How well did you know this?
1
Not at all
2
3
4
5
Perfectly
127
Q

Which of the following is NOT one of the criteria which is needed for Wheeldon v Burrows to imply an easement into a document?

A) There must be a quasi-easement

B) There must be some evidence on the servient land of the enjoyment of the right

C) The right must be necessary for the reasonable enjoyment of the land

D) The enjoyment of the right must be regular

E) The right must touch and concern the dominant land

A

E) The right must touch and concern the dominant land

(This is correct. This concept is part of the rules on capability which must be fulfilled before we even consider whether the right has actually been acquired)

How well did you know this?
1
Not at all
2
3
4
5
Perfectly
128
Q

Which one of the following statements is correct in stating the situations when Law of Property Act 1925, s 62 may apply to imply an easement?

A) The section operates in situations where land is sold or let for the first time, and in situations where part of the land has previously been occupied by a tenant but the lease is renewed or that land sold

B) The section operates in situations where part of the land has previously been occupied by a tenant but the lease is renewed or that land sold

C) The section operates on quasi-easements to imply an easement into a transfer deed or lease where the land is let for the first time

A

A) The section operates in situations where land is sold or let for the first time, and in situations where part of the land has previously been occupied by a tenant but the lease is renewed or that land sold

(This is correct. Although case law had suggested that the section only operated where there had previously been a landlord and tenant situation, it is now settled law that the section also operates on quasi-easements to imply an easement into a transfer deed or lease where the land is let for the first time)

How well did you know this?
1
Not at all
2
3
4
5
Perfectly
129
Q

What is the most accurate statement of the ‘ordinary operation’ of Law of Property Act 1925, s 62?

A) s62 defines what ‘land’ means.

B) s62 passes the benefit of existing easements to a new owner of the dominant land

C) s62 upgrades permissions to full legal easement when there is a transaction by deed

A

B) s62 passes the benefit of existing easements to a new owner of the dominant land

(This is correct. The section operates as a word-saving section and makes it clear that any existing rights and easements will form part of the dominant land and will automatically pass to the new owner on a sale)

How well did you know this?
1
Not at all
2
3
4
5
Perfectly
130
Q

Into what type of documents may easements be implied by Law of Property Act 1925, s 62?

A) Deeds and contracts

B) Licences

C) Deeds only

D) Contracts only

A

C) Deeds only

(This is correct. Law of Property Act 1925, s 62 has a narrower scope than the rule in Wheeldon v Burrows in that it only operates on deeds)

How well did you know this?
1
Not at all
2
3
4
5
Perfectly
131
Q

What type of rights can be implied as easements under Law of Property Act 1925, s 62?

A) Reservations only

B) Grants and reservations

C) Grants only

A

C) Grants only

(This is correct. Courts are reluctant to ‘derogate from grant’: in other words they are reluctant to acknowledge a right which was not specifically reserved by the dominant landowner, whether that person let or sold the servient land)

How well did you know this?
1
Not at all
2
3
4
5
Perfectly
132
Q

Land is divided for the first time, by transfer deed. A right of way is claimed by the buyer of the land under s 62 LPA 1925.

Which statement best explains if the buyer will be successful in the circumstances?

A) The buyer will be successful providing the land is otherwise landlocked

B) The buyer will be successful because there is a conveyance ie the transfer deed to imply the easement into

C) The buyer will be successful providing the right is continuous and apparent

D) The buyer will be successful providing the right of way is more convenient than any other access way

A

C) The buyer will be successful providing the right is continuous and apparent

(This is correct. Where there is no prior diversity of occupation (ie the land is being divided for the first time, as it is here by the transfer deed) then the right must be continuous and apparent. This means the right must have been used reasonably regularly and must be obvious on reasonable inspection of the land)

How well did you know this?
1
Not at all
2
3
4
5
Perfectly
133
Q

Which one of the following registration processes is needed to make an implied legal easement enforceable?

A) Entry of a Notice in the charges register

B) No registration is necessary because legal easements ‘rule the world’

C) Substantive registration

D) No registration is necessary providing the dominant land owner is in actual occupation of the servient land

E) No registration is necessary it will be an overriding interest (providing the conditions are met)

A

E) No registration is necessary it will be an overriding interest (providing the conditions are met)

(This is correct. Implied legal easements operate as overriding interest provided that one of the conditions in Land Registration Act 2002, Schedule 3, paragraph 3 is met)

How well did you know this?
1
Not at all
2
3
4
5
Perfectly
134
Q

An implied legal easement is an overriding interest: Land Registration Act 2002, Schedule 3, paragraph 3 provided that one of the conditions set out in paragraph 3 are met. Which one of the following is not a condition in paragraph 3?

A) The easement must have been exercised within twelve months of the transfer of the servient land

B) The easement must be held by someone who is in actual occupation of the servient land

C) The easement must be obvious on a reasonable inspection of the servient land.

D) The easement must be within the actual knowledge of the new owner

A

B) The easement must be held by someone who is in actual occupation of the servient land

(This is correct. This provision is not listed as a condition in Schedule 3, paragraph 3. It relates to equitable interests which are held by people in actual occupation of the land in question. Although Chaudhary v Yavuz suggests obiter that an equitable easement could be an overriding interest based on actual occupation, there is no direct authority on this)

How well did you know this?
1
Not at all
2
3
4
5
Perfectly
135
Q

Which statutory provision passes the benefit of an existing easement to a new dominant owner, thereby enabling that person to enforce the easement?

A) Law of Property Act 1925, s 62

B) Law of Property Act 1925, s 52

C) Law of Property Act 1925, s 205(ix)

D) Law of Property Act 1925, s 78

A

A) Law of Property Act 1925, s 62

(This is correct. The ordinary operation of s 62 operates as a word-saving provision, stating amongst other things that all rights privileges and easements which benefit the dominant land automatically pass on a transfer of that land)

How well did you know this?
1
Not at all
2
3
4
5
Perfectly
136
Q

If an equitable easement is not protected by a Notice in the Charges Register of the servient title in accordance with Land Registration Act 2002, s 32, what effect does that have on a new owner of the servient land?

A) The easement is not binding on any new owner of the servient land

B) The easement may still be binding as an overriding interest providing the dominant land owner is in actual occupation of the land

C) The easement is not binding on a purchaser for value of the dominant land

D) The easement is not binding on a purchaser for value of the servient land

A

D) The easement is not binding on a purchaser for value of the servient land

(This is correct. Buyers and lenders are purchasers for value, and will take the land free of an unprotected equitable easement, although it will still bind a volunteer (or donee), which is someone who is gifted or inherits the land)

How well did you know this?
1
Not at all
2
3
4
5
Perfectly
137
Q

Which one of the following registration processes is needed to make an express legal easement over registered land enforceable?

A) No registration is necessary as legal easements ‘bind the world’

B) Entry of a Restriction in the proprietorship register

C) Entry of a Notice in the charges register

D) No registration is necessary the easement will be an overriding interest

E) Substantive registration

A

E) Substantive registration

(This is correct. An express legal easement must be substantively registered in order to be legal, it is part of the process to create the legal easement. Once substantively registered it will appear in the charges register of the servient land and will automatically bind a new owner of that land)

How well did you know this?
1
Not at all
2
3
4
5
Perfectly
138
Q

What is the legal issue in relation to freehold covenants?

A) Whether the covenant is enforceable by and against successors in title to the original parties

B) Whether the covenant is enforceable between the original parties

C) Whether the covenant is personal or not

D) Whether the covenant is positive or negative

A

A) Whether the covenant is enforceable by and against successors in title to the original parties

(The legal problem here is that there is no contractual relationship between successors in title to the original parties. We look at whether the relevant rules have been observed to enable a successor in title to the covenantee to enforce a covenant direct against a successor in title to the covenantor)

How well did you know this?
1
Not at all
2
3
4
5
Perfectly
139
Q

In order for a successor covenantee to enforce a covenant against a successor covenantor, what must the successor covenantee show?

A) That the benefit of the covenant has passed to the successor covenantee and that the burden of the covenant has passed to the successor covenantor

B) That the covenant was binding on the original parties

C) That there is a contractual relationship between the successor covenantee and the successor covenantor

D) That the benefit of the covenant has passed to the successor covenantor and the burden passed to the successor covenantor

A

A) That the benefit of the covenant has passed to the successor covenantee and that the burden of the covenant has passed to the successor covenantor

(In order for the covenant to be enforced by a successor covenantee, they must show that they have the benefit of the covenant to enable them to sue. In order to enforce the covenant against a successor covenantor they must show that the successor covenantor has the burden of the covenant to enable them to be sued)

How well did you know this?
1
Not at all
2
3
4
5
Perfectly
140
Q

Which is the correct definition of ‘covenantor’?

A) The person who receives the benefit of the covenant and owns the servient land

B) The person who receives the benefit of the covenant and owns the dominant land

C) The person who enters into the covenant and owns the servient land

D) The person who enters into the covenant and owns the dominant land

A

C) The person who enters into the covenant and owns the servient land

(This is correct. The covenantor is the person who enters into the covenant and is subject to the burden of the promise. That person’s land is the burdened or servient land)

How well did you know this?
1
Not at all
2
3
4
5
Perfectly
141
Q

Which is the key factor about the covenantee?

A) The covenantee can sue if the covenant is breached

B) The covenantee must observe the terms of the covenant

C) The covenantee can be sued if the covenant is breached

A

A) The covenantee can sue if the covenant is breached

(The covenantee owns the land which has the benefit of the covenant and is able to enforce the covenant if it is breached)

How well did you know this?
1
Not at all
2
3
4
5
Perfectly
142
Q

What is a positive covenant?

A) A covenant which obliges the owner of the land to do something

B) A covenant which restricts what can be done on the land

C) A covenant which decreases the value of the benefitted land

D) A covenant which increases the value of the burdened land

A

A) A covenant which obliges the owner of the land to do something

(This is correct. A positive covenant obliges the covenantor to “put their hand in their pocket” and do something to comply with the covenant, either spend money or energy. The covenant cannot be complied with by inaction)

How well did you know this?
1
Not at all
2
3
4
5
Perfectly
143
Q

What must the successor in title to the dominant land show in order to enforce the covenant?

A) That the benefit of the covenant has passed to them

B) That they hold a freehold estate

C) That the covenant is not personal to the original parties

D) That the burden of the covenant has passed to them

A

A) That the benefit of the covenant has passed to them

(A successor in title can only enforce a covenant if they can show that the benefit of the covenant has passed to them, as it is holding the benefit of the covenant which enables a party to sue)

How well did you know this?
1
Not at all
2
3
4
5
Perfectly
144
Q

What is the overall approach to be taken when interpreting covenants as positive, negative or mixed?

A) Look carefully at the words used. If negative words are used then the covenant is restrictive

B) Look at whether the covenant is expressed to be personal between the parties

C) Look at underlying substance of the covenant rather than the precise words used

A

C) Look at underlying substance of the covenant rather than the precise words used

(The correct approach is to look beyond the words used to see the effect of the covenant in practice. It is the substance of the covenant, rather that the words used to express it, which is important)

How well did you know this?
1
Not at all
2
3
4
5
Perfectly
145
Q

Which one of the following is a restrictive covenant?

A) A covenant to maintain a boundary wall

B) A covenant to use the land for residential purposes only

C) A covenant to build four private houses on the land

D) A covenant to repair and maintain a fence

A

B) A covenant to use the land for residential purposes only

(This is a restrictive covenant as it restricts the use which can be made of the land)

How well did you know this?
1
Not at all
2
3
4
5
Perfectly
146
Q

Which approach is correct in interpreting a covenant not to build any extension to a house without the consent of the dominant owner?

A) This is a mixed covenant which is predominantly negative with a positive condition attached

B) This is a restrictive covenant as the obligation not to build is wholly negative

C) This is a positive covenant as the obligation to obtain consent is wholly positive

D) This is a mixed covenant which can be split into separate positive and negative obligations

A

A) This is a mixed covenant which is predominantly negative with a positive condition attached

(This is a predominantly negative covenant as the primary obligation is not to build on the land. The positive condition cannot stand alone but is only relevant when read alongside the main obligation)

How well did you know this?
1
Not at all
2
3
4
5
Perfectly
147
Q

Which case sets out the test for the ‘hand in pocket’ test?

A) Powell v Hemsley

B) Shepherd Homes v Sandham

C) Tulk v Moxhay

D) Haywood v Brunswick

A

D) Haywood v Brunswick

(This case sets out the ‘hand in pocket’ test, which states that if performance of a covenant requires expenditure of money, it is a positive covenant)

How well did you know this?
1
Not at all
2
3
4
5
Perfectly
148
Q

Which one of the following is a positive covenant?

A) A covenant not to allow a fence to fall into disrepair

B) A covenant to maintain land in an undeveloped state

C) A covenant not to sell alcohol on the land

D) A covenant not to use the land for industrial purposes

A

A) A covenant not to allow a fence to fall into disrepair

(This is a positive covenant as the substance of the covenant is to maintain the fence in a good state of repair)

How well did you know this?
1
Not at all
2
3
4
5
Perfectly
149
Q

What is the general rule on the burden of covenants at common law?

A) The burden of all covenants except personal covenants passes at common law

B) The burden of a covenant does not pass at common law

C) The burden of personal covenants only passes at common law

D) The burden of all covenants of whatever type passes at common law

A

B) The burden of a covenant does not pass at common law

(A covenant is essentially enforceable on common law contractual principles. The general rule is therefore that the covenant is personal between the parties and the burden of the covenant cannot therefore pass to a new servient owner)

150
Q

The second element of the rule in Tulk v Moxhay is that the covenant must ‘accommodate the dominant tenement’. This has three elements. Which of the following is not an element of this test?

A) The dominant owner must have retained an interest in the dominant land at the time of creation of the covenant

B) The dominant and servient land must be proximate

C) The covenant must touch and concern the dominant land

D) There must be an intention for the benefit to pass

A

D) There must be an intention for the benefit to pass

(The intention for the benefit to pass relates to the passing of the benefit and has not place in the test for passing the burden)

151
Q

In the absence of express intention for the burden of a covenant to pass to the successor covenantor, which statutory provision implies such intention?

A) LRA 2002, s 32

B) LPA 1925, s 78

C) LPA 1925, s 198

D) LPA 1925, s 79

E) LPA 1925, s 52

A

D) LPA 1925, s 79

(This is correct. If there is not express intention that the burden should run (look to the wording of the covenant) then s 79 will imply such intention unless contrary intention can be shown)

152
Q

The successor covenantor must have notice of the covenant. In registered land and unregistered land, what is the effect if the covenant has not been protected in accordance with the relevant rule?

A) The new owner would not be bound by the covenant

B) A donee would not be bound by the covenant, but a purchaser for value would be

C) The new owner would be bound if the land is unregistered, but not if it is registered

D) A purchaser for value will not be bound by the covenant but a donee would be

A

D) A purchaser for value will not be bound by the covenant but a donee would be

(This is correct. Although the rules are different depending upon whether the land is registered or not, the principle is the same. A purchaser of the land would not be bound, but a donee (someone who is gifted or inherits the land) will be)

153
Q

Which one of the following statements is the best definition of annexation?

A) Annexation occurs when the rules in Ellison v Reacher are met

B) This is the most usual method for passing the benefit of a covenant in equity

C) The benefit of the covenant forms part and parcel of the dominant land from the time of creation and automatically passes to a new owner of the dominant land when the land is transferred

D) The benefit of the covenant is passed from one owner of the benefitted land to a new owner every time the land is transferred

E) The benefit of the covenant is attached permanently to the land as soon as the covenant is made

A

C) The benefit of the covenant forms part and parcel of the dominant land from the time of creation and automatically passes to a new owner of the dominant land when the land is transferred

(This is the most comprehensive definition of annexation, which operates to make the benefit an integral part of the dominant land itself)

154
Q

Which statutory provision automatically annexes the benefit of a covenant to the dominant land?

A) LPA 1925, s 79

B) LCA 1971, s 79

C) LPA 1925, s 78

D) LRA 2002, s 78

A

C) LPA 1925, s 78

(S 78 operates to annex the benefit of a covenant to the dominant land as soon as the covenant is made, unless this is expressly excluded)

155
Q

Where the benefit of a covenant is expressly assigned, which formalities must be complied with?

A) The assignment must be in writing and signed by the person disposing of the benefit, LPA 1925, s53(1)(c)

B) The assignment must be created in writing and signed by the person creating it, LPA 1925, s53(1)(a)

C) The assignment must be in writing, contain all of the terms and be signed by the seller and the buyer, LP(MP)A 1989, s 2

D) The assignment must be in the form of a deed complying with LP(MP)A 1989, s 1

A

A) The assignment must be in writing and signed by the person disposing of the benefit, LPA 1925, s53(1)(c)

(This is correct. The benefit is treated as the disposition of an equitable interest in land and the formalities in LPA 1925, s 53(1)(c) must be complied with)

156
Q

Which statement best describes the test for the passing of the benefit of a covenant under a building scheme?

A) The benefit of a restrictive covenant will pass if the elements set out in Ellison v Reacher are strictly complied with

B) The benefit of any covenant will pass if the parties intended to create a scheme of mutually enforceable obligations

C) The benefit of any covenant will pass if the elements set out in Ellison v Reacher are strictly complied with

D) The benefit of a restrictive covenant will pass if the parties intended to create a scheme of mutually enforceable obligations

A

D) The benefit of a restrictive covenant will pass if the parties intended to create a scheme of mutually enforceable obligations

(The test concerns the intention of the parties, relaxing the strict requirements set out in Ellison v Reacher)

157
Q

Which is the correct authority for the proposition that at common law, the burden of a freehold covenant cannot pass to a new owner?

A) Rhone v Stephens

B) Austerberry v Oldham Corporation

C) Halsall v Brizell

D) Tophams v Earl of Sefton

E) Tulk v Moxhay

A

B) Austerberry v Oldham Corporation

(Austerberry v Oldham Corporation states that at common law, the burden of an obligation cannot pass to the successor in title of the original covenantor)

158
Q

What remedy can be sought against the original covenantor?

A) Prohibitory injunction

B) Specific performance

C) Mandatory injunction

D) Damages only

E) Injunction and damages

A

D) Damages only

(As the original covenantor is no longer in possession of the land, they cannot carry out the terms of an injunction or order for specific performance. The only remedy which can be awarded is damages)

159
Q

Which statement most accurately describes the effect of an indemnity covenant?

A) It enables the original covenantor to recover any damages paid out in respect of a successor’s breach. The damages can only be recovered from the original covenantor’s immediate successor in title

B) It enables the original covenantor to recover any damages paid out in respect of a successor’s breach. The damages can be recovered direct from the person in breach

C) It enables the burden of a positive covenant to pass to a successor in title to the burdened land

A

A) It enables the original covenantor to recover any damages paid out in respect of a successor’s breach. The damages can only be recovered from the original covenantor’s immediate successor in title

(An indemnity covenant is made between the original covenantor and his successor in title and is personal to them. If the original covenantor pays out damages for a successor’s breach, the indemnity covenant enables the original covenantor to recover the outlay from the immediate successor in title, who may not be the person in breach)

160
Q

To what type of covenants does the general common law rule on passing the burden apply?

A) Positive covenants only

B) Restrictive covenants only

C) All covenants of whatever type

D) Personal covenants only

A

C) All covenants of whatever type

(The general common law rule applies to all covenants of whatever nature, although they are typically used for positive covenants for which the equitable rules do not apply. Generally, the burden of an obligation will not pass to a successor in title at common law)

161
Q

Which is the most accurate statements on the effect of LPA 1925, s79?

A) The section implies wording into freehold covenants to pass the burden of a freehold covenant to a successor in title

B) The section implies wording into freehold covenants to make the original covenantor liable for all breaches of covenant by the immediate successor in title

C) The section implies wording into freehold covenants to make the original covenantor liable for all breaches of covenant by any successor in title

D) The section implies wording into freehold covenants to pass the benefit of a freehold covenant to a successor in title

A

C) The section implies wording into freehold covenants to make the original covenantor liable for all breaches of covenant by any successor in title

(The section makes the original covenantor liable for all breaches of covenant by any person who holds the land after the original covenantor has parted with it. It does not pass the burden of the covenant)

162
Q

What is the authority for the exception to the general rule that the burden of a freehold covenant does not pass at common law?

A) Halsall v Brizell

B) Rhone v Stephens

C) Thamesmead v Allotey

D) Austerberry v Oldham Corporation

E) Davies v Jones

A

A) Halsall v Brizell

(Halsall v Brizell gives the mutual benefit and burden exception to the general rule. Its scope was refined in subsequent cases)

163
Q

To which of these covenants would the doctrine of mutual benefit and burden apply so that the burden of the covenant could pass at common law?

A) A covenant to contribute towards the costs of the maintenance of a tennis courts, which the the servient land owner has a right to use and chooses to use

B) A covenant to keep the servient land well maintained and tidy

C) A covenant to contribute towards the repair and maintenance of an outdoor swimming pool, which the servient land owner has a right to use. The servient land owner is, however, choosing not to use the swimming pool and it does not like swimming.

D) A covenant to erect and maintain a boundary fence between the dominant and servient land

E) A covenant to contribute towards the costs of the maintenance of a shared driveway, which the servient land owner has no choice but to use to access the servient land

A

A) A covenant to contribute towards the costs of the maintenance of a tennis courts, which the the servient land owner has a right to use and chooses to use

(This is correct. The is a mutual benefit - the right to use the tennis court, which the servient land owner is choosing to take. Therefore, the burden will pass using the rule in Halsall v Brizell)

164
Q

Which of these statements is the most accurate in describing how Rhone v Stephens refined the rule in Halsall v Brizell?

A) A person who takes the benefit in the nature of an easement conferred in a deed must submit to any burden imposed in that deed

B) The benefit in the nature of an easement and the burden of the covenant must be conferred in the same transaction

C) There must be a close link between the benefit in the nature of an easement and the burden of the covenant

D) The benefit in the nature of an easement and the burden of the covenant need not be explicitly inter-related as long as there is some link between the two

A

C) There must be a close link between the benefit in the nature of an easement and the burden of the covenant

(Rhone v Stephens restricted the scope of Halsall v Brizell as it had been thought that the benefit and the burden simply had to be conferred in the same transaction. It made it clear that the benefit and burden had to be directly linked)

165
Q

Which are the correct formalities for expressly assigning the benefit of a covenant at common law?

A) In writing, with notice given to the covenantor: LPA 1925, s 136

B) In writing containing all the terms and signed by both parties: LP(MP)A 1989 s 2

C) In a document headed a deed, signed by the grantor, witnessed and dated: LP(MP)A 1989, s 1

D) In writing and signed by the assignor: LPA 1925, s 53(1)(c)

A

A) In writing, with notice given to the covenantor: LPA 1925, s 136

(The assignment can be made simply in writing, but notice must be given to the covenantor. This is to inform the covenantor that a new person is entitled to enforce the covenant)

166
Q

Which authority sets out the requirements for implied assignment of the benefit of a covenant at common law?

A) P&A Swift Investments v Combined English Stores plc

B) Tulk v Moxhay

C) Smith and Snipes Hall Farm Ltd v River Douglas Catchment Board

D) LPA 1925, s 78

A

A) P&A Swift Investments v Combined English Stores plc

(Correct. This case sets out the 4 requirements for implied assignment of the benefit of a covenant at common law)

167
Q

Which of the following covenants demonstrates express intention that the benefit should pass?

A) The buyer covenants with the intention of burdening land know of 31 Yew Tree Close to only use the land for residential purposes

B) The buyer covenants to only use the land for residential purposes

C) The buyer covenants with the intention of benefiting land know of 31 Yew Tree Close to only use the land for residential purposes

D) The buyer covenants on behalf of itself and its successors in title to only use the land for residential purposes

A

C) The buyer covenants with the intention of benefiting land know of 31 Yew Tree Close to only use the land for residential purposes

(Intention of the parties is expressed here with the reference to the covenant ‘benefiting land know as…’
If there is no express intention, LPA 1925, s 78 implies an intention for the benefit to pass unless it is expressly excluded)

168
Q

A sells part of its land to B. B enters into the following covenant with A in the transfer deed:

B covenants with A to wash A’s car every week

Which of the following reasons best explains why the covenant will not pass to a successor in title to A?

A) There is no express assignment of the covenant

B) There is no express intention that the covenant should run

C) The covenantee does not hold a legal estate in the land at the time the covenant is granted

D) The covenant does not touch and concern the dominant land

A

D) The covenant does not touch and concern the dominant land

(A covenant to wash the covenantee’s car every week will not touch and concern the land as it has no impact on the quality, enjoyment or utility of the land. It is simply a personal privilege)

169
Q

Which judicial body determines applications to discharge or modify covenants?

A) The Upper Tribunal (Lands Chamber)

B) The County Court

C) The Upper Chamber (Lands Tribunal)

D) The High Court

A

A) The Upper Tribunal (Lands Chamber)

(The Upper Tribunal (Lands Chamber) has jurisdiction to hear applications of this nature)

170
Q

Which statutory provision gives the tribunal the jurisdiction to discharge or modify covenants?

A) LP(MP)A 1989 s 84(1)

B) LRA 1925, s 84(1)

C) LPA 1925, s 84(1)

D) LRA 2002 s 84(1)

A

C) LPA 1925, s 84(1)

(s84(1) gives the Upper Tribunal (Lands Chamber) the power to discharge the whole or part of a covenant)

171
Q

Which these is not a statutory ground for discharging or modifying a covenant?

A) The original dominant owner has sold the dominant land and cannot be located in order to execute a formal discharge

B) The dominant owner will not suffer loss

C) The covenant has become obsolete due to changes in the property or the neighbourhood.

D) The dominant owner has expressly or impliedly agreed to discharge

A

A) The original dominant owner has sold the dominant land and cannot be located in order to execute a formal discharge

(There is no provision in s84(1) LPA 1925 for an application to be made simply because the dominant owner has sold the land. In fact, the appropriate person to execute a discharge is the current dominant owner, but even so, there is no provision in the statute for this type of application)

172
Q

What type of covenants can be discharged or modified by the tribunal under LPA 1925, s84?

A) Positive covenants only

B) Restrictive covenants only

C) Personal covenants only

D) Any type of covenant

A

B) Restrictive covenants only

(s84(1) specifically states that the tribunal can only discharge or modify restrictive covenants)

173
Q

To which of the follow scenarios would the basic rule of priority apply?

A) An interest has not been properly protected in accordance with the rule under the LRA 2002

B) A man gift their land to a woman

C) A man sells their land to a woman

A

B) A man gift their land to a woman

(This is correct. The basic rule of priority applies whenever there is not ‘a disposition of a registered estate for valuable consideration’ which this is not. The woman is being gifted the land, she is a ‘donee’ and not a purchaser for valuable consideration. The basic rule of priority in s 28 of the LRA 2002 will therefore apply and the woman will take the land subject to all pre-existing interests)

174
Q

A woman is the owner of a registered freehold estate (the ‘Land’). The woman grants to a man an interest in the Land. The man does not protect the interest in accordance with the relevant rule under the LRA 2002.

Two years ago, the woman gifted the Land to their friend.

A year later, the friend sold the Land to a buyer.

Last month, the buyer died. Their daughter inherited the Land.

Assuming the man never protects his interest. Who on the facts will be bound by the man’s interest during their period ownership of the Land?

A) The woman, the friend and the buyer’s daughter

B) The woman only

C) The woman and the friend only

D) All parties - the woman, the friend, the buyer and the buyer’s daughter

A

C) The woman and the friend only

(This is correct. The woman granted the interest over the Land and is therefore bound by it (often under the ordinary rules of contract law). The enforcement rules are only relevant to subsequent owners of the Land. The friend is bound under the basic rule of priority as the friend is a donee.
The buyer and the buyer’s daughter will not be bound. The buyer is a purchaser for valuable consideration and therefore s 29 LRA 2002 will apply. Because the man has failed to protect the interest in accordance with the relevant registration rule, the buyer will not be bound by it. Although the buyer’s daughter is a donee, rather than a purchaser for valuable consideration, and the basic rule of priority will therefore apply, the man’s interest has been ‘lost’ when the Land was sold to the buyer. It will not resurrect itself)

175
Q

Which of the following options is a ‘registrable disposition’ to which s 29 will apply?

A) Contract to transfer the freehold estate

B) Grant of a easement or restrictive covenant

C) Transfer /grant of a legal estate / grant of a legal mortgage

D) Sale of a freehold estate only

A

C) Transfer /grant of a legal estate / grant of a legal mortgage

(This is correct. A registrable disposition is a transaction which must be completed by registration I.e., the transactions that must be completed by registration under LRA 2002, s 27(2) but it really is most applicable on transfer of a legal estate or the grant of a legal mortgage or legal lease)

176
Q

Which one of the following is NOT a registrable disposition under the Land Registration Act 2002?

A) Creation of a restrictive covenant

B) Transfer of freehold

C) Grant of a legal easement

D) Grant of a landlord’s right of entry

E) Creation of mortgage or charge

A

A) Creation of a restrictive covenant

(This is correct. A restrictive covenant is not an interest which must be registered as part of the process of creating it)

177
Q

Which statutory provision states that a purchaser for valuable consideration will take land subject to any pre-existing registrable disposition which were properly registered?

A) Land Registration Act 2002, s28

B) Land Registration Act 2002, sch 3

C) Land Registration Act 2002, s29(2)

D) Land Registration Act 2002, s27

A

C) Land Registration Act 2002, s29(2)

178
Q

What is the term given to interests which MUST be registered in order to be legal interests in land?

A) Interests protected by entry of notice

B) Overriding interests

C) Registrable dispositions

A

C) Registrable dispositions

(This is correct. Registrable dispositions are transactions which must be completed by registration. This includes creation of certain interests in land)

179
Q

Which statement is an accurate explanation of LRA 2002, s29(1) as far as it relates to registrable dispositions?

A) A purchaser for valuable consideration will take land free of any interest not properly protected

B) A purchaser for valuable consideration will only be bound by overriding interests

C) A donee will take land free of any prior registrable dispositions which have not been registered

A

A) A purchaser for valuable consideration will take land free of any interest not properly protected

180
Q

Which one of the following equitable interests CANNOT be protected by entering a notice in the charges register?

A) Contract for the purchase of the freehold

B) Equitable easements

C) Interest in a trust of land

D) Estate contracts

E) Restrictive covenants

A

C) Interest in a trust of land

(This is correct. Land Registration Act 2002, s33 specifically states trust interests cannot be protected by notice)

181
Q

Which of the following statements is the best explanation of restrictions?

A) A restriction imposes conditions which must be complied with before a disposition of land can be registered

B) A restriction protects an interest in a trust of land by requiring the purchaser to overreach the interest.

C) A restriction makes the relevant interest binding as a new owner of the land

A

A) A restriction imposes conditions which must be complied with before a disposition of land can be registered

(This is correct. A restriction does not make an interest binding as a new owner, it simply tells that person what to do in order to be able to complete the transaction)

182
Q

If a covenant should be protected by a notice in the charges register of the servient land, but has not been protected, what happens when the servient land is gifted to A?

A) A is not bound by the covenant because Land Registration Act 2002, s29(1) applies

B) A is bound by the covenant because Land Registration Act 2002, s28 applies

C) A is bound by the covenant because Land Registration Act 2002, s29(2) applies

A

B) A is bound by the covenant because Land Registration Act 2002, s28 applies

(This is correct. A is a volunteer, or donee, which means that A has not paid for the land. Volunteers are always bound by pre-existing interests, irrespective of whether they have been protected or not)

183
Q

Which one of the following is an interest capable of being overreached?

A) Easement

B) Beneficial interest under a trust

C) Mortgage

D) Lease

E) Restrictive covenant

A

B) Beneficial interest under a trust

(This is correct. This type of interest is the only type of interest which can be overreached. This is because it is not an interest (unlike easements and covenants) which is intended to affect one piece of land indefinitely)

184
Q

Which of the following statements accurately sets out the process for overreaching?

A) The purchaser must pay the purchase money to at least one trustee

B) The purchaser must pay the purchase money direct to the beneficiaries

C) The trustee must pay the purchase money to the beneficiaries

D) The purchaser must pay the purchase money to at least two trustees

A

D) The purchaser must pay the purchase money to at least two trustees

(This is correct. LPA 1925, s2, s27 state that capital money must be paid to at least two trustees for overreaching to take effect)

185
Q

Which of the following statements correctly explains what happens if a beneficial interest behind a trust is not overreached?

A) The interest disappears and is lost forever

B) The interest will be binding on a new owner of the land

C) The interest remains in the land, but will not necessarily be binding

A

C) The interest remains in the land, but will not necessarily be binding

(This is correct. If an interest is not overreached, it remains in the land. It is not automatically binding on a purchaser though, it will depend if the holder of the interest is in ‘actual occupation’)

186
Q

To whats system of land ownership does the doctrine of overreaching apply?

A) Unregistered land only

B) Both registered and unregistered land

C) Registered land only

A

B) Both registered and unregistered land

(This is correct. You must keep the two systems of land ownership distinct. The rules that relate to each system do not relate to the other, they are completely separate systems. However, overreaching applies to a beneficial interest behind a trust (rather than one particular system.) The doctrine therefore applies to trusts of land over both registered and unregistered land)

187
Q

Which case is a good example of how interests are overreached?

A) William’s + Glyn’s Bank V Boland

B) City of London Building Society v Flegg

C) Link lending v Bustard

D) Chhokar v Chhokar

A

B) City of London Building Society v Flegg

(This is correct. As the mortgage money lent by the building society was paid to the Maxwell – Browns as legal trustees, the Fleggs’ interests were overreached and converted its interests in money)

188
Q

Which of the following statements is the most accurate definition of an overriding interest?

A) An overriding interest is an interest which will bind a purchaser of the burdened land even though it does not appear on the register of title

B) An overriding interest is an interest which must be protected by an entry on the register of title to be binding on a new owner of the burdened land

C) An overriding interest is an interest which can be moved from land to the proceeds of sale or loan money if the money is paid to two trustees/legal owners

A

A) An overriding interest is an interest which will bind a purchaser of the burdened land even though it does not appear on the register of title

(This is correct. An overriding interest does not have to be registered in order to be binding)

189
Q

Which of the following is an overriding interest within the scope of land Registration Act 2002, Schedule 3, para 1?

A) An equitable lease for a term of four years

B) An equitable lease for a term of ten years

C) A legal lease for a term of fifteen years

D) A legal lease granted for a term of eight years

E) A legal lease granted for a term of five years

A

E) A legal lease granted for a term of five years

(Correct. Legal leases for a term of seven years or less do not need to be protected by registration in order to be enforceable)

190
Q

Which one of the following interests will NOT ever give rise to an overriding interest within Land Registration Act 2002, schedule 3, paragraph 2 (interests of people in actual occupation of the land)?

A) An interest in a trust of land

B) An option to purchase

C) A restrictive covenant

D) An equitable lease

A

C) A restrictive covenant

(Correct. To be overriding interest under para 2, the holder must be in actual occupation of the burdened land. This is not the case for the holder of the benefit of a covenant)

191
Q

Which one of the following cases is the best authority for the proposition that temporary absence from a property will not necessarily defeat actual occupation?

A) Link Lending v Bustard

B) Stockholm Finance v Garden Holdings

C) Abbey National v Cann

D) Williams & Glyn’s Bank v Boland

A

A) Link Lending v Bustard

(Correct. Temporary absence did not defeat actual occupation. This case is good illustration of how courts consider each case on its own facts)

192
Q

A woman has the benefit of a right of way which is a legal easement over a man’s land implied under the rule in Wheeldon v Burrows. The woman uses the right of way on a monthly basis. The man has just sold the burdened land to a buyer. Can the woman enforce the easement against the buyer?

A) The woman cannot enforce it against the buyer as the woman is not in actual occupation of the burdened land

B) The woman can enforce it against the buyer as implied easements are automatically binding as overriding interests

C) The woman can enforce it against the buyer as it is an overriding interest

D) The woman cannot enforce it against the buyer as a legal easement is a registrable disposition and it has not been properly registered on the facts

A

C) The woman can enforce it against the buyer as it is an overriding interest

(Correct. Schedule 3, para 3 states thatimplied legal easements can be overriding interests provided that they are obvious from a reasonable inspection of the burdened land (which this probably is) OR that they have been exercised within the 12 months immediately prior to the sale (which this definitely has been).
Express legal easements are registrable disposition under s 27 LRA 2002)

193
Q

On the sale of unregistered land, the seller must deduce title (proof ownership) by sending what to the buyer?

A) The Land Charges Register

B) The official copies

C) Epitome of title

C) The doctrine of notice

D) All the historic title deeds

A

C) Epitome of title

(As the seller may have lots of old title deeds of varying degrees of usefulness relating to the property, s/he must only select those which contain relevant detail about the property and then make a copy of these documents. This bundle of relevant copy documents is called an epitome of title. On a sale, the seller is required to show the epitome of title to the buyer. This is called deducing title)

194
Q

Which of the following category of rights may need protecting by way of a Land Charge?

A) Equitable interests created pre-1926

B) Equitable interests created post-1926

C) Legal interests

A

B) Equitable interests created post-1926

(The majority of equitable interests created after 1926 should be protected by the entry of a Land Charge against the name of the land owner at the time the interest is created. This was intended to be a ‘stop-gap’ system until all land became registered in England and Wales)

195
Q

Which of the following category of rights are only subject to the doctrine of notice?

A) Equitable interests created pre-1926

B) Equitable interests created post-1926

C) Legal interests

A

A) Equitable interests created pre-1926

(Equitable interests created prior to 1926 remain subject to the doctrine of notice, as do equitable interests under a trust that have not been overreached. For this reason, the doctrine of notice has fairly limited application nowadays)

196
Q

Could a person who is gifted the land ever be ‘equity’s darling’?

A) Yes, because they get the land in ‘good faith’ i.e. they are bona fide

B) No, because they are not a purchaser for value

C) No, because they have notice

D) Yes, because they are a bona purchaser of a legal estate without notice

A

B) No, because they are not a purchaser for value

(This is correct. Equity’s darling is a bona fide purchaser for value of a legal estate without notice. Someone who is gifted the land (or inherits it) could never be equity’s darling because they are never a ‘purchaser for value’)

197
Q

Which of the following category of rights will always bind a third party purchaser of a piece of land?

A) Legal interests

B) Equitable interests created post-1926

C) Equitable interests created pre-1926

A

A) Legal interests

(Legal interests will ‘bind the whole world.’ This is the case even if the new owner does not know about the interest prior to the purchase and there is no evidence of it)

198
Q

The entry of a Land Charge is protected against what?

A) The title number of the land

B) The name of the land owner at the time the interest is created

C) The address of the land

D) The name of the right holder at the time the interest is created

A

B) The name of the land owner at the time the interest is created

(The land charge must be entered against the correct name(s) of the estate owner(s) at the time of the creation of the land charge: LCA 1972, s 3(1))

199
Q

Which of the following rights over unregistered land should be protected by a Land Charge?

A) Legal mortgage

B) Equitable easement created in 1922

C) Restrictive covenant created in 1910

D) Puisne mortgage

E) A beneficial interest in a trust a trust

A

D) Puisne mortgage

(A second legal mortgage is the only type of legal interest that can be protected by way of a Land Charge. It is the exception to the rule that Land Charges protect equitable interests only)

200
Q

A buyer is purchasing a piece of unregistered land, which is subject to an equitable easement granted in 1960 by the current owner. The owner is called Peter James Wells, though his name in the title deeds appears as Peter Wells. A search of Land Charges against the name Peter James Wells reveals no entries.

Which of the following options best explains what this means for the buyer?

A) The buyer will not be bound by the equitable easement because it has not been correctly protected

B) The buyer will not be bound by the equitable easement because it is registered against the wrong name

C) The buyer could still be bound by the equitable easement because the search was not against the correct name

D) The buyer could still be bound by the equitable easement if it appears in the Charges Register

E) The buyer will not be bound by the equitable easement because it is a purchaser for value

A

C) The buyer could still be bound by the equitable easement because the search was not against the correct name

(The land charge must be entered against the correct name(s) of the estate owner(s) at the time of the creation of the land charge. This is the version of the name as it appears in the title deeds ie Peter Wells. The buyer has searched against the wrong name on the facts, and therefore could still be bound by the easement if it has been entered as a Land Charge against the name Peter Wells)

201
Q

A landlord and tenant entered into an agreement for lease last year. The parties failed to complete the lease, but the tenant moved in anyway under the terms of the agreement, which it did nothing with. The landlord died a few months ago. The new owner inherited the unregistered freehold estate from the landlord. The new owner knows about the agreement for lease.

Which of the following options best describes if the new owner will be bound by the agreement for lease?

A) The new owner will be bound by the estate contract because the interest holder is in actual occupation of the land

B) The new owner will be bound by the estate contract because it is not a purchaser for value

C) The new owner will not be bound by the estate contract because it will be regarded as Equity’s Darling

D) The new owner will be bound by the estate contract because it has actual notice of the interest

E) The new owner will not be bound by the estate contract because the estate contract has not been protected

A

B) The new owner will be bound by the estate contract because it is not a purchaser for value

(The agreement for lease, as an estate contract, should be protected by a Class C (iv) Land Charge under the LCA 1972. Failure to do this means that it will not be binding on a purchaser for value. However, the new owner is not a purchaser for value, it has inherited the freehold estate. For this reason, the new owner will be bound by the equitable interest. Notice is irrelevant as the estate contract was created after 1926)

202
Q

Which of the following rights over unregistered land would be subject to the doctrine of notice?

A) Equitable easement created in 1928

B) Interest in a trust of land that has not been overreached

C) Puisne mortgage

D) Legal mortgage

E) Restrictive covenant created in 1960

A

B) Interest in a trust of land that has not been overreached

(An equitable interest in trust of land cannot be protected by way of a Land Charge. If such an interest has not been overreached, the interest will be binding unless the purchaser is ‘equity’s darling’)

203
Q

Which of the following options correctly describes Equity’s Darling?

A) A bona fide purchaser for value of a legal estate without notice

B) A bona fide seller of a legal estate without notice

C) A bona fide purchaser of a legal estate with notice

D) A purchaser for value of an estate without notice

E) A purchaser for value of an estate with notice

A

A) A bona fide purchaser for value of a legal estate without notice

(‘Equity’s Darling’ is a ‘bona fide purchaser for value of a legal estate without notice’ of the equitable interest)

204
Q

Who of the following would you describe as Equity’s Darling?

A) A good faith buyer who pays monetary consideration for a terms of years absolute and who instructs solicitors and inspects the land but finds no evidence of the equitable interest

B) A good faith buyer who pays monetary consideration for a fee simple absolute in possession who does not inspect the land but does instruct a solicitor who finds evidence of the equitable interest

C) A good faith new owner who has inherited a fee simple absolute in possession and who instructs solicitors and inspects the land but finds no evidence of the equitable interest

D) A good faith new owner who has been gifted a fee simple absolute in possession and who instructs solicitors and inspects the land but finds no evidence of the equitable interest

A

A) A good faith buyer who pays monetary consideration for a terms of years absolute and who instructs solicitors and inspects the land but finds no evidence of the equitable interest

(This person is Equity’s Darling. They are a good faith purchaser for value of a legal estate without notice. All elements are satisfied)

205
Q

Which of the following options correctly describes constructive notice?

A) A buyer will be bound by those equitable interests it ought reasonably to have known about

B) A buyer will be bound by those equitable interests its solicitor knows about

C) A buyer will be bound by all legal interests as legal rights bind the world

D) A buyer will be bound by all equitable interests its agent knows or ought to know about

E) A buyer will be bound by those equitable interests it knows about

A

A) A buyer will be bound by those equitable interests it ought reasonably to have known about

(LPA 1925, s 199(1)(ii)(a) requires a purchaser to act as a prudent person. A purchaser cannot attempt to avoid being bound by equitable interests by shutting its eyes and ears and thus not having actual notice. A purchaser will be fixed with constructive notice of an interest if it fails to pursue a line of enquiry which ought reasonably to have been made)

206
Q

Once an interest is void against Equity’s Darling, will it be binding on a subsequent owner who has actual notice of the right?

A) Yes, if the land is then registered

B) No, it cannot be revived so as to bind a subsequent buyer

C) Yes, if the subsequent owner is not Equity’s Darling

D) No, it will not be binding because it should be entered as a Land Charge

A

B) No, it cannot be revived so as to bind a subsequent buyer

(An equitable interest is void against such a purchaser or someone claiming through them. It follows that once an equitable interest becomes void for want of notice, it cannot be revived so as to bind a subsequent buyer who does have notice of the equitable right)

207
Q

Which statute governs the operation of trusts in relation to co-owned land?

A) Trustee Act 1925

B) Trustee Act 2000

C) Trusts of Land and Appointment of Trustees Act 1996

D) Law of Property Act 1925

A

C) Trusts of Land and Appointment of Trustees Act 1996

(This is correct. This statute deals with the powers and duties of trustees and beneficiaries and the resolution of disputes relating to trusts of land)

208
Q

Which of the following best describes the position of the legal owners where there is a trust of land?

A) They are the absolute owners of the land

B) They are able to sign transfer deeds and mortgage documents

C) They are trustees acting in an administrative capacity

D) They are entitled to an equal share of the sale proceeds

E) They are the registered owners in the proprietorship register

A

C) They are trustees acting in an administrative capacity

(This is correct. The legal owners are named as the proprietors in the register, but have duties of trusteeship. They have the power to sign transfer deeds, contracts and mortgage documents but they hold the property on trust for the beneficiaries)

209
Q

A couple (A and B) buy a registered property together, each contributing 50% of the purchase price and being registered jointly as the legal owners as the Land Registry.

Is a trust imposed in the circumstances? Pick the best option that explains the position below.

A) Yes, a trust is automatically imposed. A and B hold the property on trust for themselves. They are both the legal and equitable owners

B) No, a trust will not be imposed because A and B are both registered as the legal owners so it is not necessary in the circumstances.

C) No, a trust will not be imposed because the legal and equitable title would be held by the same people so it is not necessary in the circumstances

A

A) Yes, a trust is automatically imposed. A and B hold the property on trust for themselves. They are both the legal and equitable owners

(This is correct. Whenever land is co-owned, as it is here (by A and B) a trust is automatically imposed. Here, the legal and equitable title is very simple and may be exactly the same. It does not matter. A trust still exists)

210
Q

A commercial property is purchased by A and B who each contribute half of the purchase price. The legal title is registered in A’s name only. What is the position regarding A and B?

A) A holds the property on resulting trust for B

B) A holds the property on constructive trust for A and B

C) A holds the property on resulting trust for A and B

D) A holds the property on constructive trust for B

E) There is no trust as A is named as legal owner

A

C) A holds the property on resulting trust for A and B

(This is correct. Where, in a non-domestic context, someone makes a direct contribution to the purchase price, a resulting trust is implied to reflect their contribution)

211
Q

What formalities must be complied with in order to create an express trust of land?

A) No formalities are necessary for the creation of a trust of land

B) The declaration of trust must be in writing signed by the person creating the trust

C) The declaration of trust must be evidenced in writing signed by the person creating the trust

D) The declaration of trust must be created by way of a valid deed

A

C) The declaration of trust must be evidenced in writing signed by the person creating the trust

(This is correct: Law of Property Act 1925, s 53(1)(b))

212
Q

Which one of the following is essential in order for co-ownership to exist?

A) Unity of interest

B) Unity of intention

C) Unity of possession

D) Unity of title

E) Unity of time

A

C) Unity of possession

(This is correct. Without unity of possession, there is no co-ownership)

213
Q

Which of the following is the best explanation of the concept of survivorship?

A) When a joint tenant dies, that person’s interest does not pass to the person named in their will

B) When a joint tenant dies, that person’s interest passes to the person named in their will.

C) When a joint tenant dies, that person’s share immediately accrues to the surviving joint tenants.

D) When a joint tenant dies, that person’s notional interest immediately accrues to the surviving joint tenants

E) When a joint tenant dies, that person’s share passes to the person named in their will.

A

D) When a joint tenant dies, that person’s notional interest immediately accrues to the surviving joint tenants

(This is correct. Joint tenants are viewed as a single entity and therefore do not have a distinct ‘share’ of the property. Instead they have a notional interest which disappears on their death meaning the property accrues to the surviving joint tenants)

214
Q

In which of the following circumstances will an equitable tenancy in common be presumed?

A) Where the land is bought as a home

B) Where there is no express declaration of joint tenancy

C) Where there have been unequal contributions to the purchase price

D) Where there is wording such as ‘in equal shares’

E) Where there have been equal contributions to the purchase price

A

C) Where there have been unequal contributions to the purchase price

(This is correct. This will be presumed in non-domestic cases only. The presumption can be rebutted by an express declaration of an equitable joint tenancy)

215
Q

What is a tenancy in common?

A) An interest held by one of a number of people seen as a collective entity

B) A distinct but undivided share in land

C) A distinct and divided share in land

A

B) A distinct but undivided share in land

(This is correct. A tenancy in common is a distinctly quantified share in the whole property, but the holder cannot say that any particular part is theirs: hence, ‘undivided’ share)

216
Q

Which of these statements is an accurate explanation of unity of time?

A) The co-owners must sign the transfer deed at the same time

B) The co-owners’ interests must all last for the same length of time

C) The co-owners must move into the property at the same time

D) The co-owners’ interests must all take effect at the same time

A

D) The co-owners’ interests must all take effect at the same time

(This is correct. The interests must ‘vest’ at the same time)

217
Q

Which statement best describes the effect of severance of an equitable joint tenancy?

A) It brings co-ownership to an end

B) It changes the basis on which the equitable title will be held in future

C) It enables a co-owner to dispose of his equitable share to whoever he pleases

D) It prevents survivorship from operating

A

B) It changes the basis on which the equitable title will be held in future

(This is correct. Severance does not destroy co-ownership but simply changes the basis on which the parties hold the equitable interest in the future)

218
Q

A, B, C and D bought a house together. A and D contributed 15% of the purchase price each; B contributed 20% and C contributed 50%. They declared themselves to be equitable joint tenants. C severed the joint tenancy. What share will C now have?

A) C has 50% of both the legal and equitable titles

B) C continues as legal joint tenant but has 25% of the equitable title as tenant in common

C) C has 25% of both the legal and equitable titles

D) C continues as legal joint tenant but has 50% of the equitable title as tenant in common

A

B) C continues as legal joint tenant but has 25% of the equitable title as tenant in common

(This is correct. The legal joint tenancy cannot be severed. If the equitable joint tenancy is severed the holder takes an equal share as tenant in common, irrespective of the size of their initial contribution)

219
Q

What effect does making a will have on an equitable joint tenancy?

A) It will have no effect

B) It will sever the legal and equitable joint tenancies

C) It will sever the legal joint tenancy

D) It will sever the equitable joint tenancy

A

A) It will have no effect

(This is correct. A will does not have any effect until after death by which time survivorship has operated)

220
Q

A and B bought a property together and paid 75% and 25% of the purchase price respectively. They declared themselves to be equitable joint tenants. A severed the joint tenancy. What is the position now in equity?

A) A is now a tenant in common as to 75% and B is a joint tenant as to 25%

B) A and B are now tenants in common as to 50% each

C) A and B are now tenants in common as to 75% for A and 25% for B

D) A is now a tenant in common as to 50% and B is a joint tenant as to 50%

A

B) A and B are now tenants in common as to 50% each

(This is correct. When the joint tenancy was severed, A received an equal share irrespective of the size of A’s initial contribution: Goodman v Gallant)

221
Q

A, B, C and D bought a house together. A and D contributed 15% of the purchase price each; B contributed 20% and C contributed 50%. They declared themselves to be equitable joint tenants. C severed the joint tenancy. What share will C now have?

A) C has 50% of both the legal and equitable titles

B) C continues as legal joint tenant but has 50% of the equitable title as tenant in common

C) C has 25% of both the legal and equitable titles

D) C continues as legal joint tenant but has 25% of the equitable title as tenant in common

A

D) C continues as legal joint tenant but has 25% of the equitable title as tenant in common

(This is correct. The legal joint tenancy cannot be severed. If the equitable joint tenancy is severed the holder takes an equal share as tenant in common, irrespective of the size of their initial contribution)

222
Q

The main purpose of the writing may not be severance, but the intention to sever can be implied from what is written.

Which of the following pieces of writing (if either of them) is severance likely to be inferred from in the circumstances?

A) An application to court for an order that the matrimonial home be sold immediately and the proceeds of sale split equally

B) A request for such order ‘as may be just’ relating to the matrimonial home

C) Severance cannot be inferred from either of the other options in the circumstances

A

A) An application to court for an order that the matrimonial home be sold immediately and the proceeds of sale split equally

(Yes this is correct and is akin to the facts of Re Draper’s Conveyance [1969] where a divorce petition that had this wording was held to be enough to sever the joint tenancy)

223
Q

What statutory formalities apply to a notice in writing?

A) The notice must be contained in a valid deed

B) The notice need not comply with any formalities

C) The notice must be made in signed writing

D) The notice in must be in writing, incorporate all the relevant terms and must be signed

A

B) The notice need not comply with any formalities

(This is correct. No statutory formalities are prescribed)

224
Q

On whom must notice in writing be served to effect severance?

A) On all legal and equitable joint tenants

B) On all equitable joint tenants

C) On all legal and equitable owners.

D) On all equitable owners

A

B) On all equitable joint tenants

(This is correct. The equitable joint tenants are the only people who will be affected by the severance and LPA 1925, s 36(2) specifically states that they must all be served)

225
Q

Which one of the following is a unilateral act of partial alienation which could sever an equitable joint tenancy?

A) A sale of an equitable interest

B) A gift of an equitable interest

C) A mortgage of an equitable interest

D) A properly served notice in writing

E) A declaration of bankruptcy

A

C) A mortgage of an equitable interest

(This is correct. A mortgage is an act of partial alienation because when the loan is repaid the owner will retain the equitable interest unencumbered)

226
Q

Why did the judge in Davis v Smith conclude that the joint tenancy had been severed by mutual conduct?

A) The parties had negotiated on the basis that their assets including the house would be realised and divided

B) The parties had engaged in prolonged negotiations

C) The parties had agreed to put the house on the market

D) The parties had already split the proceeds of surrender of a life policy

A

A) The parties had negotiated on the basis that their assets including the house would be realised and divided

(This is correct. The parties had always envisaged a split of the sale proceeds of the house and other assets and had conducted negotiations on that basis)

227
Q

Why was there no severance by mutual conduct in Gore and Snell v Carpenter?

A) Because the ongoing negotiations had not produced a final agreement

B) Because it was not possible to infer from ongoing negotiations any mutual working towards a final end in view

C) Because the ongoing negotiations had not continued for a long enough period

A

B) Because it was not possible to infer from ongoing negotiations any mutual working towards a final end in view

(This is correct. The negotiations did not have any particular aim (unlike in Davis v Smith) and there was no evidence that there was no ‘mutual’ acceptance that Mrs Carpenter would hold tenancies in common. In that situation, the parties were not ‘mutually’ conducting themselves towards the same end)

228
Q

In Burgess v Rawnsley what did the court say must be present in order for the joint tenancy to be severed by mutual agreement?

A) An express or inferred agreement to change the basis on which co-ownership will continue

B) An inferred agreement to end the co-ownership

C) A specifically enforceable contract for one party to ‘buy the other out’

D) An express agreement to sell the property

A

A) An express or inferred agreement to change the basis on which co-ownership will continue

(This is correct. Co-ownership does not end on severance of the joint tenancy: it continues on a different basis. An agreement to do this can be expressly stated, or (as in this case) inferred from what the parties said and did)

229
Q

Who is entitled to make a court application in relation to co-owned land under Trusts of Land and Appointment of Trustees Act 1996, s 14?

A) The legal owners only

B) The beneficial owners only

C) The legal owners and any legal mortgagees only

D) Anyone who has an interest in the land

E) Anyone who has or expects to have an interest in the land

A

D) Anyone who has an interest in the land

(This is correct. This means that legal trustees, beneficiaries, and secured creditors can all make applications)

230
Q

Which of the factors in Trusts of Land and Appointment of Trustees Act 1996, s 15 is given the greatest weight in making a decision, according to the provisions set out in the statute?

A) The statute states that the interests if secured creditors carry greater weight

B) None – the statute gives no indication of any factor having greater weight than the others

C) The statute states that the purpose for which the property was purchased carries greater weight

D) The statute states that the circumstances and wishes of the beneficiaries carry greater weight

E) The statute states that the interests of minors living in the property carry greater weight

A

B) None – the statute gives no indication of any factor having greater weight than the others

(This is correct. There is nothing in TLATA 1996, s 15 which states that any factor is to be given greater weight than others)

231
Q

In which case did the judge state that the law had changed with Trusts of Land and Appointment of Trustees Act 1996 and that the interests of creditors are one of four factors to be considered, not the decisive factor?

A) Mortgage Corporation v Shaire

B) Fred Perry v Genis

C) Bank of Ireland Home Mortgages v Bell

D) Putnam v Taylor

A

A) Mortgage Corporation v Shaire

(This is correct. The judge said that the law had changed with TLATA 1996 and all factors had to be weighed evenly)

232
Q

Which of the following confers a proprietary right in the land?

A) Both a lease and a licence

B) Lease

C) Licence

D) Neither a lease or a licence

A

B) Lease

(A lease is an estate in the land, a recognised proprietary right to possess the land. One of the key features of a proprietary right is that it is capable of being enforced against third parties)

233
Q

A lease granted for 1000 years can never be a lease and fails for lack of a certain term. Is this statement true or false?

A) True

B) False

A

B) False

(A certain term means you know the duration of the lease and when it will end. This is often evidenced by a fixed term. A fixed term can be as little as a day or as 1000 years, it is still certain and we know when it will end)

234
Q

Two parties enter in a document, which they label as a ‘Licence to Occupy’. The term of the licence for ‘as long as the occupier is a student at university’. The right is granted by deed. Which of the following statements correctly explains whether a lease or licence has been granted on the facts?

A) The document is a licence because the correct formalities have not been complied with

B) The document is a licence because it has been labelled as such

C) The document is a lease because it has been granted by deed

D) The document is a lease because the document confers a right to possess the land

E) The document is a licence because there is no certain term

A

E) The document is a licence because there is no certain term

(The document is a licence because the term is uncertain, and we do not know when it will end. There is no certain term. The fact that the document is labelled as a licence is inconclusive and will not be a determining feature. The courts would look at the substance of the document, rather than the label it is given)

235
Q

Last year two parties entered into a lease for five years of an office space. The lease provides that the rent is ‘a peppercorn’ (ie nothing). The lease confers exclusive possession onto the occupier and complies with the correct formalities. Last month the landlord sold the freehold reversion to a third party. Yesterday, the new owner changed the locks and refused to let the occupier in. Which of the following statements correctly explains whether the occupier will be able to recover possession of the office space?

A) The occupier will not be able to recover possession of the office space because there is no certain term, so the document is a licence

B) The occupier will not be able to recover possession of the office space because it only has a personal right in the land

C) The occupier will not be able to recover possession of the office space because it is not paying rent, so the document is a licence

D) The occupier will be able to recover possession of the office space because the document is labelled as a lease and will therefore bind third parties

E) The occupier will be able to recover possession of the office space because it has a proprietary right in the land under the lease

A

E) The occupier will be able to recover possession of the office space because it has a proprietary right in the land under the lease

(The occupier is a tenant. There is a certain term, the occupier has exclusive possession and the correct formalities have been complied with. It does not matter that the rent is ‘a peppercorn’ as a rent does not need to be payable for there to be a lease. As the occupier has a lease, it has a proprietary right in the land, the defining feature of which is that it is capable of being enforced against third parties i.e. the new freehold owner. For this reason, the tenant will be able to recover possession of the office space, it will not have to settle for damages)

236
Q

Occupy for 2 years of an office space. The licence provides that the rent is £10,000 per annum and that the licensor can share occupation with the licensee, which it does. The licence complies with the correct formalities. Yesterday, the new owner changed the locks and refused to let the occupier in. Which of the following statements correctly explains whether the occupier has a lease or a licence?

A) The occupier has a licence because the document is labelled as a licence

B) The occupier has a lease because the correct formalities have been met

C) The occupier has a licence because there is no exclusive possession

D) The occupier has a lease because it is paying rent

E) The occupier has a licence because there is no certain term

A

C) The occupier has a licence because there is no exclusive possession

(The occupier is a licensee only. There is a certain term, but the occupier does not have exclusive possession as it shares possession with the licensor – it cannot exclude the licensor from the premises. It makes no difference that the correct formalities have been complied with, the requirements of a lease have not been met)

237
Q

In order for there to be lease a rent must be payable. Is this true or false?

A) False

B) True

A

A) False

(This is correct. Although Lord Templeman indicated that payment of rent is essential in Street v Mountford [1985] AC 809, LPA 1925, s 205(1)(xxvii) makes it clear that rent is not essential. This was confirmed in the case of Ashburn Anstalt v Arnold [1989] Ch 1)

238
Q

Two parties enter into a document which is described as a lease of a flat with a rent payable of £12,000 per annum payable by equal monthly payments. What kind of lease does the tenant have?

A) A fixed term lease of one year

B) An implied periodic tenancy of one year

C) An express periodic tenancy of one month

D) An express periodic tenancy of one year

E) An implied periodic tenancy of one month

A

D) An express periodic tenancy of one year

(The lease does not have a fixed term, it has a periodic term, which is determined by reference to have rent is calculated, which is ‘per annum’ on the facts i.e. yearly. It does not matter that it is payable monthly. The lease is written down, it is express on our facts)

239
Q

Two parties enter into document labelled as a lease and granted ‘for as long as the tenant is trading’. The occupier moves in and begins paying the annual rent under the lease, which is expressed as being payable quarterly in advance. What kind of lease does the tenant have (if any)?

A) A fixed term lease of one year

B) A fixed term lease for as long as the tenant trades

C) An express periodic tenancy of one year

D) An implied periodic tenancy of one year

E) There is no certain term so there is no lease on the facts

A

D) An implied periodic tenancy of one year

(The lease does not have a certain term. A term ‘for as long as the tenant is trading’ will fail for lack of certainty, we do not know how long this will be. However, because the tenant has moved in and is paying an annual rent, a periodic tenancy of one year will be implied)

240
Q

Two parties enter into a document which grants a right to possess for 5000 years with a rent of £100 per annum payable each year. Which of the following statements best explains the nature of the document?

A) The document is not a lease but there is an implied periodic tenancy on the facts

B) The document is a lease because there is an express periodic term

C) The document is a lease because there is a fixed term

D) The document is a licence because the term is uncertain

A

C) The document is a lease because there is a fixed term

(Although 5000 years is a very long time, the term is certain. We know when the lease will end. This is a fixed term)

241
Q

If an agreement contains a right to introduce others, this will always defeat exclusive possession. Is this statement true or false?

A) False, the courts will always looks at the substance of a sharing clause

B) True, a sharing clause will defeat exclusive possession in every situation

C) True, if it is the landlord sharing occupation

D) False, the courts have held there is no magic in a sharing clause

A

A) False, the courts will always looks at the substance of a sharing clause

(The courts will look at the substance of a sharing clause. Although such a clause does at first sight indicate there is no exclusive possession, because the occupier cannot exclude whoever the landlord can introduce, the courts will look at the substance of the arrangement and the reality of it, whether the clause is in fact a sham. If it is a sham, such a clause will not defeat exclusive possession)

242
Q

A party occupies a flat under a document which is described as a lease. Which of the following statements correctly explains the circumstances that must exist for the occupier to have exclusive possession?

A) Exclusive possession will be defeated if the landlord retains a key to the flat

B) Exclusive possession means the landlord owner would not be able to enter the flat under any circumstances

C) Exclusive possession means the occupier is able to exclude everyone from the flat including the landlord

D) Exclusive possession means the courts will look at the substance of the document, not the label the parties give it

A

C) Exclusive possession means the occupier is able to exclude everyone from the flat including the landlord

(For there to be a lease the occupier must have exclusive possession, which is correctly defined by this statement)

243
Q

An individual occupies a flat under a document labelled as a ‘Licence’. The Licence provides that the Licensor will enter the flat once a week to change the towels and clean. Which of the following statements correctly describes whether the individual has exclusive possession?

A) There is no exclusive possession as the document is labelled as a licence

B) There will be no exclusive possession if the licensor retains a key

C) There could be exclusive possession as the access is restricted

D) There is no exclusive possession as the licensor can share occupation

E) There could be exclusive possession if the clause has never been exercised

The label of the document is inconclusive, the court will look at the substance of the agreement rather than the label. This means the court will also look at the reality of whether the clause has been exercised, or whether it is a sham clause. If the clause has never been exercised then this would indicate it is a sham clause and so there could still be exclusive possession

A

E) There could be exclusive possession if the clause has never been exercised

(The label of the document is inconclusive, the court will look at the substance of the agreement rather than the label. This means the court will also look at the reality of whether the clause has been exercised, or whether it is a sham clause. If the clause has never been exercised then this would indicate it is a sham clause and so there could still be exclusive possession)

244
Q

In considering whether a clause in a lease is a sham clause, not reflecting reality and therefore not defeating a tenant from having exclusive possession, which of the following circumstances would likely indicate such a clause is a sham?

A) A landlord retaining a key to access the premises in case of an emergency

B) A two-bedroomed flat where one room is vacant and the landlord has a right to introduce one other into the premises

C) A widely-drafted sharing clause that has been exercised by the landlord on several occasions

D) A couple sharing a one-bedroomed flat and the landlord has a right to introduce others to share occupation

A

D) A couple sharing a one-bedroomed flat and the landlord has a right to introduce others to share occupation

(The courts will look at the reality of a situation, looking beyond the label. In doing this, the court would consider a number of factors, as laid down in A G Securities v Vaughan and Antoniades v Villiers both reported at [1990] 1 AC 417t. This includes the relationship between the occupiers and whether it would be appropriate to introduce another to share given the relationship. It would be an unlikely situation to introduce someone into a one bedroomed flat which is being occupied by a couple on the facts and this would indicate the clause could be a sham)

245
Q

Which of the following situations would indicate a business occupier does not have exclusive possession of premises it is occupying?

A) The landlord retains a key for the premises

B) The occupier is not permitted to alter the premises

C) The landlord grants security of tenure to the tenant

D) The landlord can relocate the occupier upon given notice

E) The landlord retains a right of access in the event of an emergency

A

D) The landlord can relocate the occupier upon given notice

(In Dresden Estates v Collinson [1988] 55 P & CR 47 a clause entitled the licensor to relocate the occupier was held to defeat exclusive possession)

246
Q

An individual occupies a flat under a Licence agreement. The agreement reserves a right of entry to the licensor in the event of an emergency. The licensor retains a key for this purpose. The clause has never been exercised. Which of the following statements correctly describes whether the individual has exclusive possession?

A) There is no exclusive possession as the access is unrestricted

B) There is no exclusive possession because the individual cannot exclude the landlord from the flat in the event of an emergency

C) There could be exclusive possession if the clause is a sham

D) There could be exclusive possession as access is restricted

E) There is no exclusive possession as the licensor is providing services

A

D) There could be exclusive possession as access is restricted

(Access by the landlord is restricted to only if there is an emergency. This is more of an acknowledgement by the licensor that there is exclusive possession (as it has to reserve this right) rather than a clause that will defeat it. The fact that the licensor retains a key is inconclusive, it is whether access is restricted or unrestricted which will be the determining feature)

247
Q

Why might the courts be more willing to accept the label given to an agreement by the parties where the parties are in a commercial arrangement?

A) The courts will always have regard to the label given to an arrangement by the parties

B) This is the principle laid down in the case of Street v Mountford [1985] AC 809

C) The bargaining power between commercial entities tends to more equal

D) Commercial entitles will always be represented by solicitors so will have had legal advice

A

C) The bargaining power between commercial entities tends to more equal

(The courts are more prepared to accept the reality of the label ‘licence’ than they are in the residential context as there tends to be more equality in bargaining power, with commercial leases often negotiated and parties legally represented)

248
Q

Which of the following statements is true about the application of Street v Mountford to business tenancies?

A) The case only applies to certain business arrangements

B) The case has no application to business arrangements

C) The case applies equally to business arrangements as to residential ones

D) Business occupiers would only need to establish they have a certain term

E) An arrangement relating to business premises will always be a lease

A

C) The case applies equally to business arrangements as to residential ones

(A business tenant must also show they have a certain term and exclusive possession in order to show they have a lease, rather than a licence)

249
Q

An individual occupies a flat under a document labelled as a ‘Licence’. The Licence provides that the Licensor will enter the flat once a week to change the towels and clean. Which of the following statements correctly describes whether the individual has exclusive possession?

A) There will be no exclusive possession if the licensor retains a key

B) There is no exclusive possession as the document is labelled as a licence

C) There could be exclusive possession as the access is restricted

D) There could be exclusive possession if the clause has never been exercised

E) There is no exclusive possession as the licensor can share occupation

A

D) There could be exclusive possession if the clause has never been exercised

(The label of the document is inconclusive, the court will look at the substance of the agreement rather than the label. This means the court will also look at the reality of whether the clause has been exercised, or whether it is a sham clause. If the clause has never been exercised then this would indicate it is a sham clause and so there could still be exclusive possession)

250
Q

Which of the four unities would be defeated if the occupiers each have their own locked bedroom which the other occupiers cannot access?

A) Unity of title

B) Unity of time

C) Unity of possession

D) Unity of interest

A

C) Unity of possession

(All occupiers must be entitled to occupy the whole of the premises. No-one has exclusive use of any part. The occupiers could only ever be individual licensees sharing with other in this instance)

251
Q

If the occupiers of a flat do not have the four unities what does this mean?

A) They will not have a joint tenancy and can only ever be individual licensees of the premises

B) They could have a joint tenancy if they can individually show they have exclusive possession of their own part of the flat

C) They will not have a joint tenancy and cannot hold a lease together. However, if they can individually show exclusive possession of part of the property, they may have individual leases of their own part

D) They will not have a joint tenancy but may have a lease if the correct formalities are followed

A

C) They will not have a joint tenancy and cannot hold a lease together. However, if they can individually show exclusive possession of part of the property, they may have individual leases of their own part

(If it is found that the occupants do not have the four unities, they cannot have a joint tenancy. But if they can show that they each have exclusive possession of a part of the property then it is possible for them to have individual leases of their own part. If neither a joint tenancy nor an individual tenancy exists then the occupants can only be individual licensees sharing with each other)

252
Q

Which of the following correctly describes unity of title?

A) All must be entitled to occupy the whole of the premises

B) All must have a leasehold interest for the same term under the same conditions and must be jointly liable for the rent

C) All of the interests must derive from the same document or from separate but identical documents which are interdependent

D) All of the interests must derive from the same document

E) All of the interests must start at the same time

A

C) All of the interests must derive from the same document or from separate but identical documents which are interdependent

(As it was held in Antoniades v Villiers [1990] 1 AC 417, the occupiers need not sign the same document for there to be unity of title providing they sign identical documents that are interdependent)

253
Q

Which of the following correctly describes unity of interest?

A) All of the interests must start at the same time

B) All of the interests must derive from the same document

C) All must have a leasehold interest for the same term under the same conditions and must be jointly liable for the rent

D) All must be entitled to occupy the whole of the premises

E) All of the interests must derive from the same document or from separate but identical documents which are interdependent

A

C) All must have a leasehold interest for the same term under the same conditions and must be jointly liable for the rent

(All the occupiers must be subject to the same terms/conditions of occupation and must hold the same interest in the land meaning the same lease term and restrictions. All the occupiers must also be jointly liable for the rent, meaning they are each fully liable for the whole rent, rather than just for a share of it)

254
Q

If the occupiers of a flat each begin their occupation at different times, could they still have exclusive possession of the flat? Which of the following statements correctly explains whether they can still have a lease as joint tenants?

A) No, as they do not have unity of time

B) No, as they do not have unity of interest

C) Yes, at they have unity of possession

D) Yes, providing they have the other unities

A

A) No, as they do not have unity of time

(In order for there to be unity of time, all of the interests must start at the same time, which is not the case here. If it is found that the occupants do not have the four unities, they cannot have a joint tenancy)

255
Q

Which of the following would be evidence that parties to an occupation arrangement intend to create legal relations?

A) Payment of market rent

B) Employer/employee relationship

C) Family and friend relationship between the occupier and owner

A

A) Payment of market rent

(If there is a degree of formality to the occupation agreement and/or a rent is paid then this would evidence an intention to create legal relations between the parties)

256
Q

In which of the following scenarios would something which would otherwise be a lease be merely a licence?

A) The occupation is an act of generosity from a family member

B) The parties do not enter into a written agreement

C) The rent payable is below the market average

D) The occupation is a perk of a job

A

A) The occupation is an act of generosity from a family member

(In Facchini v Bryson [1952] 1 TLR 1386, Denning LJ set out the circumstances from which a lack of intention to create legal relations can be deduced, making an arrangement simply a licence. He said that where there is a family arrangement, an act of friendship or generosity, it can be presumed that there is a lack of such intention)

257
Q

Which of the following would be classed as a service occupancy?

A) An uncle letting their niece occupy a room in a house for zero rent

B) A lawyer living in a flat above the office for her own convenience

C) A hairdresser living in the flat above the shop as a perk of the job

D) A teacher living in a house at the school in order to assist with boarding house duties

A

D) A teacher living in a house at the school in order to assist with boarding house duties

(This teacher is living at the school for the ‘better performance of his/her duties.’ This is similar to Norris v Checksfield [1991] 1 WLR 1241 and would be classed as a service occupancy)

258
Q

What presumption would the court make if an arrangement to occupy was between family members?

A) The occupation is a service occupancy

B) There is a lease rather than a licence

C) There is no exclusive possession

D) The occupation is an act of generosity

E) There is intention to create legal relations

A

D) The occupation is an act of generosity

(In Facchini v Bryson [1952] 1 TLR 1386, Denning LJ set out the circumstances from which a lack of intention to create legal relations can be deduced, making an arrangement simply a licence. He said that where there is a family arrangement, an act of friendship or generosity, it can be presumed that there is a lack of such intention. This presumption can be rebutted though, and just because an agreement is between family members/ friends, it does not mean it will not be a lease if there is a degree of formality to the arrangement / market rent is payable)

259
Q

Which of the following formalities should be used to create a three-year legal lease in which the tenant is paying the market rent for the premises?

A) A deed must be used but it need not be registered at the Land Registry

B) A deed must be used and the deed must be registered at the Land Registry

C) No formalities are required, and the lease does not need to be registered at the Land Registry

D) No formalities are required but the lease must be registered at the Land Registry

E) A contract must be used which need not be registered at the Land Registry

A

C) No formalities are required, and the lease does not need to be registered at the Land Registry

(This lease would fall within the exception for short leases of three years or fewer (LPA 1925, s 54(2)) and need not therefore be created by deed or with any formality at all. The lease is at a market rent so the conditions in the section appear to be satisfied. There is no requirement to register legal leases of seven years or fewer. Such a lease will be an overriding interest under LRA 2002, sch 3 para 1)

260
Q

Which of the following formalities should be used to create a five-year legal lease in which the tenant is paying the market rent for the premises?

A) A deed must be used but it need not be registered at the Land Registry

B) A contract must be used which need not be registered at the Land Registry

C) No formalities are required but the lease must be registered at the Land Registry

D) A deed must be used and the deed must be registered at the Land Registry

E) No formalities are required and the lease does not need to be registered at the Land Registry

A

A) A deed must be used but it need not be registered at the Land Registry

(The lease must be created by deed (LPA 1925, s 52). There is no requirement to register legal leases of seven years or fewer. Such a lease will be an overriding interest under LRA 2002, sch 3 para 1)

261
Q

In which of the following scenarios would the requirements of LPA 1925, s 54(2), in respect of short terms leases, be satisfied?

A) A landlord and tenant enter into a ten-year lease by deed, which complies with LP(MP)A 1989, s 1. The tenant registers the deed at the Land Registry

B) A landlord and tenant enter into a five-year lease by deed, which complies with LP(MP)A 1989, s 1. The tenant does not register the deed at the Land Registry

C) A landlord and tenant enter into a two-year lease by deed, which complies with LP(MP)A 1989, s 1. The tenant does not register the deed at the Land Registry

D) A landlord and tenant agree to the grant of a lease at the advertised market rent of £5000 per annum. The tenant moves in and pays the rent by monthly instalments

E) A landlord grants a lease to commence in six months’ time. The lease is for one year and the tenant will pay a market rent of £10,000 per annum

A

D) A landlord and tenant agree to the grant of a lease at the advertised market rent of £5000 per annum. The tenant moves in and pays the rent by monthly instalments

(This is a yearly periodic tenancy that has been created on the facts – the tenant has moved in and is paying an annual rent. The term is regarded as being of one year. There is no indication that any premium was payable by the tenant and a market rent is being paid. The conditions in LPA 1925, s 54(2) have therefore been satisfied)

262
Q

A landlord and a tenant enter into a seven year lease of Apartment 10 by deed. The tenant takes no further action in respect of the lease. The landlord sells the freehold estate of the apartment block (of which Apartment 10 forms part) to a new owner. Will the tenant’s seven year lease bind the new freehold owner of the apartment block?

A) Yes, as the seven-year lease does not need to be registered and is an overriding interest (LRA 2002 , schedule 3 para 1)

B) No, a lease of seven years must be registered to be legal. It will not therefore bind the new freehold owner

C) No, the lease is not for a certain term and will therefore be a licence which is a personal right in the land and will not bind the new freehold owner

D) Yes, as the seven-year lease has been registered by the tenant

A

A) Yes, as the seven-year lease does not need to be registered and is an overriding interest (LRA 2002 , schedule 3 para 1)

(A lease of seven years or fewer must be created by deed, but need not be registered. It is binding on a new freehold owner as an overriding interest)

263
Q

What are the essential requirements for the creation of a lease?

A) Certainty of term, exclusive possession and the correct formalities

B) Exclusive possession and the correct formalities.

C) Certainty of term and unity of possession.

D) Certainty of term and the correct formalities

E) Certainty of term and exclusive possession.

A

A) Certainty of term, exclusive possession and the correct formalities

264
Q

What are Prescribed Lease Clauses?

A) A list of set clauses which must be included to set out the tenant’s obligations in the lease

B) A list of set clauses which must be included to set out the landlord’s obligations in the lease

C) A list of set clauses at the front of a lease which help to speed up registration of registrable leases at the Land Registry

A

C) A list of set clauses at the front of a lease which help to speed up registration of registrable leases at the Land Registry

265
Q

What is meant by a covenant for quiet enjoyment in a lease?

A) A landlord’s covenant not to interfere with the tenant’s possession or enjoyment of the property during the term of the lease

B) A tenant’s covenant to enjoy the land quietly during the term of the lease

C) A tenant’s covenant not to cause disturbance during the term of the lease

D) A landlord’s covenant not to allow all the tenants in a building or on an estate to cause disturbance during the term of the lease

A

A) A landlord’s covenant not to interfere with the tenant’s possession or enjoyment of the property during the term of the lease

266
Q

What are some of the usual tenant’s covenants in a lease?

A) Payment of rent, payment of service charge, use of premises, quiet enjoyment

B) Payment of rent, payment of service charge, use of premises, covenant against assignment and subletting

C) Payment of rent, quiet enjoyment, use of premises, covenant against assignment and subletting

D) Payment of rent, quiet enjoyment, covenant against assignment and subletting.

E) Payment of rent, payment of service charge, insured risks

A

B) Payment of rent, payment of service charge, use of premises, covenant against assignment and subletting

(Correct. This is usual tenant covenants contained in a lease i.e. restrictions on what the tenant can/cannot do with the premises and promises it makes to the landlord)

267
Q

Which of the following statements most accurately describes the security of provisions of the Landlord and Tenant Act 1954?

A) The Act originally only covered residential tenancies, and has only recently been widened to commercial tenancies

B) The fundamental rationale has remained the same, and the detailed provisions have not changed

C) Although the detailed provisions have changed, the fundamental rationale remains the same

D) The Act is now outdated and most legal commentators now call for a new version

E) The Act has been rendered largely unnecessary by the Code for Leasing Business Premises

A

C) Although the detailed provisions have changed, the fundamental rationale remains the same

(Correct. There have changes to the detail of the Act, but the fundamental rationale remains the same)

268
Q

Which of the following is not a benefit of security of tenure for the landlord?

A) The reviewed rent may be higher for a secured tenancy

B) Security of tenure might encourage the tenant to treat the premises as their own and therefore look after them

C) The landlord may be able to show that it has complied with the Code for Leasing Business Premises

D) Security of tenure provides the landlord with greater flexibility over its own property

E) The landlord may find it easier to attract good tenants

A

D) Security of tenure provides the landlord with greater flexibility over its own property

(Correct. This is not a benefit - the landlord is more restricted in its use of the property)

269
Q

Which of the following is not a good reason for having security of tenure for business tenancies?

A) It guards against tenants facing the costs of upheaval at the end of the contractual term unless the landlord has a good reason for recovering the premises

B) It allows retail tenants to reap the benefits of acquiring goodwill at the premises

C) It is administratively simpler

D) It avoids tenants being exploited by landlords at the end of a contractual term (say with a large rent increase).

E) It gives tenants the ability to plan their business beyond the contractual term

A

C) It is administratively simpler

(Correct. This is not a good reason - it is more complex to deal with a protected tenancy)

270
Q

Which of the following arrangements could qualify for security of tenure under the Landlord and Tenant Act 1954?

A) A one year assured shorthold tenancy for a family to occupy house

B) A six month tenancy of a warehouse unit with no previous occupation

C) A monthly tenancy for a shop with a flat above it

D) A tenancy at will of an office building pending negotiations of a final lease

E) A licence for a coffee seller to take a stall in a department store

A

C) A monthly tenancy for a shop with a flat above it

(Correct. A periodic tenancy can qualify for security of tenancy. The flat does not prevent the premises from qualifying as it is a subsidiary residential use)

271
Q

A farmer let’s a grazing field and cowshed to a neighbouring farmer for a term of twelve months. The farmer does not go through the contracting out procedure of the Landlord and Tenant Act 1954.

Which of the following statements regarding security of tenure is most accurate?

A) The tenancy would not have security of tenure under the Landlord and Tenant Act 1954 because it is only a 12 month term

B) The tenancy does not have security of tenure under the Landlord and Tenant Act 1954 because it is an agricultural tenancy

C) The tenancy would not have security of tenure under the Landlord and Tenant Act 1954 because it is a service tenancy

D) The tenancy would have security of tenure under the Landlord and Tenant Act 1954 provided the neighbouring farmer uses the land for its farming business

E) The tenancy would have security of tenure under the Landlord and Tenant Act 1954 because the contracting out procedure wasn’t followed

A

B) The tenancy does not have security of tenure under the Landlord and Tenant Act 1954 because it is an agricultural tenancy

(Correct. As an agricultural tenancy it would not qualify for security of tenure under the Landlord and Tenant Act 1954. It may qualify for a different type of security of tenure under the agricultural tenancy regime, but that is beyond the scope of this course)

272
Q

A tenant has agreed to take a lease contracted out of the security of tenure provisions of the Landlord and Tenant Act 1954.

The landlord served notice of contracting out 7 days ago and completion is taking place today. What is needed to ensure that the lease is validly contracted out?

A) As insufficient time has elapsed, a court order approving the contracting out

B) An ordinary declaration and reference to both the notice and statutory declaration in the lease itself

C) An ordinary declaration and a statutory declaration and reference to both the notice and statutory declaration in the lease itself

D) A statutory declaration and reference to the statutory declaration in the lease itself

E) A statutory declaration and reference to both the notice and statutory declaration in the lease itself

A

E) A statutory declaration and reference to both the notice and statutory declaration in the lease itself

(Correct. As 14 days have not elapsed since the landlord’s notice, a statutory declaration is required. It is also necessary to refer to both notice and statutory declaration in the lease itself)

273
Q

In which of the following scenarios is the landlord not expected to act reasonably in refusing its consent to allow alterations?

A) The lease contains an absolute covenant against structural alterations and the tenant requests a one-off consent from the landlord to an extension affecting the structure of the property

B) The lease contains a fully qualified covenant against non-structural alterations and the tenant requests consent from the landlord to construct partition walls inside the property

C) The lease contains a qualified covenant against non-structural alterations and the tenant requests consent from the landlord to make improvements to the property

A

A) The lease contains an absolute covenant against structural alterations and the tenant requests a one-off consent from the landlord to an extension affecting the structure of the property

(Correct. This is an absolute covenant and the statute has no effect here. The landlord does not need to grant consent or act reasonably in considering the request)

274
Q

A tenant may be subject to repair obligations under a repairing covenant. What is the tenant’s standard of repair?

A) The standard of repair is that which is set out in the express repair covenant

B) The standard of repair is to keep the premises in the condition in which they would be kept by a reasonably minded owner, having regard to the character and type and age of the premises

C) The standard of repair is to keep the premises in the same condition as they were in when the lease was granted

D) The standard of repair is to keep the premises in the condition in which they would be kept by the owner of the reversion, having regard to the character and type and age of the premises

A

B) The standard of repair is to keep the premises in the condition in which they would be kept by a reasonably minded owner, having regard to the character and type and age of the premises

275
Q

What is meant by a tenant’s covenant to keep the premises in repair?

A) It is an obligation to leave the premises in a state of repair at the end of the term of the lease

B) It is an obligation to keep the premises in repair throughout the term of the lease

C) It is an obligation to put the premises into repair at the beginning of the term if they are out of repair and to leave them in a state of repair at the end of the term of the lease

D) It is an obligation to put the premises into repair at the beginning of the term if they are out of repair and to keep them in repair throughout the term of the lease

A

D) It is an obligation to put the premises into repair at the beginning of the term if they are out of repair and to keep them in repair throughout the term of the lease

(Correct. This obligation is therefore quite onerous if the premises are in a state of disrepair at the beginning of the lease term)

276
Q

If a tenant’s covenant states that the tenant must not alter the property without the landlord’s consent, what sort of covenant is this?

A) Qualified

B) Absolutely qualified

C) Fully qualified

D) Absolute

A

A) Qualified

277
Q

A lease contains a clause prohibiting assignment of the whole premises without the consent of the landlord, such consent not to be unreasonably withheld. The landlord refuses consent to the tenant who asks for permission to assign the lease. In which case would the court consider it reasonable for the landlord to refuse its consent to the assignment?

A) The landlord considers that an assignment to the proposed assignee would affect the value of the freehold land

B) The landlord has received a poor financial reference from the proposed assignee

C) The landlord does not want to lease to students and the proposed assignee is a student

D) The landlord would prefer the tenant to surrender the lease rather than assign it to the proposed assignee

A

B) The landlord has received a poor financial reference from the proposed assignee

(Correct. The landlord has refused consent on the basis of the landlord/tenant relationship. This is entirely reasonable)

278
Q

What is an assignment?

A) When the tenant purchases the landlord’s interest and becomes its own landlord

B) When the tenant sells the remainder of their lease to another party

C) When the tenant grants a new lease to another party

D) When the tenant hands the lease back to the landlord with the landlord’s consent

A

B) When the tenant sells the remainder of their lease to another party

(Correct. This is very common in practice, especially where the lease is longer than 5-10 years)

279
Q

A tenant requests permission from the landlord to assign a new lease. In which of the following scenarios is the landlord acting reasonably?

A) The landlord refuses consent because the tenant has not reinstated the property to its original specification which the landlord has recently imposed as a condition of assignment. The tenant refuses to enter into an authorised guarantee agreement to the landlord although this is required by the terms of the lease

B) The landlord refuses consent because the tenant has not reinstated the property to its original specification which the landlord has recently imposed as a condition of assignment. However, the tenant has agreed to provide an authorised guarantee agreement to the landlord as required by the terms of the lease

C) The landlord refuses consent because the tenant has not reinstated the property to its original specification which was a condition of assignment set out in the lease when it was granted. However, the tenant has agreed to provide an authorised guarantee agreement to the landlord as required by the terms of the lease

A

C) The landlord refuses consent because the tenant has not reinstated the property to its original specification which was a condition of assignment set out in the lease when it was granted. However, the tenant has agreed to provide an authorised guarantee agreement to the landlord as required by the terms of the lease

280
Q

A tenant asks for permission to assign their three year lease (which was created informally) to which the landlord agrees. What are the formalities for the assignment of the lease?

A) A contract signed by the landlord and the tenant

B) A valid deed and registration

C) Writing signed by the landlord.

D) A valid deed

E) No particular formalities are required as the lease was created informally

A

D) A valid deed

(Correct. LPA 1925, s 52 requires a deed to be used to transfer a legal estate in land. This applies even if the lease has been created under LPA 1925, s 54(2) (i.e. with no formalities))

281
Q

A landlord refuses permission to a tenant to assign the lease pursuant to a fully qualified alienation covenant. Which of the following statements is correct in relation to the refusal of consent?

A) A landlord’s consent is reasonably withheld if the refusal is designed to achieve a collateral purpose unconnected with the landlord and tenant relationship, and the burden of proving that a refusal of consent is reasonable is on the landlord

B) A landlord’s consent is reasonably withheld if the detriment to the tenant is disproportionate to the benefit to the landlord, and the burden of proving that consent has been unreasonably withheld is on the tenant

C) A landlord’s consent is unreasonably withheld if the refusal is designed to achieve a collateral purpose unconnected with the landlord and tenant relationship, and the burden of proving that a refusal of consent is reasonable is on the landlord

D) A landlord’s consent is reasonably withheld if the detriment to the tenant is disproportionate to the benefit to the landlord, and the burden of proving that a refusal of consent is reasonable is on the landlord

E) A landlord’s consent is unreasonably withheld if the refusal is designed to achieve a collateral purpose unconnected with the landlord and tenant relationship, and the burden of proving that consent has been unreasonably withheld is on the tenant

A

C) A landlord’s consent is unreasonably withheld if the refusal is designed to achieve a collateral purpose unconnected with the landlord and tenant relationship, and the burden of proving that a refusal of consent is reasonable is on the landlord

282
Q

A landlord is considering whether to grant permission to a tenant to sublet the property pursuant to a fully qualified alienation covenant. Which of the following statements is correct?

A) The landlord must give written consent within 14 days and is entitled to request additional information to enable them to make a decision. The landlord must provide written reasons if they refuse consent

B) The landlord must give written consent within 28 days but is entitled to additional time after the 28 day period has expired to request additional information to enable them to make a decision. The landlord must provide written reasons if they refuse consent

C) The landlord must give written consent within 28 days and is entitled to request additional information to enable them to make a decision. The landlord need not provide any reasons if they refuse consent

D) The landlord must give written consent within 28 days and is entitled to request additional information to enable them to make a decision. The landlord may provide written reasons if they refuse consent

E) The landlord must give written consent within 28 days and is entitled to request additional information to enable them to make a decision. The landlord must provide written reasons if they refuse consent

A

E) The landlord must give written consent within 28 days and is entitled to request additional information to enable them to make a decision. The landlord must provide written reasons if they refuse consent

(Correct. This is as per Landlord and Tenant Act 1988, s 1(6))

283
Q

Which of the following is the best definition of a leasehold covenant?

A) A promise contained in a lease given by the landlord to the tenant

B) A Promise contained in a lease given by the tenant to the landlord

C) A promise contained in a deed

D) A promise contained in a lease given by a landlord or a tenant

A

D) A promise contained in a lease given by a landlord or a tenant

284
Q

A lease is entered into on 1 December 1990. What kind of lease is this?

A) Old lease governed by the LTA 1995

B) Old lease governed by the old system of rules

C) New lease governed by the LTA 1995

A

B) Old lease governed by the old system of rules

(Correct. The lease was entered into prior to 1 January 1996. It is therefore an ‘old lease’ and is governed by the old rules, not the LTA 1995. Though, as you will see, the LTA 1995 has some provisions that apply retrospectively to both old and new leases)

285
Q

A landlord granted a new lease to a tenant. The tenant assigned the lease to Assignee 1 with the landlord’s consent and gave the landlord an authorised guarantee agreement. Assignee 1 has recently assigned the lease to Assignee 2 without the landlord’s consent.

If Assignee 2 were to breach any of the tenant’s covenants, which of the following statements would be correct?

A) The landlord may pursue the tenant or Assignee 2 for a remedy for the breaches of covenant

B) The landlord may pursue Assignee 2 only for a remedy for the breaches of covenant

C) The landlord may pursue Assignee 1 or Assignee 2 for a remedy for the breaches of covenant

D) The landlord may pursue the tenant, Assignee 1 or Assignee 2 for a remedy for the breaches of covenant

A

D) The landlord may pursue the tenant, Assignee 1 or Assignee 2 for a remedy for the breaches of covenant

(Correct. The burden of the tenant’s covenants passes to Assignee 2 under the LTA 1995, s 3. Assignee 1 has made an excluded assignment under the LTA 1995, s 11. Assignee 1 is therefore not automatically released from liability under the LTA 1995, s 5. Consequently, the AGA entered into by the tenant cannot fall away because the tenant’s immediate assignee remains liable for the tenant’s covenants (as per the LTA 1995, ss 11(2) and 16(4)))

286
Q

A landlord and tenant enter into a lease. The landlord subsequently sells the freehold reversion to a third party (the ‘reversioner’) and the tenant then assigns the lease to a third party (the ‘assignee’).

Who has privity of contract under the lease in the circumstances?

A) The assignee and the reversioner

B) The landlord and the assignee

C) The landlord and the reversioner

D) The tenant and the assignee

E) The landlord and the tenant

A

E) The landlord and the tenant

(This is correct. Between the original landlord and original tenant there is a contractual relationship ie privity of contract exists here)

287
Q

Two years ago, a landlord granted a ten-year lease to a tenant. The tenant has assigned the lease to the assignee. The assignee is now in breach of several leasehold covenants.

Which statement best describes the liability of the tenant for the breaches of covenant?

A) The tenant is liable for breaches for the entire term of the lease

B) The tenant was automatically released from liability upon assignment of the lease to the assignee

C) The tenant was released from liability if the landlord granted it a release

D) The tenant is liable for the assignee’s breaches but only until the assignee assigns the lease

A

B) The tenant was automatically released from liability upon assignment of the lease to the assignee

(Correct. This is because of the operation of the LTCA 1995, s 5)

288
Q

A landlord granted a 40-year legal lease to a tenant on 31 November 1995. Three years later, the tenant assigned the lease to the assignee. The assignee has breached one of the leasehold covenants. Which statement best describes the liability of the tenant?

A) The tenant was released from liability upon assignment of the lease to the assignee

B) The tenant is liable for the assignee’s breaches but only until the assignee assigns the lease

C) The tenant is liable for the entire term of the lease

D) The tenant was released from liability because it no longer has privity of estate with the landlord

A

C) The tenant is liable for the entire term of the lease

(Correct. The original tenant has privity of contract and is liable for its assignees for the entire term of the lease)

289
Q

A landlord granted a 40-year legal lease to a tenant on 1 January 1994.

Three years later, the tenant assigned the lease to Assignee 1.

Four years later, Assignee 1 assigned the lease to Assignee 2.

Assignee 2 made a direct covenant with the landlord to observe the leasehold covenants.

Ten years later, Assignee 2 assigned the lease to Assignee 3, who is still the tenant.

Which statement best sets out the parties whom the landlord may choose to sue for the breach of covenant?

A) The original tenant, Assignee 3

B) The original tenant, Assignee 1, Assignee 2

C) The original tenant, Assignee 2

D) The original tenant, Assignee 1, Assignee 2, Assignee 3

E) The original tenant, Assignee 2, Assignee 3

A

E) The original tenant, Assignee 2, Assignee 3

(Correct. The original tenant is liable as it remains so for the entire term; Assignee 2 is liable for the entire term as it has made a direct covenant to that effect with the landlord; Assignee 3 has privity of estate with the landlord and is liable as the current tenant. Assignee 1 has neither privity of contract nor privity of estate)

290
Q

Ten years ago, a landlord granted a lease to a tenant. The lease was assigned five years ago to Assignee 1, and the tenant provided an authorised guarantee agreement to the landlord. The lease was then assigned two years ago to Assignee 2, and Assignee 1 provided an authorised guarantee agreement to the landlord. Assignee 2 is now in breach of the repair covenant.

Which of the following statements is correct in relation to suing a former leasehold owner for the breach of covenant?

A) The landlord should sue Assignee 1 for damages. Assignee 1 may try to recoup its losses via an express or common law indemnity from Assignee 2

B) The landlord should sue the tenant or Assignee 1 for damages. The tenant may try to recoup its losses via an express indemnity from Assignee 1 or via a common law indemnity from Assignee 2. Assignee 1 may try to recoup its losses via an express or common law indemnity from Assignee 2

C) The landlord should sue the tenant. The tenant may try to recoup its losses via an express indemnity from Assignee 1 or via a common law indemnity from Assignee 2

A

A) The landlord should sue Assignee 1 for damages. Assignee 1 may try to recoup its losses via an express or common law indemnity from Assignee 2

(Correct. The tenant is no longer liable as the authorised guarantee agreement falls away under the LTA 1995, s 16(4) when Assignee 1 assigns the lease. Assignee 1 is liable under its authorised guarantee agreement and could try to recoup any losses from Assignee 2 (if possible) if there is an express indemnity between them or via the common law rule from Moule v Garrett (1872))

291
Q

Two years ago, a landlord granted a lease to a tenant. A year ago, the tenant granted a sublease to the subtenant. The sublease contains the same leasehold covenants as the headlease. The subtenant is now in breach of the user covenant and the decorating covenant.

Which of the following statements correctly describes who the landlord could sue?

A) The landlord may sue the subtenant for the breach of the user covenant and the decorating covenant

B) The landlord may sue the tenant for the breach of the user covenant and the subtenant for the breach of the decorating covenant

C) The landlord may sue the tenant for the breach of the user covenant and the decorating covenant. The landlord does not have a direct relationship with the subtenant

D) The landlord may sue the subtenant for the breach of the user covenant and the tenant for the breach of the decorating covenant

A

D) The landlord may sue the subtenant for the breach of the user covenant and the tenant for the breach of the decorating covenant

(Correct. The LTCA 1995, s 3(5) allows the landlord to sue a subtenant directly for breach of a restrictive covenant. The landlord would have to sue the tenant for the breach of the decorating covenant as this covenant is positive and is also in the headlease (as the sublease contains the same leasehold covenants as the headlease))

292
Q

A landlord sues a former tenant who is liable under an authorised guarantee agreement for the current tenant’s breach of the alterations covenant in the lease.

Which of the following statements is correct?

A) The landlord does not need to serve a notice on the former tenant under the LTA 1995, s 17. The former tenant may however apply for an overriding lease to maintain some control over the actions of the current tenant

B) The landlord must serve a notice on the former tenant under the LTA 1995, s 17. The former tenant may then apply for an overriding lease to maintain some control over the actions of the current tenant

C) The landlord does not need to serve a notice on the former tenant under the LTA 1995, s 17. The former tenant may not apply for an overriding lease to maintain some control over the actions of the current tenant

D) The landlord must serve a notice on the former tenant under the LTA 1995, s 17. The former tenant may not apply for an overriding lease to maintain some control over the actions of the current tenant

A

C) The landlord does not need to serve a notice on the former tenant under the LTA 1995, s 17. The former tenant may not apply for an overriding lease to maintain some control over the actions of the current tenant

(Correct. The landlord is not suing the former tenant for a ‘fixed charge’ so there is no need to serve a notice under the LTA 1995, s 17, and the tenant may not therefore apply for an overriding lease)

293
Q

A rent review has taken place in accordance with the rent review provisions set out in the lease and the rent has been increased. The landlord sues the former tenant who is liable under an authorised guarantee agreement for the current tenant’s breach of the rent covenant.

Which of the following statements is correct?

A) The former tenant is liable for the increased amount of rent in the lease

B) The former tenant is only liable for the increased amount of rent in the lease if the rent review took place before the lease was assigned

C) The former tenant is not liable for the increased amount of rent in the lease, only for the original rent

A

A) The former tenant is liable for the increased amount of rent in the lease

(Correct. This was a foreseen variation under the LTA 1995, s 18)

294
Q

Which of the following statements is correct in relation to an overriding lease granted pursuant to the LTA 1995, s 19?

A) If the former tenant requests an overriding lease, the landlord may choose whether to grant it. The overriding lease will last for the length of the original lease plus 3 days and the landlord and the former tenant may choose the terms of the overriding lease

B) If the former tenant requests an overriding lease, the landlord may choose whether to grant it. The overriding lease will last for the length of the original lease plus 3 days and contain the same covenants

C) If the former tenant requests an overriding lease, the landlord is obliged to grant it. The overriding lease will last for the length of the original lease plus 3 days and contain the same covenants

D) If the former tenant requests an overriding lease, the landlord is obliged to grant it. The overriding lease will last for the length of the original lease plus 3 days and the landlord and the former tenant may choose the terms of the overriding lease

A

C) If the former tenant requests an overriding lease, the landlord is obliged to grant it. The overriding lease will last for the length of the original lease plus 3 days and contain the same covenants

(Correct. This is set out in the LTA 1995, s 19)

295
Q

A landlord sues a former tenant who is liable as the original tenant of an old lease for the current tenant’s breach of the rent covenant in the lease.

Which of the following statements is correct?

A) The landlord must serve a notice on the former tenant under the LTA 1995, s 17. The former tenant may not apply for an overriding lease to maintain some control over the actions of the current tenant

B) The landlord must serve a notice on the former tenant under the LTA 1995, s 17. The former tenant may then apply for an overriding lease to maintain some control over the actions of the current tenant

C) The landlord does not need to serve a notice on the former tenant under the LTA 1995, s 17. The former tenant may not apply for an overriding lease to maintain some control over the actions of the current tenant

D) The landlord does not need to serve a notice on the former tenant under the LTA 1995, s 17. The former tenant may however apply for an overriding lease to maintain some control over the actions of the current tenant

E) The landlord does not need to serve a notice on the former tenant under the LTA 1995, s17. The former tenant may not apply for an overriding lease to maintain some control over the actions of the current tenant. This is an old lease so the LTA 1995 does not apply

A

B) The landlord must serve a notice on the former tenant under the LTA 1995, s 17. The former tenant may then apply for an overriding lease to maintain some control over the actions of the current tenant

(Correct. The landlord is suing the former tenant for a ‘fixed charge’ so must serve a notice under the LTA 1995, s 17, and the tenant may then apply for an overriding lease)

296
Q

A tenant fails to carry out repairs to the property in accordance with their repairing covenant. Which remedy should the landlord be advised to pursue?

A) Distress

B) Inhibitory injunction

C) Damages

D) Specific performance

A

C) Damages

(Correct. A landlord is extremely unlikely to be awarded any other remedy by the court. This is the best remedy for a landlord to pursue in the circumstances)

297
Q

Which of the following would not be an appropriate remedy for a landlord to pursue in respect of a tenant’s failure to pay the rent due under the lease?

A) Commercial rent arrears recovery

B) Forfeiture

C) Specific performance

D) Action for debt

A

C) Specific performance

(Correct. This remedy is not appropriate in the event of non payment of a rent covenant)

298
Q

What is the definition of commercial rent arrears recovery?

A) Seven days’ notice served on a commercial tenant that the landlord is going to seize goods from the leasehold property because of a failure to pay rent

B) Seven days’ notice served on a commercial or a residential tenant that the landlord is going to seize goods from the leasehold property because of a failure to pay rent

C) Seven days’ notice served on a commercial tenant that the landlord is going to seize goods from a leasehold property because of a failure to comply with a non-rent covenant

A

A) Seven days’ notice served on a commercial tenant that the landlord is going to seize goods from the leasehold property because of a failure to pay rent

299
Q

What is a break clause?

A) When either a landlord or a tenant is allowed by the lease to bring the lease to a premature end

B) When the tenant hands the lease back to the landlord with the landlord’s consent

C) When either a landlord or a tenant serves a notice to quit

D) When the tenant purchases the landlord’s interest and becomes its own landlord

A

A) When either a landlord or a tenant is allowed by the lease to bring the lease to a premature end

(Correct. A break clause may allow the landlord or the tenant or both parties to bring the lease to a premature end)

300
Q

How much notice must either a landlord or a tenant serve to bring an annual periodic tenancy to an end?

A) Half a year’s notice

B) A month’s notice

C) A quarter’s notice

D) A year’s notice

A

A) Half a year’s notice

(Correct. It is necessary to serve at least half a year’s notice to end an annual periodic tenancy)

301
Q

A tenant is in breach of its rent covenant and the landlord wishes to bring the lease to an end. Which method of terminating the lease would you advise the landlord to use?

A) Forfeiture

B) Effluxion of time

C) Notice to quit

D) Surrender

A

A) Forfeiture

(Correct. This is the procedure to follow when a landlord is looking to terminate a lease early for breach of a tenant covenant)

302
Q

A tenant pays monthly rent in advance under the terms of their lease. The tenant has failed to pay the rent for the last three months and the landlord now wishes to forfeit the lease. The landlord’s agent has sent a rent demand for the next month’s rent but is unaware of the breach of the rent covenant.

Which of the following statements is correct about whether the landlord has a right to forfeit?

A) The landlord may not forfeit the lease as the agent (who acts for the landlord) has waived the right to forfeit. The landlord’s right to forfeit will arise in the subsequent month if no rent demand is sent out for that month’s rent payment

B) The landlord may forfeit the lease as the landlord did not send out the rent demand; therefore there has been no waiver

C) The landlord may not forfeit the lease as the agent (who acts for the landlord) has permanently waived the right to forfeit as this is a non-continuing breach

D) The landlord may forfeit the lease as the agent, who acts for the landlord, was unaware of the breach of covenant; therefore there has been no waiver

A

A) The landlord may not forfeit the lease as the agent (who acts for the landlord) has waived the right to forfeit. The landlord’s right to forfeit will arise in the subsequent month if no rent demand is sent out for that month’s rent payment

(Correct. This is a waiver. The landlord had knowledge of the breach and the agent, who acts for the landlord, has acted unequivocally by sending out a rent demand for next month’s rent. This is a non-continuing breach; however, the right to forfeit may arise again in the subsequent month if there is a breach of the next rent covenant)

303
Q

Which of the following statements is correct in respect of exercising the right to forfeit?

A) In the case of mixed use business and residential premises, a landlord may forfeit by either peaceable re-entry or by court order

B) The type of premises is irrelevant; a landlord may choose to forfeit by either peaceable re-entry or by court order

C) In the case of business premises, a landlord may forfeit by either peaceable re-entry or by court order

D) In the case of residential premises, a landlord may forfeit by either peaceable re-entry or by court order

A

C) In the case of business premises, a landlord may forfeit by either peaceable re-entry or by court order

(Correct. In the case of residential or mixed use premises however, a landlord may only forfeit by court order)

304
Q

Which of the following statements is correct?

A) A right to forfeit for non-payment of rent is implied into a legal lease

B) A right to forfeit for non-payment of rent is implied into an equitable lease

C) A right to forfeit for breach of a non-rent covenant is implied into an equitable lease

D) A right to forfeit for non-payment of rent and for breach of a non-rent covenant are implied into a legal lease

E) A right to forfeit for non-payment of rent and for breach of a non-rent covenant are implied into an equitable lease

A

B) A right to forfeit for non-payment of rent is implied into an equitable lease

(Correct. A right to forfeit (for non-payment of rent) is implied into equitable leases only. In legal leases the right to forfeit for any breach of covenant must be expressly reserved)

305
Q

Which of the following breaches of covenant is classed as non-continuing?

A) Using the property for an illegal use in breach of the covenant against immoral or illegal use

B) Failing to insure the property in breach of the covenant to insure

C) Failing to fix the roof of the property in breach of the repair covenant

D) Using the property to run a business instead of as a residence in breach of the user covenant

E) Subletting the property without the landlord’s consent

A

E) Subletting the property without the landlord’s consent.

(Correct. This is a breach of covenant that can only be committed once and cannot be undone)

306
Q

A landlord wishes to forfeit a lease for non-payment of rent by the tenant. Which of the following statements is correct?

A) The landlord must make a formal demand for the rent due on the day when it becomes payable unless the lease waives this requirement

B) The landlord must serve a s 146 statutory notice on the tenant

C) The landlord must make a formal demand for the exact amount of rent due on any day after it becomes payable upon the premises between the hours of sunrise and sunset

D) The landlord can immediately forfeit the lease by court order or peaceable re-entry

A

A) The landlord must make a formal demand for the rent due on the day when it becomes payable unless the lease waives this requirement

(Correct. A formal demand for the rent due must have been made unless the lease waives this requirement, which it usually does by expressing the rent as being payable ‘whether formally demanded or not’)

307
Q

A landlord forfeits a lease for non-payment of rent via a court order.

Which of the following statements is correct in respect of when relief may be granted?

A) The tenant may be granted relief after the date of the court order if all the arrears and costs are paid

B) The tenant may be granted relief after the date of the court order if all the arrears and costs are paid even if the landlord has granted a new lease to a third party

C) The tenant may be granted relief up to a year after the date of the court order

D) The tenant will not be granted relief after the date of the court order even if all the arrears and costs are paid

A

A) The tenant may be granted relief after the date of the court order if all the arrears and costs are paid

(Correct. The court has a discretion to grant relief even after it has made an order to forfeit the lease on condition that all arrears and costs are paid)

308
Q

A landlord forfeits a lease by peaceable re-entry for non-payment of rent. Which of the following statements is correct?

A) The tenant may be granted relief after the date of the peaceable re-entry if the application for relief is made within six months of the peaceable re-entry

B) The tenant may be granted relief after the date of the peaceable re-entry if all the arrears and costs are paid

C) The tenant may be granted relief after the date of the peaceable re-entry if the circumstances are exceptional

D) The tenant will not be granted relief after the date of the peaceable re-entry even if all the arrears and costs are paid

A

B) The tenant may be granted relief after the date of the peaceable re-entry if all the arrears and costs are paid

(Correct. The court has an inherent equitable jurisdiction to grant relief to the tenant; this may even be some time after the landlord has forfeited the lease)

309
Q

In which of the following scenarios is a s 146 notice required to be served on the tenant by the landlord?

A) Where there has been a breach of the covenant to repair

B) Where there has been a breach of any covenant other then the covenant to pay rent

C) Where there has been a breach of the covenant to pay rent

D) Where the breach is capable of remedy by the tenant

A

B) Where there has been a breach of any covenant other then the covenant to pay rent

(This is correct. Whenever there has been a breach of a covenant other than the covenant to pay rent, the landlord must serve a s 146 notice on the tenant if it wishes to forfeit the lease. This applies regardless of whether the breach is capable of remedy or not. Note, there is additional protection if the breach is of a repair covenant. The landlord (in addition to serving the s 146 notice) must also inform the tenant of its rights (under the Leasehold Property (Repairs) Act 1938) to serve a counter notice within 28 days. If the tenant serves a counter notice, the landlord cannot proceed to claim forfeiture or damages without first obtaining the lease of the court)

310
Q

Which of the following covenants is unlikely to be capable of remedy?

A) Failure to decorate the premises

B) Failure to insure the premises

C) Change of use in breach of covenant

D) Subletting in breach of covenant

E) Unauthorised alterations to the premises

A

D) Subletting in breach of covenant

(Correct. This is a breach of covenant that can only be committed once and cannot be undone. It is unlikely to be capable of remedy)

311
Q

You act for a refuse collection and disposal company who recently entered into negotiations with Waste Limited, the owner of a landfill site. Waste Limited has offered your client the opportunity to use the site. The first offer is a 12-month licence to use the site for a payment of £200,000. The second (alternative) offer is the grant of a 12-month lease of a specified part of the site for a payment of £225,000. In either case the maximum amount of refuse that your client may deposit is 25 metric tonnes.

Which of the following options is the best advice to your client as to which offer they should accept?

A) Your client should accept the licence because it would give it a proprietary right in the land

B) Your client should accept the lease because it would enable it to recover use of the right to use the site if Waste Limited subsequently tried to revoke the right

C) Your client should accept the lease because it would give it a personal right in the land

D) Your client should accept the licence because it is capable of being enforced against third parties should Waste Limited sell the site

E) Your client should accept the lease because it will entitle R to sue for breach of contract should Waste Limited breach the terms

A

B) Your client should accept the lease because it would enable it to recover use of the right to use the site if Waste Limited subsequently tried to revoke the right

(This is correct. A lease is a proprietary right in the land. This means it is enforceable in rem, the right can be recovered. A licence is a personal right, which means the right cannot be recovered if Waste subsequently tried to revoke it, because a personal right is enforceable in personam)

312
Q

A buyer and seller enter into an agreement to buy/sell a freehold estate. They enter into a document which is described as a contract for the sale of land. The document contains all the agreed terms and is signed by the seller in the presence of a witness who attests their signature.

Which of the following options correctly describes the validity of this document?

A) The document is valid because it complies with the requirements of a valid contract for sale at law

B) The document is invalid because it does not comply with the requirements for a land contract

C) The document is valid because it complies with the requirements for a land contract

D) The document is invalid because it has not been registered at the Land Registry yet

E) The document is valid because it complies with the requirements for a deed

A

B) The document is invalid because it does not comply with the requirements for a land contract

(This is correct. A contract for the sale of land must comply with LP(MP)A 1989, s 2; it must be in writing, contain all the agreed terms and be signed by both parties. The document is not a valid contract as it has not been signed by both the buyer and the seller.
There is no valid deed as the document has not been intended as a ‘deed’ or delivered. Further, the parties are trying to enter into an contract, rather than the actual transfer of the land.
If this had been validly created, it would grant the buyer an equitable interest in the land called an ‘estate contract’)

313
Q

Your client recently entered into a contract for lease for a unit in a shopping centre. The written contract was signed by both the landlord and your client and contained all the agreed terms.

Which of the following options describes the interest yours holds in the shopping centre?

A) A legal easement

B) An interest in a trust of land

C) A restrictive covenant

D) An estate contract

E) An equitable easement

A

D) An estate contract

(This is correct. An estate contract is a contractual right to an estate in the land. Here, your client has a contract for a leasehold estate. It has been validly created as it complies with LP(MP)A 1989, s 2)

314
Q

Your client owns the freehold to wine bar with a popular roof terrace. The adjoining property is a shop, which sells drones. The shop owner frequently flies the drones above the roof terrace to demonstrate them to customers, to the annoyance of your client and their wine bar customers. Your client wishes to apply for an injunction to prevent the shop owner from trespassing on the land.

Which statement is the best advice for your client as to their chances of success in the proceedings?

A) Your client is unlikely to be successful as the drone is not causing any physical damage to their land

B) Your client is unlikely to be successful as the drone would be classed as being in the upper airspace

C) Your client is unlikely to be successful as the Civil Aviation Act 1982, s 76 grants immunity from trespass for aircrafts

D) Your client is likely to be successful as the shop owner is trespassing into your client’s lower airspace

E) Your client is likely to be successful as a freehold estate extends up to the heavens above

A

D) Your client is likely to be successful as the shop owner is trespassing into your client’s lower airspace

(This is correct. A freehold estate includes the ‘lower’ airspace. This is defined in Bernstein v Skyviews [1978] as the height which is necessary for the ordinary use and enjoyment of the land. The ordinary use and enjoyment of this land would include the peaceful enjoyment of the roof terrace)

315
Q

The owner of a registered freehold property granted a legal mortgage in favour of a lender as security for a loan. The owner built a summer house in the property’s garden in which they regularly spend time working from home.

In which of the following scenarios is the summer house likely to be a chattel which does not form part of the mortgaged property?

A) The summer house is not specified as forming part of the security in the mortgage agreement

B) The summer house complements the design of the rest of the garden

C) The summer house has its own water supply

D) The summer house would be destroyed if it were removed from the land

E) The summer house is a moveable structure

A

E) The summer house is a moveable structure

(This is correct. After considering the degree of annexation and purpose of annexation tests, if the summer house can be moved or removed without being damaged or destroyed then a court is likely to hold it is a chattel as per the case of Elitestone v Morris (1997))

316
Q

Your client has agreed to purchase the unregistered freehold of a large farm. The seller signs and dates the transfer deed in the presence of a witness who attests the seller’s signature.

Which of the following statements best describes your client’s position in respect of the legal ownership of the farm?

A) The client holds the legal title but it will revert back to the seller if not registered within two months

B) The transfer deed is of no effect because it does not include all of the terms expressly agreed by the parties

C) The transfer will not take effect at law until title is registered with the Land Registry

D) The seller holds the property on trust for the client until the transfer is registered with the Land Registry

E) The transfer deed is of no effect because the client has not signed the document

A

A) The client holds the legal title but it will revert back to the seller if not registered within two months

(This is correct. The transfer of the unregistered freehold has been effected by deed so the legal title passes to the client on completion, who then has two months to register the land for the first time. See ss 4, 6 and 7 Land Registration Act 2002)

317
Q

A purchaser has agreed to buy the registered freehold of a large office block. The buyer and seller both sign a document headed ‘deed’ and their signatures are attested by a witness. The document is then dated.

Which of the following statements best describes the legal position?

A) The purported deed is defective and the purchaser has no interest in the property

B) The transfer will not take effect at law until title is registered with the Land Registry

C) The purported deed is defective but a contract may be construed from the failed grant

D) The purchaser holds the legal title but it will revert back to the seller if not registered within two months

E) The purchaser holds the legal title to the property

A

B) The transfer will not take effect at law until title is registered with the Land Registry

(This is the best answer. The transfer must be completed by registration to take effect at law. See LRA 2002, s27(2)(a) and s27 (1))

318
Q

A borrower and lender enter into a document which is described as a ‘mortgage deed’. The document purports to grant a mortgage over the borrower’s registered legal freehold. The agreement is signed by both the borrower and lender, witnessed and then dated. The lender does not do anything further with the document.

Which of the following options best describes what kind of mortgage (if any) has been granted by the borrower?

A) The borrower has granted an equitable mortgage because it is a mortgage of an equitable interest in the land

B) The borrower has granted a legal mortgage because the statutory requirements of a deed have been met

C) The borrower has not granted a mortgage because the document has not been registered

D) The borrower has granted an equitable mortgage. Equity will recognise the ‘failed legal mortgage’ as a ‘contract to grant a legal mortgage’

E) The borrower has not granted a mortgage because the document does not comply with the statutory requirements of a deed

A

D) The borrower has granted an equitable mortgage. Equity will recognise the ‘failed legal mortgage’ as a ‘contract to grant a legal mortgage’

(This is correct. Although a valid deed has been created on the facts - the document complies with LP(MP)A 1989, s 1 - it has not been registered by the lender. No valid legal mortgage has therefore been created on the facts. Equity will, however, recognise this an equitable mortgage in the circumstances. The document complies with LP(MP)A 1989, s 2 and equity will therefore recognise it as a ‘contract to grant a legal mortgage’. An equitable mortgage does not need to be registered to be validly created)

319
Q

A lender loans money to a borrower in return for a charge over the borrower’s registered land. The interest rate for the loan is 10% above the Bank of England’s base rate.

Which of the following statements best sets out the circumstances in which the interest rate is most likely to be held to be unconscionable?

A) The land is a freehold estate, the borrower is a commercial owner and both lender and borrower are individuals

B) The land is a leasehold estate, the borrower is a commercial tenant and has also received legal advice

C) The land is a freehold estate, the borrower is a residential owner and has also received legal advice

D) The land is a freehold estate, the borrower is a residential owner and the loan is to fund the borrower’s business expansion

E) The land is a leasehold estate, the borrower is a residential tenant and the loan is to pay for the extension of the leasehold term

A

E) The land is a leasehold estate, the borrower is a residential tenant and the loan is to pay for the extension of the leasehold term

(This is correct and the facts are akin to the case of Cityland v Dabrah (1968). As with this case, if the property is a leasehold and the borrower is a residential tenant who needs the money to extend the length of the lease, there is a risk that the lender could take advantage of the borrower’s circumstances and impose a higher interest rate. The borrower is more likely to agree to unconscionable terms to ensure the loan is made)

320
Q

A couple recently bought a holiday home in the Alps. In order to fund the purchase they borrowed money from the bank and secured the loan by granting to the bank a legal mortgage over the freehold of their home in England.

Which one of the following options best describes whether the lender would have been put on notice of possible undue influence in the circumstances?

A) The lender would not be put on enquiry of undue influence because there is no relationship of trust and confidence between the couple

B) The lender would not be put on enquiry of undue influence because the loan is for the joint benefit of the couple

C) The lender would be put on enquiry of undue influence because there is more than one borrower

D) The lender would be put on enquiry of undue influence because the couple are buying a property in a different country

E) The lender would not be put on enquiry of undue influence because the lender has taken the necessary steps to bring home the risk of the mortgage

A

B) The lender would not be put on enquiry of undue influence because the loan is for the joint benefit of the couple

(This is correct. The lender would not be put on enquiry of undue influence because the loan is for the joint benefit of the couple. In CIBC Mortgages plc v Pitt the House of Lords confirmed that a lender would not be put on notice that there is a risk of undue influence where a transaction is ostensibly for a couple’s joint benefit, as it is on the facts here)

321
Q

Last year, a business owner granted to a lender a legal mortgage over the business premises to secure a capital and interest repayment loan. The mortgage deed did not mention any power of sale for the lender. The business is declining and the owner has not made any mortgage payments for four months. Today, the owner received a letter from the lender stating that the lender intends to sell the property and recover the money due from the sale proceeds.

Which of the following statements best explains whether the lender has a right to immediately sell the property?

A) The lender can sell the property as the power of sale has arisen and has become exercisable on the facts

B) The lender cannot sell the business owner’s property as there is no express right for the lender to do so

C) The lender can sell the property as the legal date for redemption has passed

D) The lender cannot sell the business owner’s property without first complying with the Pre-Action Protocol for Possession Claims 2008

E):The lender cannot sell the property until three months pass after the letter warning of the sale has been received

A

A) The lender can sell the property as the power of sale has arisen and has become exercisable on the facts

(This is correct: if there is no express power of sale in a mortgage deed (as here) then the statutory provisions in LPA 1925 apply and s 101 will give the lender such a right in a legal mortgage.
The right has arisen as one instalment of capital became due as soon as one payment had been missed: Payne v Cardiff; and the right is exercisable as some interest has been in arrears for two months: LPA 1925, s 103(ii).
As the right has become exercisable due to the missed interest repayments, there is therefore no need for the lender to serve a 3 month written warning/notice of the sale. The legal date for redemption, which is the earliest date on which the borrower can redeem the mortgage, has nothing to do with the lender’s right to sell)

322
Q

A borrower and lender enter into a document which is described as a ‘mortgage deed.’ The document purports to grant a mortgage over the borrower’s registered legal freehold. The agreement is signed by both the borrower and lender, witnessed and then dated. The lender does not do anything further with the document.

Which of the following options best describes what kind of mortgage (if any) has been granted by the borrower?

A) The borrower has not granted a mortgage because the document has not been registered

B) The borrower has granted an equitable mortgage. Equity will recognise the ‘failed legal mortgage’ as a ‘contract to grant a legal mortgage’

C);The borrower has granted an equitable mortgage because it is a mortgage of an equitable interest in the land

D) The borrower has not granted a mortgage because the document does not comply with the statutory requirements of a deed

E) The borrower has granted a legal mortgage because the statutory requirements of a deed have been met

A

B) The borrower has granted an equitable mortgage. Equity will recognise the ‘failed legal mortgage’ as a ‘contract to grant a legal mortgage’

(This is correct. Although a valid deed has been created on the facts - the document complies with LP(MP)A 1989, s 1 - it has not been registered by the lender. No valid legal mortgage has therefore been created on the facts. Equity will, however, recognise this an equitable mortgage in the circumstances. The document complies with LP(MP)A 1989, s 2 and equity will therefore recognise it as a ‘contract to grant a legal mortgage’. An equitable mortgage does not need to be registered to be validly created)

323
Q

The owner of a registered freehold property, used as a venue for weddings and parties, grants a legal mortgage over the property in favour of a lender as security for a loan. The mortgage deed contains the following terms:

(i) the lender may use the property free of charge for their annual Christmas party until the end of the mortgage term; and

(ii) the lender has an option to purchase the freehold until the end of the mortgage term.

Which of the following statements is correct in respect of the validity of the mortgage terms?

A) Term (i) and term (ii) are both unenforceable terms and likely to be struck out by a court

B) Term (i) and term (ii) are both enforceable terms and likely to be upheld by a court

C) Term (i) is an enforceable collateral advantage but term (ii) is inconsistent with the right to redeem the mortgage

D) Term (i) and term (ii) are both unenforceable terms and likely to be rewritten by a court

E) Term (i) is an unenforceable collateral advantage but term (ii) is likely to be upheld by a court

A

C) Term (i) is an enforceable collateral advantage but term (ii) is inconsistent with the right to redeem the mortgage

(This is correct. Term (i) is a collateral advantage but as it expires at the end of the mortgage term and as long as it is not onerous or in the nature of a penalty, it will be upheld by the court as in the case of Biggs v Hoddinott.
Term (ii) gives the lender the option to purchase the freehold at any time during the term and is likely to be struck out as in the case of Samuel v Jarrah Timber and Wood Paving Corporation Limited)

324
Q

Five years ago, a business owner needed to borrow money to expand their business. The business owner granted a first legal mortgage over the registered freehold of their home to a bank as security for the loan. The ten year repayment mortgage was granted by deed, included an express power of sale and the legal date of redemption was set for one month after the mortgage was granted.

The mortgagor’s business has been struggling recently and they have been unable to pay the last three mortgage instalments.

Which of the following options represents the best advice to the lender in respect of enforcing the security?

A) The lender should take possession through self-help, rather than applying for a court order, and then sell the property

B) The lender should apply for a court order to obtain possession and then sell the property. The mortgagor will not be able to apply to have the possession order postponed

C) The lender should appoint a receiver and initiate a debt action against the mortgagor because the power of sale has not yet arisen

D) The lender should apply for a court order to obtain possession and then sell the property. However, the mortgagor may apply to have the possession order postponed

E) The lender is not entitled to enforce the security on the facts

A

D) The lender should apply for a court order to obtain possession and then sell the property. However, the mortgagor may apply to have the possession order postponed

(This is correct and is the best answer. It is advisable for the lender to apply for an order for possession, rather than exercising self-help and it is true that the mortgagor can apply to have the possession order postponed under section 36 Administration of Justice Act 1970. The other options are less correct because possession through self-help is rarely if ever advisable with regard to residential property, the Administration of Justice Act 1970 would apply here as the mortgaged property is a dwelling, and the facts indicate that the power of sale has arisen and is exercisable (in addition to the fact that it would not make sense to appoint a receiver here))

325
Q

A landlord was the registered freehold proprietor of two adjoining plots of land. Fifteen years ago the landlord made a valid contract with a tenant in which it agreed to grant a 20-year legal lease of the southern plot (commencing on the date of the contract). The contract stipulated that the tenant was to use the southern plot for a specified purpose.

Immediately the contract was made, the landlord and tenant appreciated that the tenant could only use the southern plot for the specified purpose if the tenant was able to park a car on the northern plot.

The landlord, who had retained possession of the northern plot, told the tenant that it could park a car on that plot. The tenant has parked a car on the northern plot for the past 15 years.

Which one of the following statements is a correct description of the tenant’s right to park on the northern plot?

A) The tenant can park their car by reason of an express equitable easement

B) The tenant can park their car by reason of a prescriptive easement

C) The tenant can park their car by reason of an implied legal easement

D) The tenant can park their car by reason of an express legal easement

E) The tenant can park their car by reason of an implied equitable easement

A

E) The tenant can park their car by reason of an implied equitable easement

(This is the correct answer. Where an estate is transferred or granted to a person for a particular purpose, and that purpose can only be realised if the transferee or grantee can exercise an easement over land retained by the transferor or grantor, that easement will be implied by reason of common intention: Wong v Beaumont [1965] and Donovan v Rana [2014] EWCA Civ 99.
Since the tenant can only use the southern plot for the specified purpose if it is able to park on the northern plot, an easement of parking will be implied.
Implied easements derive their legal or equitable status from the transaction into which they are implied. Since the tenant acquired an equitable lease (arising by reason of the specifically enforceable contract to grant a lease: R v Tower Hamlets LBC ex parte Von Goetz [1999] QB 1019), any easements implied into that lease would be equitable)

326
Q

A man and a woman are adjoining land owners. The man has told the woman that he plans to build a large extension at the back of its house. The woman is very concerned that this will block the sun to her garden and make it very dark.

Can the women claim an easement in respect of the man’s land that could stop the extension from being built?

A) The woman cannot claim an easement of light as there is no defined aperture in a garden

B) The woman can claim an easement of light because it is a recognised negative easement

C) The woman cannot claim and easement as there is no such thing as a right to a view

D) The woman cannot claim an easement of light as the court will not recognise new negative easements

E) The woman can claim an easement of light which the proposed extension would block

A

A) The woman cannot claim an easement of light as there is no defined aperture in a garden

(This is correct. A right to light is one of a few recognised negative easements - it is not a ‘new’ negative easement, which the court will not recognise.
However, there is no general right to light. A right to light is an easement that gives a landowner the right to receive light through defined apertures in buildings or on their land. There is not a defined aperture in the woman’s garden to through which the right to light could be claimed on the facts, the women is just trying to claim a general right)

327
Q

A farmer owns the freehold of a farm. Five years ago the farmer leased out the southern part of the farm to a tenant under a four year legal lease. The farmer retained the northern part of the farm to live in. During the lease, the farmer continued to use the drains which serve the northern part of the farm and run under the southern part of the farm leased to the tenant.

Last year, the four year lease expired. The farmer granted the tenant a new four year legal lease on exactly the same terms as the previous one. There was no reference to the right to use the drains.

Which of the following statements best explains whether the farmer has acquired an easement to use the drains?

A) The farmer has acquired an easement impliedly under s.62 Law of Property Act 1925

B) The farmer has acquired an easement impliedly by necessity

C) The farmer has not acquired an easement

D) The farmer has acquired an easement impliedly under the rule in Wheeldon v Burrows

E) The farmer has acquired an easement impliedly by common intention

A

C) The farmer has not acquired an easement

(This is correct. This is an attempted reservation. There is no mention of the right to use the drains in the lease, so it cannot be an express easement. The only methods of implied acquisition available for reservations are: necessity and common intention. Necessity will not apply - necessity is only applicable to rights of way where there is no other means of access to the land. The courts will only allow an easement of common intention in a reservation situation if there is no other possible interpretation of the facts (Peckham v Ellison; Re Webb’s Lease). Here, the farmer can use its retained land without the drainage right and had two opportunities to reserve this right (in the first and second leases) (Yeung v Patel) so the court is unlikely to imply this reservation)

328
Q

A landowner owns the freehold of a large property. The self-contained basement flat is rented to a tenant under a 3 year lease. In the lease, the tenant is granted a right to use the landowner’s garden shed to store bikes. The tenant’s bikes take up all of the space in the shed. As the bikes are expensive, the tenant secures the shed with a padlock, which the landowner does not have a key for.

Which of the following statements best explains why the right to store is not capable of being an easement?

A) The right is not capable of being an easement because the right is negative and the court is not prepared to recognise new negative easements

B) The right is not capable of being an easement because it does not touch and concern the dominant land

C) The right is not capable of being an easement because the servient land owner is not left with any reasonable use of the shed

D) The right is not capable of being an easement because permission must be sought by the dominant land owner to exercise the right

E) The right is not capable of being an easement because there is no diversity of ownership

A

C) The right is not capable of being an easement because the servient land owner is not left with any reasonable use of the shed

(This is correct. The right is not capable of being an easement because it grants exclusive possession to the dominant land owner. The tenant is using all of the available space in the shed so the landowner cannot make any reasonable use of it (Batchelor v Marlow [2003] 1 WLR 764). In addition, the landowner does not ‘retain possession or control’ of the shed (the test suggested by Moncrieff v Jamieson [2007] UKHL 42) as the tenant locks the shed, which the landowner does not have a key for.
In order for a right to be capable of being an easement it must satisfy the requirements established in Re Ellenborough Park [1955] EWCA Civ. If the right does not satisfy any of these requirements, it is not capable of being an easement and will be merely a personal permission (a licence) rather than a proprietary right in the land, which an easement it. Even if a right satisfies the requirements of an easement laid down in Re Ellenborough Park, the right can be defeated (i.e. will not be an easement) if it requires payment by the servient land owner, grants exclusive possession to the dominant land owner or requires permission from the servient land owner in order to be exercised. If a right is capable of being an easement then you then need to consider if it has been acquired (expressly or impliedly) as an easement on the facts)

329
Q

A freeholder owns the freehold of a large property and garden. 6 years ago the freeholder let the top floor flat to a tenant under a 5 year legal lease. After moving in, the tenant asked the freeholder to use part of the garden to grow vegetables and relax in. The freeholder agreed. Last year, the 5 year lease expired and a new 5 year legal lease was granted on exactly the same terms as the previous one. There was no reference to the right to use the garden.

Which of the below statements best explains whether the tenant has acquired an easement to use the garden?

A) The tenant has acquired an express legal easement under the new 5 year lease

B) The tenant has not acquired an easement because this is a reservation and the right to use the garden is not necessary to access the top floor flat

C) The tenant has acquired an easement impliedly under s.62 of the Law of Property Act 1925

D) The tenant has not acquired an easement because the top floor flat can be used for residential purposes without use of the garden

E) The tenant has acquired an easement impliedly under the rule in Wheeldon v Burrows (1879)

A

C) The tenant has acquired an easement impliedly under s.62 of the Law of Property Act 1925

(This is correct. the tenant has acquired an easement impliedly under s.62 of the Law of Property Act 1925. On the grant of the second 5 year lease, the informal permission to use the garden is upgraded to an easement and implied into this lease. The facts are similar to Wright v Macadam [1949] 2 KB 744, which demonstrates this mode of acquisition.
An easement can be acquired expressly or impliedly. You should consider first if there has been express acquisition – is the easement written down and does it comply with the necessary formalities? If not, consider if the circumstances mean the easement could have been acquired impliedly by one of the recognised modes. Certain modes of implied acquisition only apply if it is a grant, rather than a reservation, situation. It is therefore important that you understand not only the different modes of acquisition, but also the distinction between reservation and grant)

330
Q

A retired lawyers purchases the registered freehold of two adjoining semi-detached townhouses. The lawyer renovates the townhouses including creating a shared underground garage for parking. This makes the properties more convenient as the nearest parking is on the next street.

The lawyer decides to live in the larger townhouse and sells the smaller townhouse to an accountant. The transfer deed contained a right for the owner of the smaller townhouse to park a car in the shared garage and the following covenant:

“to contribute 50% of the costs of maintaining the shared garage ”

After several years the accountant sells the smaller townhouse to a friend. The friend uses the shared garage but refuses to contribute to its maintenance costs.

Which of the following best describes whether the lawyer can enforce the covenant against the friend?

A) The covenant cannot be enforced against the friend because there is no clear link between the benefit and burden

B) The covenant cannot be enforced against the friend because there is no element of choice as to whether to take the mutual benefit

C) The covenant can be enforced against the friend because the burden of the covenant will pass under the rule in Tulk v Moxhay

D) The covenant can be enforced against the friend because the burden of the covenant will pass under the doctrine of mutual benefit and burden

E) The covenant cannot be enforced against the friend because the covenant is positive

A

D) The covenant can be enforced against the friend because the burden of the covenant will pass under the doctrine of mutual benefit and burden

(This is correct. The covenant is likely to pass to the friend under the rule in Halsall v Brizell [1957] as there is a mutual burden (to contribute towards the maintenance costs) and benefit (use of the garage). As per Rhone v Stephens [1994], there is a correlation between the benefit and burden and the benefit and burden as conferred in the same transaction (Davies v Jones [2009]). There is a genuine choice whether to accept the benefit of the right to park (Thamesmead Town Ltd v Allotey (1998)) as the friend could park on the next street instead. This exception to the general rule that the burden will not pass at common law (Austerberry) can therefore be applied on the facts and used by the lawyer to enforce the covenant against the friend.
The rule in Tulk v Moxhay cannot be used to pass the burden of the covenant in equity as the covenant is positive)

331
Q

A man and woman are adjoining land owners. 5 years ago the man entered into a covenant with the woman not to keep any animals on his freehold land.

The women has granted a ten year legal lease of her land to a tenant. The man has now brought ten dogs onto his land, and their barking is keeping the tenant awake at night. The tenant wants to enforce the covenant against the man.

Which of the following options is the best advice to the tenant as to whether it can enforce the covenant against the man at common law?

A) The tenant will be able to enforce the covenant against the man providing the covenant is deemed to touch and concern the land

B) The tenant will not be able to enforce the covenant against the man as the tenant is not the freehold owner of the dominant land

C) The tenant will not be able to enforce the covenant against the man as the burden of covenants cannot pass at common law

D) The tenant will be able to enforce the covenant against the man because the burden will pass under the rule in Tulk v Moxhay

E) The tenant will be able to enforce the covenant against the man because express assignment has occurred

A

A) The tenant will be able to enforce the covenant against the man providing the covenant is deemed to touch and concern the land

(This is the correct answer. Provided that the covenant is deemed to touch and concern the land, this will satisfy the rules for an implied passing of the benefit at common law under P&A Swift Investments v Combined English Stores.
The other conditions of P&A Swift will be satisfied because if there is no express wording showing an intention for the parties to pass the benefit to successors, s78 Law of Property Act 1925 will imply this intention. Further, both the covenantee and their successor will also hold legal estates in land, as the woman owned the freehold and the tenant has a legal lease. Smith & Snipes Hall Farm v River Douglas Catchment Board confirms that different estates are acceptable)

332
Q

A man was the owner of two adjoining registered freehold properties, one called House 1 and one called House 2.

Three years ago, the man sold House 2 to a woman. The woman covenanted in the transfer deed with the intention of binding the land known as House 2 not to build any structure in the grounds of House 2 without the approval of the owner of House 1. The man’s solicitor did nothing to protect the covenant.

Two years ago, the woman sold House 2 to a new owner. The new owner has recently built a structure in the garden of House 2 without obtaining the man’s approval.

Which statement best explains why the burden of this covenant has failed to pass to the new owner in equity?

A) The burden of the covenant will not run because the original parties did not intend the burden to run

B) The burden of the covenant will not pass because the new owner had no notice of the covenant

C) The burden of the covenant will not pass because the dominant land and servient land are not sufficiently proximate

D) The burden of the covenant will not pass because the covenant does not touch and concern the dominant land

E) The burden of the covenant will not pass because the covenant is positive

A

B) The burden of the covenant will not pass because the new owner had no notice of the covenant

(This is correct. As House 2 is registered land, the man should have protected the covenant by the entry of a notice on the charges register of House 2 (s32 LRA 2002). This did not happen at the time the covenants were entered into. Therefore, the new owner, as a purchaser for valuable consideration of House 2, takes free of the unprotected covenant (s29(1) LRA 2002).
The other answers are incorrect because the covenant is actually overall negative with a positive aspect (following Powell v Hemsley) and it does touch and concern the dominant land since restricting building on your neighbour’s land will protect the view from the dominant land thereby preserving its value, quality etc. It is incorrect to assert that the dominant and servient land are not sufficiently proximate since House 1 and House 2 are adjoining properties. There is also express wording here that shows that the original parties (the man and woman) did intend the burden of this covenant to run with House 2, so it is incorrect to state that the original parties did not intend the burden to run)

333
Q

A landowner is the registered freeholder of a large detached house. The landowner decides to split the house into two semi-detached houses and sells one to a buyer.

The buyer covenants with the landowner “to repaint the window frames every five years using only colours approved by the landowner.”

Which of the following options best describes what type of covenant this is?

A) The covenant is a mixed covenant. It cannot be split and is overall negative

B) The covenant is outright positive

C) The covenant is a mixed covenant which can be split into two free-standing covenants; the first positive, the second negative

D) The covenant is outright negative

E) The covenant is a mixed covenant. It cannot be split and is overall positive

A

E) The covenant is a mixed covenant. It cannot be split and is overall positive

(This is correct. Applying the ‘hand-in-pocket’ test (Haywood v Brunswick Permanent Benefit Building Society [1881] 8 QBD 403), the obligation to paint the windows requires money and time to be spent. The part of the covenant requiring the buyer to ‘only use colours approved by the landowner’ is restrictive and therefore negative - this part of the covenant can be complied with by inaction. The negative element cannot, however, stand alone and therefore Powell v Hemsley [1909] 1 Ch. 680 applies and the covenant is seen as one obligation with a condition attached. The main obligation is positive and the covenant is therefore seen as an overall positive covenant)

334
Q

Your client is the registered freeholder of two adjacent plots of land, Plot 1 and 2. Five years ago, your client decided to keep Plot 1 but sold Plot 2 to a buyer. Both your client and the buyer signed the transfer deed which contained the following provision:

“the buyer covenants on behalf of itself and its successors in title not to use Plot 2 for any purpose other than as a private dwelling house.”

Last month your client sold Plot 1 to a new owner. Which of the following statements best explains how the benefit of the covenant has passed to the new owner in equity?

A) The benefit has passed by express annexation

B) The benefit has passed under a scheme of development

C) The benefit has passed by implied assignment

D) The benefit has passed by express assignment

E) The benefit has passed by statutory annexation

A

E) The benefit has passed by statutory annexation

(This is the best answer. Section 78(1) Law of Property Act 1925 serves to annex the benefit of the covenant, as long as the parties have not expressly excluded s78, which there is no indication of on the facts.
The other answers are incorrect because there is no evidence that your client assigned the benefit of the covenant to the new owner; the wording of the covenant relates to the burden rather than the benefit so there was no express annexation and there is no evidence of a scheme of development. Further, implied assignment is not a means of passing the benefit in equity, it is a means of passing the benefit at common law)

335
Q

The owner of a registered freehold property (‘the Property’) grants their neighbour, by signed writing, a right to share the use of part of their garden, until the neighbour’s own garden has been landscaped. The owner is now planning to sell the Property.

Which of the following statements best describes how the neighbour should protect the right against a purchaser of the Property?

A) The neighbour should substantively register the right as part of its creation

B) The neighbour should protect the right by entering a restriction on the proprietorship register of the Property in order to trigger overreaching

C) The neighbour does not need to take action to protect the right as they have an implied legal easement overriding interest

D) The neighbour does not need to take action to protect the right as they have an actual occupation overriding interest

E) The neighbour should protect the right by the entry of a notice on the charges register of the Property

A

E) The neighbour should protect the right by the entry of a notice on the charges register of the Property

The neighbour has an inherently equitable easement over registered land- it is an easement for an uncertain term, so can only take effect in equity (s 1(3) LPA 1925). The easement has been properly created in accordance with s 53(1)(a) LPA 1925 and does not need to be registered in order to be validly created.
To bind a purchaser for valuable consideration, this type of easement should be protected by entering a section 32 Land Registration Act 2002 notice on the charges register of the Property.

336
Q

You act for the buyer of a registered freehold property (the ‘Property’). You client tells you they believe the Property has the benefit of a right of way.

What will you be looking for when examining the official copies of the title to the Property to confirm if the Property does enjoy the benefit of this right?

A) An restriction protecting the interest in the proprietorship register

B) An entry relating to a restrictive covenant in the charges register

C) An entry relating to an easement in the property register

D) An entry relating to a restrictive covenant in the property register

E) An entry relating to an easement in the charges register

A

C) An entry relating to an easement in the property register

(This is correct. A right of way would be an easement. You are told the land has benefit of this right, rather than the burden. Interests that benefit a piece of registered land are found in the property register)

337
Q

You act for the buyer of a registered piece of land. The sellers are a man and a woman, though only the woman is registered as the proprietor on the official copies of title. The man tells your client, on inspection of the land, that both him and the woman equally contributed to the purchase price.

What do you need to ensure happens for your client to take the land free of any interest the man has in it?

A) You do not need to do anything as there are two sellers so your client will not be bound by any interest due to overreaching

B) You need to check the charges register of the official copies of title to see if there is an entry relating to the interest. If not your client, as a purchaser for value, will not be bound

C) You need to check the official copies to see if there is an entry in the proprietorship register relating to the interest. If not, you do not need to be concerned

D) You need to check the Land Charges Registry to see if there is an entry against the woman’s name in respect of the interest. If not, your client, as a purchaser for value, will not be bound

E) You need to ensure the purchase monies are paid to both the man and the woman so that overreaching occurs

F) You need to ensure the purchase monies are paid to both the man and the woman because your client has actual notice of the interest

A

E) You need to ensure the purchase monies are paid to both the man and the woman so that overreaching occurs

(This is correct. The facts indicate that the man has an equitable interest in a trust of the land that arose impliedly due to his contribution to the purchase price. Ideally, this would be protected by the entry of a restriction in the proprietorship register to trigger overreaching. It is not clear if this has been done. Regardless, you need to ensure the purchase monies are paid to both the man and the woman otherwise your client will be bound by the equitable interest if the man is in ‘actual occupation’ of the land, which the facts indicate he might be)

338
Q

Your client is gifted a piece of registered land from their parent, which they take occupation of. The next door neighbour tells your client that it has an option to purchase the land, which is in writing and signed by both the parent and the neighbour, that it now wishes to exercise. There is no entry relating to the option in the charges register of the official copy of title.

Is your client bound by the option to purchase?

A) No, your client is not bound because the interest has not been protected

B) Yes, your client is bound under the basic rule of priority

C) Yes, your client is bound because the neighbour has an overriding interest in the land

D) No, your client is not bound because the neighbour has no valid interest in the land

E) Yes, your client is bound because overreaching has not occurred on the facts

A

B) Yes, your client is bound under the basic rule of priority

(This is correct. Your client is not a purchaser for value, they are a donee - they were gifted the land. Therefore, they will be bound by the option to purchase, which is a form of estate contract.
The estate contract is an equitable interest in the land. It has been validly created as it complies with LP(MP)A 1989, s 2 - it is in writing and signed by both parties (the neighbour and the parent) and there is nothing to indicate that all the terms are not contained. An estate contract over registered land should be protected by a notice in the charges register of the burdened land (LRA 2002, s 32) otherwise it will not bind a purchaser for value. However, your client is not a purchaser for value on the fact so will still be bound (LRA 2002, s 28) under the normal rules of priority)

339
Q

A buyer of a registered house has come to you for advice. They purchased the house last month. Last week, a person claimed they had an interest in the house because they contributed to the purchase price. The person had been out of the country on business for the last few months, so had not realised the house had been sold. Their partner was the sole legal owner who has now disappeared with the purchase monies. Your client says they did notice belongings of someone else when they looked round the house, but the sole legal owner assured them no-one else lived there.

Is your client bound by the interest of the person?

A) No, your client will not be bound because the person has failed to disclose their interest upon reasonable enquiry

B) No, your client will not be bound because the person’s interest is not obvious upon reasonable inspection of the land

C) Yes, your client will be bound because the person’s interest has not been overreached

D) Yes, your client will be bound because the person is in actual occupation of the land

E) No, your client will not be bound because the person is not in actual occupation of the land

A

D) Yes, your client will be bound because the person is in actual occupation of the land

(This is correct. The person has an equitable interest in a trust of land that arose impliedly by virtue of their contribution to the purchase price (LPA 1925, s 53(2)). This interest has not been overreached because the monies were paid to a ‘sole legal owner.’ The persons interest therefore remains in the land. It will be binding on your client because they will be deemed to be in ‘actual occupation’. Although clearly temporaily absence, their belongings remained and there was a clear intention to return)

340
Q

A proprietor of unregistered land has today granted a lender a legal mortgage by deed. There is already a charge by way of legal mortgage over the land.

Which of the following statements best describes the legal position in respect of how this interest should be protected?

A) The interest does not need protecting as will be binding under the rule ‘legal interests bind the world’

B) The interest must be substantively registered to bind future purchasers for valuable consideration

C) The interest must be protected by a land charge to bind future purchasers for valuable consideration

D) The interest must be protected by a notice in the charges register of the title to bind future purchasers for valuable consideration

E) The interest does not need protecting as will be binding as an overriding interest

A

C) The interest must be protected by a land charge to bind future purchasers for valuable consideration

(This is correct. You are told on the facts the land is unregistered. This is a second legal mortgage - a puisne mortgage - and must be protected by a Class C(i) land charge to bind a purchaser for valuable consideration of any interest in the land. See s2(4)(i) and s4(5) Land Charges Act 1972.
If this were a first legal mortgage over unregistered land, it would be a trigger for first registration of the land (s 4 LRA 2002).
If this second legal mortgage were granted over registered land, it would need to be substantially registered in order to take effect as a legal interest)

341
Q

A solicitor is investigating the title to a piece of unregistered land.

Which of the following is the best answer to determine what matters will need to be investigated on behalf of the client?

A) The solicitor will need to review a search carried out of the Land Charges Register, and investigate the possibility of overriding interests that may affect the property

B) The solicitor will need to review the Land Registry title to the property and investigate the possibility of interests that are subject to the doctrine of notice.

C) The solicitor will need to review a search carried out of the Land Charges Register, and investigate the possibility of interests that are subject to the doctrine of notice

D) The solicitor will need to review the Land Registry title to the property and investigate the possibility of overriding interests that may affect the property.

E) The solicitor will need to determine whether there are any legal interests at the property as those that are registered at the Land Registry may still affect the property

A

C) The solicitor will need to review a search carried out of the Land Charges Register, and investigate the possibility of interests that are subject to the doctrine of notice

(This is the correct answer. Title to unregistered land has no register at the Land Registry to review. As such, when investigating unregistered land, solicitors must investigate the possibility of any Land Charges that are registered on the Land Charges Register and will bind the land, but also consider whether there are any other interests (predominantly pre-1926 interests or beneficial interests under a trust that have not been overreached) that could also bind the property subject to the doctrine of notice)

342
Q

The sole legal owner of an unregistered freehold property granted a second legal mortgage to a lender bank. Before loaning the money, the lender bank instructed their surveyor to inspect and value the property. The surveyor spoke to the owner’s partner, who lives at the property, and has an interest in it by way of an express trust of the land, which they informed the surveyor of. The surveyor failed to pass this information onto the lender bank. The lender bank proceeded to loan the money to the sole legal owner, who is now in default of their repayments.

Is the bank bound by the partner’s interest and therefore unable to enforce the mortgage and recover possession of the property?

A) The bank is not bound by the interest because it was overreached on payment of the loan monies

B) The bank is bound by the interest because it had actual notice of it

C) The bank is not bound by the interest because it is a purchaser for value and the interest has not been protected

D) The bank is bound by the interest because it had constructive notice of it

E) The bank is bound by the interest because it had imputed notice of it

A

E) The bank is bound by the interest because it had imputed notice of it

(This is correct. The surveyor has actual notice, and the surveyor is the bank’s agent. The bank will be bound by the actual or constructive notice of their agents, which is imputed notice. Overreaching has not occurred because the bank only paid the loan monies to the sole legal owner. It is therefore bound by the partner’s equitable interest because it is not equity’s darling)

343
Q

A couple are the legal owners of some unregistered development land. A man has a beneficial interest under an implied trust in the land. Last week, the man heard a rumour that the couple were in the process of selling the land to your client. The man wrote to your client informing it of the trust interest. Your client read the letter but ignored it and completed the purchase yesterday.

If your client bound by the man’s trust interest?

A) Your client is bound by the interest as it remains in the land and your client had constructive notice of it

B) Your client is not bound by the interest because it has been overreached

C) Your client is bound by the interest as it remains in the land and binding on your client as the man is in actual occupation

D) Your client is bound by the interest because it remains in the land and your client had actual notice of it

E) Your client is not bound by the interest as it should have been protected by a Land Charge against the names of the couple and as this has not been done, the interest is now unenforceable

A

B) Your client is not bound by the interest because it has been overreached

(This is correct. Your client paid the money to two trustees, thus overreaching it and moving it into the sale proceeds. The couple, as trustees must pay the man the sum which represents the man’s share of the land)

344
Q

Your client has the benefit of a right of way for 20 years created by valid deed over adjoining land. The adjoining land is unregistered and the new owner has blocked your client from using the right of way.

Will the neighbour be bound by your client’s interest?

A) The right of way will be binding if it has been protected by land charge registered against the name of the grantor

B) The right of way will be binding if it has been protected by land charge registered against the name of the new owner

C) The right of way will be binding as an implied legal easement overriding interest

D) The right of way will be binding if it has been protected by land charge registered against the land

E) The right of way will be binding under the rule ‘legal interests bind the world’

A

E) The right of way will be binding under the rule ‘legal interests bind the world’

(This is correct. A legal interest over unregistered land will be binding under the principle ‘legal interests bind the world’. We know the right of way is a legal easement because it is granted for term equivalent to a freehold or leasehold estate (20 years - a certain term) and is granted by way of valid deed.
If the easement was for an uncertain term or was granted by way of a failed deed, it could be equitable. In this situation, it would need to be protected by a land charge against the name of the grantor)

345
Q

Five friends (all aged over 18) purchase a restaurant with the intention of running it as a business. The friends contribute equally to the purchase price and the property is conveyed to all of the friends in the same document. There is no express declaration of trust and there are no words in the transfer deed as to how the property is to be held.

Which of the following statements best explains how the friends hold the property in equity?

A) The first four of the five friends named in the transfer deed will hold the property as joint tenants

B) Given the nature of the property and purpose of the purchase, there is a presumption that the friends hold the property as tenants in common

C) There is a presumption that the friends hold the property as joint tenants as there are no words of severance

D) As there is no express declaration of trust, equity presumes that the friends hold the property as tenants in common

E) The friends can only hold the property as tenants in common as only unity of possession is present in the circumstances

A

B) Given the nature of the property and purpose of the purchase, there is a presumption that the friends hold the property as tenants in common

(This answer is correct as it states that the friends wish to use the property for business purposes. Equity is therefore likely to presume a tenancy in common. See cases such as [Lake v Craddock] [Stack v Dowden] and [Jones v Kernott] as to equitable presumptions commercial situations where there is no express declaration of trust or words of severance)

346
Q

A couple own a house as joint tenants in law and equity. The relationship falls apart and both parties instruct solicitors. The man’s solicitor issues divorce proceedings and makes an application to the court for the house to be sold and the proceeds split evenly.

Which of the following statements best describes the severance of the joint tenancy in equity?

A) The man’s interest only has been severed in equity by the issuing of the proceedings

B) The issuing of proceedings may amount to severance of the equitable joint tenancy by written notice

C) The issuing of proceedings amounts to a ‘unilateral act operating on a share’ and the joint tenancy has been severed

D) The joint tenancy has been severed by mutual agreement

E) The instruction of solicitors indicates a common mutual intention to sever the joint tenancy (i.e. severance by mutual conduct)

A

B) The issuing of proceedings may amount to severance of the equitable joint tenancy by written notice

(This is correct. Whilst not definite, following (Re Drapers Conveyance) it is likely that the issuing of proceedings will amount to severance by written notice under s.36(2) of the Law of Property Act 1925.
Although it may be possible to infer mutual agreement from the issuing of and engagement in legal proceedings (see Hunter v Babbage), here we do not know enough about the other partner’s position to establish mutual agreement, implied or otherwise, to sever. Further, whilst the instruction of solicitors may a persuasive factor in establishing whether there is severance by mutual conduct, it is unlikely to be determining factor in its own right.
If severance occurs, both parties will become tenants in common, it is not possible to have a single joint tenant)

347
Q

Four people purchase a registered freehold property for use as office premises to run their architect firm. The four people hold the property as joint tenants. The equitable interest of one of the people (the ‘partner’) is subsequently mortgaged with a local bank. On examining the related paperwork, the partner notices that the mortgage contract has not been signed by the bank.

Which of the following statements best represents the legal position in respect of the partner’s equitable interest?

A) The partner’s equitable interest automatically vests in the bank

B) All of the four people now hold an equitable interest as tenants in common, 25% each

C) The partner now holds a 50% interest in equity as a tenant in common

D) The partner’s equitable interest has been severed by partial alienation

E) The partner’s equitable interest has not been severed as the contract has not been signed by both parties

A

D) The partner’s equitable interest has been severed by partial alienation

(This is correct. A mortgage of an equitable interest has been created on the facts. The necessary formalities for this mortgage are as per s. 53(1)(c) Law of Property Act 1925, which have been complied with on the facts (it does not matter that the lender has not signed the mortgage, it only needs to be signed by the grantor - the partner). Such a mortgage has the effect of severing the interest of the mortgagor in equity - it is an ‘act operating on your own share’, a form of partial alienation (First National Security v Hegarty [1985]))

348
Q

Four siblings (A, B, C, D) are the legal and equitable owners of a registered freehold property. When the property was purchased, they made an express declaration to hold the property ‘on trust for themselves as equitable joint tenants’. Last month, C was declared bankrupt and a trustee in bankruptcy was appointed. Last week, A died. In A’s will, A left everything to E.

Which of the following options correctly describes the equitable ownership of property?

A) E holds 25% as a tenant in common. B, D and C hold 75% as equitable joint tenants

B) B, D, C and E each hold 25% as tenants in common

C) B and D hold 75% as equitable joint tenants. C holds 25% as a tenant in common

D) B and D hold 50% as equitable joint tenants. C holds 50% as a tenant in common

E) B, C and D hold 100% as equitable joint tenants

A

C) B and D hold 75% as equitable joint tenants. C holds 25% as a tenant in common

(This is correct. A died as an equitable joint tenant, and therefore the right of survivorship operated and A’s interest in the property passed to the remaining joint tenants, B and D.
C’s interest had already been severed at this point. When C was declared bankrupt this automatically severed C’s interest, which vested in C’s trustee in bankruptcy. On severance, C got an equal share, which at the time was 25% (Goodman v Gallant). The subsequent death of A had no effect on this)

349
Q

Five friends (A, B, C, D and E) purchased property together as beneficial joint tenants. At the time of the purchase C was 17 years old. Six months after the purchase, D died in a car accident and by D’s will, F inherited all of D’s property.

Who currently holds the legal title?

A) A, B and E

B) A, B, C and F

C) A, B, C and E

D) A, B, E and F

E) A, B and C

A

A) A, B and E

(This is correct. Originally, the legal owners were A, B, D and E as joint tenants. C could not hold legal title as legal owners must be aged 18 or over. On D’s death, the rule of survivorship applies and the joint tenancy continues to be held by the survivors, A, B and E)

350
Q

Four journalists are the legal and equitable joint tenants of a registered freehold property. One of the journalists is seconded to Ottawa. Six months into the secondment, the journalist writes an unsigned letter to the other three journalists which is sent to the freehold property. The letter says:

‘I am thinking I want my share in the property sold in a year’s time.’

Which of the following options best describes whether severance has occurred on the facts?

A) Severance has not occurred because the notice does not show a desire to sever with immediate effect

B) Severance has not occurred because the letter has not been correctly served on the equitable joint tenants

C) Severance has occurred by an partial alienation

D) Severance has not occurred because the letter is unsigned

E) Severance has occurred by notice in writing

A

A) Severance has not occurred because the notice does not show a desire to sever with immediate effect

(This is correct. The notice in writing must demonstrate an unequivocal and immediate intention to sever - Harris v Goddard [1983] 1 WLR 1203. Here the desire to sever does not show an intention to bring about the result immediately, it says for the property to be sold in a year’s time. The letter would otherwise comply with the requirements of a ‘notice in writing’ - it is in writing, addressed to the other equitable joint tenants and correctly served on them at their place of abode. It does not matter that the letter is unsigned (Re Draper’s Conveyance [1969]))

351
Q

Three colleagues (E, F, and G) run a business together and are the registered freehold owners of their commercial premises which they hold as beneficial joint tenants.

Two weeks ago, F told G and H that F would like to retire in six months’ time. All three discussed what would happen to the premises when F retired. F wanted G and H to buy out F’s share for £230,000. G and H felt that this might be a good idea in principle, but did not want to commit itself and wanted to think about other ways of dealing with the situation. Last night F died. Nothing further had been done about buying F’s share. In their will, F has left all of their property to their partner, H.

Which of the following statements best describes the position regarding ownership of the property?

A) E H and G hold the legal and equitable titles as joint tenants

B) E and G hold the legal title and equitable titles as joint tenants

C) E H and G hold the legal title as joint tenants; E and G hold 2/3 equitable title as joint tenants and H holds 1/3 as tenant in common

D) E and G hold the legal title as joint tenants; E and G hold 2/3 equitable title as joint tenants and H holds 1/3 as tenant in common

E) E and G hold the legal title as joint tenants and H holds the equitable title

A

B) E and G hold the legal title and equitable titles as joint tenants

(This is correct. The legal joint tenancy cannot be severed (Law of Property Act 1925, s 36(2), and E F and H have not reached a mutual agreement to sever the equitable joint tenancy: Burgess v Rawnsley as they had not reached the point where all joint tenants regarded one person’s share as separate. Their further deliberations need not necessarily involve severance. As F died as a legal and equitable joint tenant, survivorship applies meaning that the interest accrues to the survivors, E and G)

352
Q

Two trustees (A and B) hold property on trust for three beneficiaries in equal shares (C, D, and E), who live at the property. The property is mortgaged to a bank and the payments are up to date. E has just turned 18, and wants the property to be sold to use the share of the proceeds to take a gap year and travel the world. C and D are trainee doctors working at the local hospital and do not wish the property to be sold.

Which of the following statements best describes whether E’s application for an order for sale is likely to be successful?

A) E’s application for an order for sale is likely to be granted because the purpose for which the property is held, as a joint home, is no longer capable of continuing

B) E’s application for an order for sale is likely to be granted as the property is mortgaged and the court gives greatest weight in practice to the interests of the secured creditor.

C) E’s application for an order for sale is likely to be granted because the combined interests of E and the bank outweigh those of C and D

D) E’s application for an order for sale is likely to be refused by the court because C and D oppose the sale and the combined value of their interests is greater than E’s

E) E’s application for an order for sale is likely to be refused because the welfare of a minor who occupies the property will outweigh all other considerations

A

D) E’s application for an order for sale is likely to be refused by the court because C and D oppose the sale and the combined value of their interests is greater than E’s

(This is correct. TLATA 1996, s 15(3) allows the court to take into account the circumstances and wishes of the beneficiaries aged 18 and over and give effect to the wishes of the majority by value. The interests of the bank will not weigh heavily as the payments are not in arrears and the bank will continue to be paid monthly instalments)

353
Q

A freehold owner of a shop grants a lease of the shop to a company. The document granting the lease is headed “deed”, signed by the freehold owner and the company, witnessed and dated. The duration of the lease is specified as “until the freeholder requires the shop for redevelopment.” The rent is specified as £100,000 per annum, payable in monthly instalments. The company goes into possession and pays the rent every month.

Which of the following options best describes what type of lease (if any) the company has?

A) The company do not have a valid lease

B) The company have a monthly periodic tenancy

C) The company has a fixed term lease

D) The company has an equitable lease

E) The company has an annual periodic tenancy

A

E) The company has an annual periodic tenancy

(Correct answer: The fixed term is void for uncertainty, but because they go into possession and pay rent calculated on an annual basis, they have an implied annual periodic tenancy. The other options are incorrect. The periods (term) of an implied periodic tenancy depend on how the rent is calculated, not how it’s paid. The fixed term is void for uncertainty, so it cannot create a valid legal or equitable fixed term lease)

354
Q

A freehold owner of an indoor market gives a jewellery maker a written “licence” to sell jewellery in a space in the market on weekdays from 9am to 5pm, for a weekly payment. There is no fixed term. The terms of the “licence” give the freeholder the right to choose the space each week, and also impose detailed restrictions on what the jewellery maker can sell and the layout and use of the space. The freeholder usually gives the jewellery maker the same weekly space, but on two occasions has moved them to another space.

Which of the following options best describes whether the jewellery maker(the occupier)is a tenant or licensee?

A) The occupier is a tenant because the relocation clause would be thrown out by the court as a sham

B) The occupier is a licensee because the agreement is called a “licence”

C) The occupier is a licensee because there is no fixed term

D) The occupier is a tenant because it is a business context, not a residential context

E) The occupier is a licensee because it does not have exclusive possession

A

E) The occupier is a licensee because it does not have exclusive possession

(Correct answer. Caselaw shows that a moving/relocation clause, and control by the landlord, are likely to preclude exclusive possession in a business context, so the best answer is that they are likely to have a licence and be a licensee.
The other options are incorrect. It is not correct to advise that it is a licence just because it’s called a licence, or to advise that they are likely to have a lease just because it’s a business context. The lack of a fixed term does not make it a licence, because it could still be an implied periodic tenancy due to the weekly payment. There are no facts given that show the court will ignore the terms as a sham, the reality is that the relocation clause is exercised)

355
Q

The freehold owner of a block of flats orally allows a person to occupy one of the flats at the market rent of £800 per month. The freehold owner orally reserves the right to enter the flat once a month to carry out any repairs that are needed. Nothing is put in writing and no fixed term is agreed.

Which of the following options best describes what type of lease (if any) the occupier has?

A) The occupier has a legal periodic tenancy

B) The occupier only has a licence because there is no certainty of term

C) The occupier only has a licence because of the lack of formalities

D) The occupier has an equitable periodic tenancy

E) The occupier only has a licence because the freehold owner’s right of entry will defeat exclusive possession

A

A) The occupier has a legal periodic tenancy

(Correct. A monthly periodic tenancy will be implied by the common law from the monthly rent and the LPA 1925 s.54 (2) short lease exception applies, which means no formalities are required to create the lease.
The other options are incorrect. A monthly periodic tenancy is implied by the common law, not equity, and doesn’t need formalities (if market rent) or a fixed term. The limited /restricted reservation of access indicates exclusive possession, and therefore a lease)

356
Q

Two years ago on June 7th, a landlord purported to grant a five-year lease of a house to a man and woman by deed. The rent had been advertised at £1,000 per month. The man and woman signed identical documents on the same day, each stating that the rent was £1,000 per month, although the man signed at 11am and the woman at 2pm on 6th June.

Which of the following option best describe if this is a joint lease or not?

A) This is not a joint lease because the man and woman do not have unity of title

B) This is not a joint lease because the man and woman do not have unity of time

C) This is a joint lease because the man and woman have the four unities

D) This is not a joint lease because man and woman do not have unity of possession

E) This is not a joint lease because the man and woman do not have unity of interest

A

C) This is a joint lease because the man and woman have the four unities

(This is correct. There is no indication that the man and woman do not have unity of possession. They have unity of interest as they both have a five year lease on the same terms with a joint obligation to pay £1000 rent; they have unity of title as they sign identical documents and they have unity of time as the interest of both started on June 7th. It is irrelevant that they signed the documents at different times.
On the facts the the separate licence documents are likely to be read interdependently as per Antoniades v Villiers (1990))

357
Q

An owner of a registered freehold house comprising a living space, kitchen, bathroom and bedroom, enters into two separate agreements with two occupiers which purport to grant them a two year licence of the house.

In which of the following scenarios are the occupiers most likely to have entered into a joint lease agreement with the owner of the registered freehold?

A) The owner and occupiers are related and they pay licence fees which are significantly lower than the current market rate

B) The occupiers pay the same licence fee and their agreements started on the same day

C) The owner and the occupiers are employer and employees respectively and the occupiers are required to live in the house to perform their employment roles

D) The occupiers pay different licence fees and their agreements started on different days

E) The owner has a right of access to provide cleaning services to the occupiers every other day

A

B) The occupiers pay the same licence fee and their agreements started on the same day

(This is correct. On these facts, the occupiers appear to have the four unities (possession, interest, title and time) as required by AG Securities v Vaughan (1990). On the facts, the separate licence agreements are likely to be read interdependently as per Antoniades v Villiers (1990). If the agreements are exactly the same, placing each occupier under the same terms, they will be considered to be ‘artificially separate’ as per Antoniades and this will not defeat the 4 unities from being present. The occupiers would be joint tenants of the property pursuant to a lease.
If the occupier are employer and employee and the occupiers are required to live in the house to perfrom their employment duties, this would defeat a lease as it would be a service occupanies)

358
Q

An occupier entered into a four year agreement by deed with the owner of a registered freehold building to rent a first floor flat. The agreement contains a clause in which the owner of the registered freehold agrees to clean the flat once every two days. The occupier moved into the premises 2 years ago and the freehold owner has not cleaned the premises once.

Which of the following statements best describes whether the occupier has a lease or licence?

A) The occupier has a licence because the deed has not been substantively registered

B) The occupier has a lease because the cleaning clause is a sham clause

C) The occupier has a lease because the agreement was created by deed

D) The occupier has a licence because there is no certain term

E) The occupier has a licence because the cleaning clause defeats exclusive possession

A

B) The occupier has a lease because the cleaning clause is a sham clause

(This is correct. Sham clauses are those which owners of land may insert into a lease to make it look like a licence (defeating the requirement of exclusive possession) and to deny the occupier certain statutory rights which attach to a lease. The cleaning clause, if genuine, would defeat exclusive possession as it means the tenant is not able to exclude the landlord from the premises ‘once every 2 days’. Whether the cleaning clause is a sham or not depends on whether it is being exercised by the owner of the registered freehold, amongst other factors.
On the facts, the cleaning clause hasn’t been exercised once in 2 years. The court would therefore ignore the clause. On the facts there is a certain term and nothing else to suggest the occupier doesn’t have exclusive possession. The agreement has been created by deed, which are the correct formalities for a 4 year lease. The occupier would therefore have a lease)

359
Q

You act for the tenant of a lease. The lease contains a covenant that the tenant will ‘keep the premises in a good state of repair and condition’.

Your client asks your to explain their repair obligation in respect of their premises. What is your advice?

A) The premises must be kept in the condition required by the landlord

B) The premises must be maintained in the condition they are at the start of the lease

C) The premises must be renewed if required

D) The premises must be kept in the condition in which they would be kept by a reasonably minded owner

E) The premises must be put into repair if they are not already and then kept in the condition in which they would be kept by a reasonably minded owner

A

E) The premises must be put into repair if they are not already and then kept in the condition in which they would be kept by a reasonably minded owner

(This is correct. A covenant to ‘keep’ premises in repair can be very onerous if the premises are not in a good state of repair at the start of the lease, as a covenant to keep the premises in repair also entails an obligation to put them in repair first. Once in repair the Court of Appeal in Proudfoot v Hart (1890) made clear that the premises should be kept in the condition in which they would be kept by a reasonably minded owner, having regard to the character, type and age of the premises)

360
Q

A landlord was the registered freehold owner of a shop. 7 years ago, the landlord granted a legal lease of the shop to a clothes retailer for 20 years. The clothes retailer covenanted to redecorate the property every year and not to assign the lease without the landlord’s consent. Four years ago the landlord sold the freehold of the shop to an investment company. Two years ago, the clothes retailer assigned the lease to a shoe shop without the investment company’s consent and without entering into an authorised guarantee agreement. Last year the shoe shop did not redecorate the shop as required under the terms of the lease.

Which statement best explains whether the investment company can sue the clothes retailer, the original tenant, for the breach of covenant by the shoes shop?*

A) The investment company cannot sue the clothes retailer because it does not have the benefit of the redecoration covenant

B) The investment company can sue the clothes retailer because the clothes retailer is liable under original tenant liability

C) The investment company can sue the clothes retailer because it assigned the lease to the shoe shop without the investment company’s consent

D) The investment company cannot sue the clothes retailer because it did not provide an authorised guarantee agreement

E) The investment company cannot sue the clothes retailer because the breach does not involve a fixed charge

A

C) The investment company can sue the clothes retailer because it assigned the lease to the shoe shop without the investment company’s consent

(This is correct. There was an unauthorised assignment two years ago and the clothes shop would not have been automatically released from liability under s 5 LTCA 1995 (see s 11 LTCA 1995). The investment company can therefore still sue the clothes retailer for the shoe shop’s breach even though the clothes retailer did not provide an authorised guarantee agreement.
The other answers are incorrect because the clothes retailer is still liable for the shoe shop’s breach even though it did not provide an authorised guarantee agreement (see above) and the breach does not need to involve a fixed charge for the clothes retailer to be liable (see s 5 LTCA 1995 and s 11 LTCA 1995). It is incorrect to argue the investment company does not have the benefit of the redecoration covenant, because the benefit will have passed to it under s 3 LTCA 1995 as the covenant is not expressed to be personal. Also, original tenant liablity only applies to old leases and this is a new lease on the facts)

361
Q

10 years ago a landlord granted a tenant a 15 year legal lease. The lease does not contain a covenant in respect of alterations.

The tenant would like to change the internal layout of the premises.

Which of the following options best explains if the tenant can undertake these alterations?

A) The tenant must obtain the landlord’s consent to the alterations

B) The tenant can undertake the alterations if it considers they are ‘improvements’

C) The tenant is not permitted to make the alteration

D) The tenant is permitted to make the alterations providing they do not devalue the premises

E) The tenant must obtain the landlord’s consent to the alterations, which cannot be unreasonably withheld

A

D) The tenant is permitted to make the alterations providing they do not devalue the premises

(This is correct. Unless the lease stipulates otherwise, which it does not on the facts, the tenant is free to carry out any alterations to the premises. This is only subject to the legal doctrine of waste, which prevents alterations which would devalue the premises)

362
Q

A landlord granted a 20 year legal lease to a tenant 6 years ago. The landlord covenanted in the lease to keep the exterior of the property in good and substantial repair. The tenant legally assigned the lease to an assignee 4 years ago. The landlord sold the reversion to a new owner 2 years ago.

The roof of the property now needs urgent repairs but the new owner is refusing to carry out the repairs as it is experiencing financial difficulties.

Which of the following statements best explains who has the benefit and who has the burden of the landlord’s covenants?

A) The assignee only has the benefit of the covenants; the new owner only has the burden of the covenants

B) The tenant and the assignee have the benefit of the covenants; the landlord and the new owner have the burden of the covenants

C) The assignee only has the benefit of the covenants; the landlord only has the burden of the covenants.

D) The assignee only has the benefit of the covenants; the landlord and new owner have the burden of the covenants

E) The tenant and the assignee have the benefit of the covenants; the new owner only has the burden of the covenants

A

D) The assignee only has the benefit of the covenants; the landlord and new owner have the burden of the covenants

(The benefit of the covenants passes to the assignee under s 3 Landlord and Tenant (Covenants) Act 1995 and the tenant ceases to be entitled to the benefit under s 5(2)(b) Landlord and Tenant (Covenants) Act 1995.
The burden of the landlord’s covenants passes to the new owner under s 3 Landlord and Tenant (Covenants) Act 1995. The burden also remains with the landlord until it applies to be released from the burden of the landlord’s covenants as per s 6(2)(a) Landlord and Tenant (Covenants) Act 1995)

363
Q

A lease contains a covenant in which the tenant covenants ‘not to assign the whole of the premises without the landlord’s consent’.

Two weeks ago he tenant made a written request to the landlord to assign the whole of the premises to a company. The company runs a business which competes with the business run by the landlord in adjoining premises.

Last week, the landlord called the tenant to refuse the application.

Which of the following options best explains if the landlord has breached any implied statutory duty to the tenant?

A) The landlord is under a duty to respond in a reasonable time, which it has not done in the circumstances

B) The landlord is under a duty to give written consent / refusal, which it has not done in the circumstances

C) The landlord is under a duty to act reasonably, which it has not done in the circumstances

D) The landlord is under no statutory duty, it has not breached any implied duty in the circumstances

E) The landlord is under a duty to act reasonably, which it has done in the circumstances

A

B) The landlord is under a duty to give written consent / refusal, which it has not done in the circumstances

(This is correct. On the facts, the landlord is in breach of this statutory duty. Although it has responded within a reasonable time (within 28 days) and its refusal is arguably reasonable, it has not given written reasons.
The covenant is qualified. LTA 1927, s 19(1)(a) applies to this covenant, imposing an obligation on the landlord to act reasonably and not unreasonably withhold or delay its consent. The landlord has refused the request to assign on the basis that the assignee, the company, would compete with the Landlord’s business. This has been held to be a reasonable reason to refuse consent.
LTA 1988, s 1 also applies to this covenant (and generally all fully qualified covenants). It provides the landlord must give written consent within a reasonable time (unless it is reasonable to refuse) and written reasons must be provided)

364
Q

A landlord grants a 30 year legal lease to a law firm in 1993. The law firm legally assigns the lease to a bank in 1996. The bank legally assigns the lease to a design company in 2001. The design company has failed to pay the rent for the past 6 months. Both the law firm and the design company are experiencing financial difficulties.

Which of the following statements best explains bank’s liability regarding the unpaid rent?

A) The bank may be pursued by the landlord for the unpaid rent as there is privity of estate between them

B) The bank may be pursued by the landlord for the unpaid rent as there is privity of contract between them

C) The bank may not be pursued by the landlord for the unpaid rent as the design company is the only party liable to pay the rent

D) The bank may not be pursued by the landlord for the unpaid rent as it has been automatically released from liability

E) The bank may be pursued by the landlord for the unpaid rent if there is a direct covenant between them

A

E) The bank may be pursued by the landlord for the unpaid rent if there is a direct covenant between them

(As the bank is an intermediate tenant, there is no privity of estate between the landlord and the bank as the bank is no longer the tenant. There is no privity of contract between the landlord and the bank unless they entered into a direct covenant, pursuant to which the landlord could sue the bank for a breach of covenant by the bank’s successor in title.
This is an old lease on the facts, so a direct covenant is in the only way in which the intermediate tenant could be liabile. If there is no direct covenant, the bank will not be liable)

365
Q

A landlord and tenant enter into a legal lease of 2 years. The lease contains the following covenant:

“The Tenant must not to alter the premises”.

The tenant would like to make some internal non-structural alterations.

Which of the following options best explains whether the landlord will have to consent to these works and the statutory duty (if any) it is under?

A) The landlord can refuse consent or impose any condition on any consent in the circumstances

B) The landlord must respond to the request with written reasons within a reasonable time

C) The landlord can refuse consent but if it does consent it cannot demand payment for it unless the change involves a change to the structure

D) The landlord cannot unreasonably withhold or delay its consent

E) The landlord cannot unreasonably withhold its consent if the tenant considers the change is an ‘improvement’

A

A) The landlord can refuse consent or impose any condition on any consent in the circumstances

(This is correct. The covenant is an absolute covenant against alterations. This means the landlord has total discretion whether to consent to the works or not. Statute (LTA 1927, s 19(2) specifically) does not apply to this covenant)

366
Q

You act for the tenant of a lease. The lease contains a covenant that the tenant will ‘keep the premises in a good state of repair and condition’.

Your client asks your to explain their repair obligation in respect of their premises. What is your advice?

A) The premises must be kept in the condition required by the landlord

B) The premises must be maintained in the condition they are at the start of the lease

C) The premises must be kept in the condition in which they would be kept by a reasonably minded owner

D) The premises must be put into repair if they are not already and then kept in the condition in which they would be kept by a reasonably minded owner

E) The premises must be renewed if required

A

D) The premises must be put into repair if they are not already and then kept in the condition in which they would be kept by a reasonably minded owner

(This is correct. A covenant to ‘keep’ premises in repair can be very onerous if the premises are not in a good state of repair at the start of the lease, as a covenant to keep the premises in repair also entails an obligation to put them in repair first. Once in repair the Court of Appeal in Proudfoot v Hart (1890) made clear that the premises should be kept in the condition in which they would be kept by a reasonably minded owner, having regard to the character, type and age of the premises)

367
Q

Six years ago a landlord granted a 15 year legal lease to a tenant. In the lease, the landlord covenanted to paint the exterior of the property every 5 years. The landlord failed to paint the exterior last year. Last week, the landlord sold the reversion to a new owner.

Which of the following statements best explains the landlord’s liability regarding the breach of the covenant?

A) The landlord is liable for the breach of covenant because it remains liable for the landlord’s covenants for the entire term of the lease

B) The landlord is not liable for the breach of covenant if the lease contains a clause releasing the landlord from liability upon sale of the reversion

C) The landlord is liable for the breach of covenant because the breach occurred prior to the sale of the reversion

D) The landlord is not liable for the breach of covenant because the burden of the covenant passed to the new owner

E) The landlord is not liable for the breach of covenant because it was automatically released from liability upon sale of the reversion

A

C) The landlord is liable for the breach of covenant because the breach occurred prior to the sale of the reversion

(This is a new lease and as per s 24(1) Landlord and Tenant (Covenants) Act 1995, a landlord is not released from liability for a breach of covenant occurring before the sale of the reversion. Even if the landlord had been released from liability in accordance with s 6 Landlord and Tenant (Covenants) Act 1995, s/he would not be released from liability in this case)

368
Q

A landlord granted a tenant a 2 year legal lease of commercial premises with an annual rent payable quarterly in advance. The landlord’s agent issues a rental demand each quarter. The latest demand was issued last month. The lease does not contain an express forfeiture clause.

The tenant has breached the repair covenant, which the landlord became aware of two weeks ago month.

Which of the following options best explains whether the landlord is able to forfeit and what steps (if any) it needs to take?

A) The landlord is not entitled to forfeit the lease because it has waived its right

B) The landlord is not entitled to forfeit the lease until it has served a s 146 notice

C) The landlord is not entitled to forfeit the lease until it has served a s 146 notice and complied with the Leasehold Property (Repairs) Act 1938

D) The landlord is not entitled to forfeit the lease because there is no express right to forfeit

E) The landlord is not entitled to forfeit until it has served a written demand

A

D) The landlord is not entitled to forfeit the lease because there is no express right to forfeit

(This is correct. A legal lease, which the landlord and tenant have entered into on the facts, must contain an express right to forfeit in the event of tenant breach. A right to forfeit will not be applied on the facts.
If there was a right to forfeit, the landlord would not have waived this on the facts as the rental demand is served before the landlord is aware of the breach. The landlord would need to serve a s 146 notice, but would not need to comply with the Leasehold Property (Repairs) Act 1938 as the lease was only granted for 2 years, so it would not apply)

369
Q

10 years ago, a landlord granted a legal commercial lease for 25 years to a tenant. The lease contains a requirement that the tenant must maintain the premises in a good state of repair. The landlord is in disrepair. The property market is currently slow and the landlord explains it would be hard to find a new tenant.

Which of the following options is the best advice to the landlord as to the remedy it should pursue in the circumstances?

A) The landlord should seek the remedy of specfic performance to force the tenant to carry out the repair

B) The landlord should forfeit the lease

C) The landlord should pursue a claim for damages for breach of repair against the tenant

D) The landlord should pursue the tenant through the Commercial Rent Arrears Recovery (CRAR) regime.

E) The landlord should seek an injunction to stop the breach of covenant by the tenant

A

C) The landlord should pursue a claim for damages for breach of repair against the tenant

(This is correct and the best advice in the circumstances. The landlord does not wish to end the lease so forfeiture is not a good option. An injunction is not the appropriate remedy as it is an order which forces something to stop, which is not relevant to this breach. Specific performance is very rarely ordered against a tenant, so is not the best advice to the landlord. Finally, CRAR is only relevant where there is a beach of a rent covenant)

370
Q

A landlord granted a 5 year legal lease to a tenant. Last week, the landlord and tenant entered into an agreement by deed in which they agreed to end the lease 2 years early on the condition the tenant paid £10,000 to the landlord.

Which of the following option correctly describes how the lease has been brought to an end?

A) The lease has been surrendered

B) The lease has ended by merger

C) The lease has ended by service of a notice to quit

D) The lease has ended by exercise of a break clause

E) The lease has ended by forfeiture

A

A) The lease has been surrendered

(The is correct. Surrender is the handing back of the lease by the tenant to the landlord with the landlord’s consent. This results in premature termination of the lease, which has occurred on the facts. A deed of surrender is often entered into and sometimes a surrender premium will be paid by the tenant to the landlord, which again is the case on the facts)

371
Q

A tenant has sub-let its leased premises in breach of the lease alienation provisions. The landlord would like to forfeit the lease, but it’s agent just sent out the rent demand unaware of the breach.

Which of the following best describes the advice to the landlord about whether it can forfeit in the circumstances?

A) The landlord has waived its right to forfeit, but this will only last until the next rent day when the right will arise again

B) The landlord has waived its right to forfeit and it can never forfeit again because this is a continuing breach

C) The landlord has waived its right to forfeit and it can never again forfeit for this specific breach

D) The landlord has not waived its right to forfeit the lease because the agent did not know about the breach

E) The landlord has not waived its right to forfeit by sending out the rent demand because of the timing of when the demand was sent

A

C) The landlord has waived its right to forfeit and it can never again forfeit for this specific breach

(This is correct. The landlord’s agents has sent out a rent demand. This is an acknowledgement of the continued existence of the lease and evidence of the landlord’s intention that the lease should continue despite the breach. The breach (unauthorised subletting) is classed as a ‘non-continuing’ breach, which means waiver is permanent and it can never again forfeit for this specific breach. It does not matter that the waiver is inadvertent and that the agent was unaware of the breach)

372
Q

A landlord applies for a court order for possession of commercial premises for tenant breach of the rent covenant. The tenant would like to apply for relief as it can now pay the arrears due.

Which of the following best describes whether the tenant is likely to be successful in the circumstances?

A) The court can grant relief under s 146(2) of the LPA 1925 in the circumstances

B) The court will stay the proceedings in the circumstances

C) The court has total discretion whether to grant relief in the circumstances

D) The court cannot grant relief in the circumstances

E) The court will grant relief under its inherent equitable jurisdiction

A

B) The court will stay the proceedings in the circumstances

(This is correct. If, on the landlord suing for possession, the tenant pays into court all arrears and costs before the trial, all further proceedings are stayed. This is the best option as the order has not yet been made)